Patent Law FAQ

This FAQ answers all your questions about patent law, patent procedure, and the patent examination process.

Here’s the complete FAQ:

c Expand All C Collapse All

MPEP 200 - Types and Status of Application; Benefit and Priority (93)

Patent examiners should use the MPEP ยถ 2.01 form paragraph under specific circumstances. The MPEP provides the following guidance:

Examiner Note: […] 2. This form paragraph should only be used if it appears that the application may be a divisional, but a benefit claim has not been properly established.

This means examiners should use this paragraph when:

  • The application appears to contain subject matter that is independent and distinct from a prior application
  • The application seems to qualify as a divisional
  • The applicant has not yet properly claimed the benefit of the prior application’s filing date

The form paragraph serves as a prompt for the applicant to properly establish the benefit claim if they intend the application to be a divisional.

For more information on Divisional application, visit: Divisional application.

A divisional application is often filed in response to a restriction requirement made by the patent examiner. As stated in the MPEP:

A divisional application is often filed as a result of a restriction requirement made by the examiner.

When an examiner determines that an application contains multiple independent and distinct inventions, they may issue a restriction requirement. The applicant can then elect one invention to pursue in the original application and file a divisional application to pursue the non-elected invention(s).

To learn more:

Continued Prosecution Applications (CPAs) are specifically for design patent applications. The MPEP ยถ 2.34 refers to CPAs in the context of design applications.

The MPEP ยถ 2.34 states: “In bracket 2, insert either –continuation– or –divisional–.”

This indicates that a CPA can be filed as either a continuation or a divisional of a prior design application. It’s important to note that CPAs are no longer available for utility or plant patent applications.

For more information on continuation application, visit: continuation application.

For more information on design patents, visit: design patents.

For more information on Divisional application, visit: Divisional application.

To claim the benefit of a prior application’s filing date for a divisional application, applicants must follow specific procedures. The MPEP ยถ 2.01 provides guidance:

Should applicant desire to claim the benefit of the filing date of the prior application, attention is directed to 35 U.S.C. 120, 37 CFR 1.78, and MPEP ยง 211 et seq.

This means applicants should:

  • Review 35 U.S.C. 120 for statutory requirements
  • Consult MPEP ยง 211 for detailed guidance on claiming benefit
  • Follow the procedures outlined in 37 CFR 1.78 for claiming benefit of an earlier-filed application

It’s crucial to properly establish the benefit claim to secure the earlier filing date for the divisional application.

For more information on Divisional application, visit: Divisional application.

For more information on USPTO procedures, visit: USPTO procedures.

When a patent examiner encounters an application that appears to be a divisional but lacks a proper benefit claim, they should follow the guidance provided in MPEP ยถ 2.01. The MPEP states:

“This form paragraph should only be used if it appears that the application may be a divisional, but a benefit claim has not been properly established.”

In such cases, the examiner should:

  1. Recognize that the application may potentially be a divisional based on its content and relationship to a prior application.
  2. Note that the applicant has not properly established a benefit claim to the prior application.
  3. Use the appropriate form paragraph (as mentioned in MPEP ยถ 2.01) to inform the applicant about the potential divisional status and the need to establish a proper benefit claim.
  4. Direct the applicant’s attention to the relevant laws and regulations for claiming benefit, specifically: 35 U.S.C. 120, 37 CFR 1.78, and MPEP ยง 211 et seq.

By following these steps, the examiner ensures that the applicant is aware of the potential divisional status and has the opportunity to properly establish the benefit claim if desired.

For more information on Divisional application, visit: Divisional application.

What is the time limit for filing a divisional application?

There is no specific time limit for filing a divisional application, but there are important considerations that effectively create a practical time frame:

  1. Parent Application Status: The divisional application must be filed while the parent application is still pending. Once the parent application is either abandoned or issued as a patent, it’s no longer possible to file a divisional from it.
  2. Priority Claim: To claim the benefit of the parent application’s filing date, the divisional must be filed within the time periods specified in 35 U.S.C. 120, 121, 365(c), or 386(c).
  3. Patent Term: The 20-year patent term is calculated from the earliest non-provisional U.S. filing date. Filing a divisional later in the parent’s pendency will result in a shorter potential patent term for the divisional.

The MPEP 201.06 states:

A divisional application is often filed as a result of a restriction requirement made by the examiner.

While this doesn’t specify a time limit, it implies that divisional applications are typically filed in response to office actions during the examination process. Applicants should consider filing a divisional promptly after receiving a restriction requirement to ensure they don’t miss any deadlines and to maximize the potential patent term.

For more information on Divisional application, visit: Divisional application.

For more information on filing deadline, visit: filing deadline.

What is the significance of the term ‘carved out’ in relation to divisional applications?

The term ‘carved out’ is significant in understanding the nature of divisional applications. According to MPEP 201.06: ‘A later application for an independent or distinct invention, carved out of a pending application and disclosing and claiming only subject matter disclosed in the earlier or parent application, is known as a divisional application or ‘division.”

The significance of ‘carved out’ includes:

  • It implies that the divisional application is derived from a portion of the parent application.
  • The divisional application focuses on a specific invention or set of claims that are distinct from those in the parent application.
  • The subject matter in the divisional is entirely based on the disclosure in the parent application.
  • No new matter can be added to the divisional application beyond what was disclosed in the parent.

This term emphasizes that while the divisional application is separate, it is intrinsically linked to the parent application’s original disclosure, ensuring that the applicant doesn’t gain an unfair advantage by introducing new subject matter.

For more information on Divisional application, visit: Divisional application.

For more information on invention disclosure, visit: invention disclosure.

For more information on patent filing, visit: patent filing.

What is the significance of the filing date in a divisional application?

The filing date of a divisional application is crucial for several reasons:

  • Benefit of earlier filing date: A divisional application is entitled to the benefit of the filing date of the prior-filed application, as stated in MPEP 201.06(c): ‘A divisional application is entitled to the benefit of the filing date of the prior-filed application if filed before the patenting or abandonment of or termination of proceedings on the prior-filed application.’
  • Prior art considerations: The earlier filing date can be used to overcome prior art references that might otherwise be applicable.
  • Patent term calculations: The filing date may affect the calculation of the patent term.
  • Priority claims: It establishes the basis for any priority claims to foreign applications.

It’s important to note that the divisional application must be filed while the parent application is still pending to claim the benefit of the earlier filing date.

For more information on Divisional application, visit: Divisional application.

For more information on patent term, visit: patent term.

For more information on prior art, visit: prior art.

What is the significance of the ‘original disclosure’ in a divisional application?

The ‘original disclosure’ is crucial in a divisional application because it defines the scope of what can be included in the divisional. According to MPEP 201.06:

‘The disclosure of a divisional application must be the same as the disclosure of the prior-filed application, i.e., the original disclosure of the parent or original nonprovisional application or provisional application.’

This means that no new matter can be added to a divisional application. The divisional must rely entirely on the content disclosed in the original parent application. This ensures that the divisional application doesn’t extend beyond the scope of what was initially invented and disclosed.

For more information on Divisional application, visit: Divisional application.

For more information on new matter, visit: new matter.

Claiming benefit to a prior application is crucial for divisional applications as it allows the divisional to retain the earlier filing date of the parent application. This can be significant for:

  • Establishing priority over potential prior art
  • Extending the potential patent term
  • Maintaining continuity in the patent family

The MPEP ยถ 2.01 states:

“Should applicant desire to claim the benefit of the filing date of the prior application, attention is directed to 35 U.S.C. 120, 37 CFR 1.78, and MPEP ยง 211 et seq.”

To properly claim this benefit, applicants must comply with the requirements set forth in 35 U.S.C. 120, 37 CFR 1.78, and MPEP ยง 211, which include making a proper benefit claim and ensuring continuity of disclosure.

For more information on Divisional application, visit: Divisional application.

For more information on patent law, visit: patent law.

What is the required oath or declaration for a divisional application?

For a divisional application, a new oath or declaration is generally required. However, there are exceptions:

  • If the application is filed on or after September 16, 2012, a copy of the oath or declaration from the prior nonprovisional application may be used, provided that it complies with 37 CFR 1.63, 1.64, or 1.67 for the divisional application.
  • If the divisional application was filed before September 16, 2012, and the prior application was filed on or after September 16, 2012, a copy of the oath or declaration from the prior application can be used in the divisional application.

As stated in MPEP 201.06: “A new oath or declaration is required in a divisional application filed under 37 CFR 1.53(b), regardless of whether a copy of the oath or declaration from the prior nonprovisional application is used in the divisional application, unless the divisional application is filed on or after September 16, 2012.”

To learn more:

What is the relationship between a divisional application and its parent application?

A divisional application is closely related to its parent application, as it stems from the same invention disclosure. According to MPEP 201.06, “A divisional application is a later application for an independent or distinct invention, carved out of a pending application and disclosing and claiming only subject matter disclosed in the earlier or parent application.” This means that:

  • The divisional application contains a subset of the subject matter from the parent application.
  • It claims an independent or distinct invention from what is claimed in the parent application.
  • It cannot introduce new matter beyond what was disclosed in the parent application.
  • It can claim the benefit of the parent application’s filing date.

The relationship allows for separate prosecution of different aspects of an invention while maintaining the priority date of the original disclosure.

To learn more:

What is the relationship between a divisional application and a restriction requirement?

A divisional application is often filed in response to a restriction requirement issued by the USPTO. The MPEP 201.06 explains:

“A later application for an independent or distinct invention, carved out of a pending application and disclosing and claiming only subject matter disclosed in the earlier or parent application, is known as a divisional application or ‘division.'”

The relationship between a divisional application and a restriction requirement is as follows:

  • When an examiner determines that a single application contains two or more independent or distinct inventions, they may issue a restriction requirement.
  • The applicant must then elect one invention to pursue in the original application.
  • The non-elected invention(s) can be pursued in one or more divisional applications.
  • Divisional applications allow applicants to protect multiple inventions disclosed in a single parent application without violating the principle of double patenting.

It’s important to note that while divisional applications are often filed in response to restriction requirements, they can also be filed voluntarily if the applicant recognizes distinct inventions in their application before receiving a restriction requirement.

To learn more:

What is the relationship between a divisional application and a continuation application?

Divisional applications and continuation applications are both types of continuing applications, but they serve different purposes. The MPEP 201.06 provides insight into the nature of divisional applications:

“A later application for an independent or distinct invention, carved out of a pending application and disclosing and claiming only subject matter disclosed in the earlier or parent application, is known as a divisional application or ‘division.'”

In contrast, a continuation application typically contains the same disclosure as the parent application but with different claims. The key differences are:

  • Divisional Application: Filed to pursue distinct inventions that were subject to a restriction requirement in the parent application.
  • Continuation Application: Filed to pursue additional aspects of the same invention disclosed in the parent application.

Both types of applications claim the benefit of the parent application’s filing date, but they differ in their purpose and the scope of claims they can contain. It’s important to choose the appropriate type of continuing application based on the specific circumstances and inventive concepts you wish to protect.

For more information on continuation application, visit: continuation application.

For more information on Divisional application, visit: Divisional application.

What is the purpose of a divisional application under 37 CFR 1.53(b)?

A divisional application under 37 CFR 1.53(b) serves to pursue claims to a distinct invention that was disclosed but not claimed in a parent application. According to the MPEP:

“A divisional application is often filed as a result of a restriction requirement made by the examiner.”

This allows inventors to protect multiple inventions that may have been included in a single original application but were deemed to be separate and distinct by the USPTO. Divisional applications maintain the benefit of the filing date of the parent application for the common subject matter.

For more information on Divisional application, visit: Divisional application.

For more information on patent filing, visit: patent filing.

What is the purpose of a divisional application in patent law?

A divisional application is a type of patent application that allows an inventor to pursue protection for different aspects of an invention that were disclosed but not claimed in a parent application. According to MPEP 201.06:

A later application for an independent or distinct invention, carved out of a pending application and disclosing and claiming only subject matter disclosed in the earlier or parent application, is known as a divisional application or ‘division.’

The main purposes of a divisional application are:

  • To protect multiple inventions disclosed in a single parent application
  • To respond to a restriction requirement issued by the USPTO
  • To pursue broader or different claim scope than what was allowed in the parent application

Divisional applications are filed under 37 CFR 1.53(b) and can benefit from the filing date of the parent application for the common subject matter.

For more information on Divisional application, visit: Divisional application.

For more information on patent law, visit: patent law.

What is the filing date requirement for a divisional application?

A divisional application must be filed before the patenting or abandonment of or termination of proceedings on the original application or on an earlier-filed divisional application directed to the same elected invention. The MPEP states:

A divisional application is often filed as a result of a restriction requirement made by the examiner. The divisional application may be filed under 37 CFR 1.53(b) (or 1.53(d) if the application is a design application) before the patenting or abandonment of or termination of proceedings on the original application or on an earlier-filed divisional application directed to the same elected invention.”

This means that the divisional application must be filed while the parent application is still pending, ensuring continuity in the examination process.

For more information on Divisional application, visit: Divisional application.

For more information on patent procedure, visit: patent procedure.

What is the filing date of a divisional application?

A divisional application is entitled to the filing date of the parent application. According to MPEP 201.06, “A divisional application is entitled to the benefit of the filing date of the prior-filed application if filed as a nonprovisional application that discloses and claims only subject matter disclosed in the prior-filed (parent) application.” This means that the divisional application inherits the filing date of its parent application, provided it meets the necessary requirements.

To learn more:

According to MPEP 201.06(c), the filing date for a divisional or continuation application filed under 37 CFR 1.53(b) is determined as follows:

‘The filing date of an application filed under 37 CFR 1.53(b) is the date on which a specification, with or without claims, is received in the Office.’

This means that the filing date is established when the USPTO receives the specification, regardless of whether claims are included. It’s important to note that other application components, such as drawings or fees, are not required to secure the filing date.

To learn more:

To learn more:

Incorporating by reference a prior application in a continuation or divisional application can provide important benefits:

  1. It allows the applicant to amend the continuing application to include subject matter from the prior application without the need for a petition, provided the continuing application is entitled to a filing date.
  2. For applications filed on or after September 21, 2004, it can help recover inadvertently omitted material.

The MPEP states: An applicant may incorporate by reference the prior application by including, in the continuing application-as-filed, an explicit statement that such specifically enumerated prior application or applications are “hereby incorporated by reference.” The statement must appear in the specification.

It’s important to note that an incorporation by reference statement added after an application’s filing date is not effective because no new matter can be added to an application after its filing date (see 35 U.S.C. 132(a)).

For more information on 37 cfr 1.57, visit: 37 cfr 1.57.

For more information on continuation application, visit: continuation application.

For more information on Divisional application, visit: Divisional application.

For more information on incorporation by reference, visit: incorporation by reference.

A preliminary amendment filed with a continuation or divisional application can have significant effects:

  1. If present on the filing date, it is considered part of the original disclosure.
  2. It must comply with the requirements of 37 CFR 1.121 for amendments.
  3. The Office may require a substitute specification for extensive preliminary amendments.
  4. The application will be classified and assigned based on the claims that will be before the examiner after entry of the preliminary amendment.

The MPEP states: Any preliminary amendment that is present on the filing date of an application filed under 37 CFR 1.53(b) is part of the original disclosure.

However, applicants should be cautious about introducing new matter through a preliminary amendment. The MPEP advises: Where a copy of the oath or declaration from a prior application was filed in a continuation or divisional application, if the examiner determines that new matter is present relative to the prior application, the examiner should so notify the applicant in the next Office action (preferably the first Office action). The examiner should require applicant to delete the benefit claim or redesignate the application as a continuation-in-part.

For more information on continuation application, visit: continuation application.

For more information on Divisional application, visit: Divisional application.

For more information on new matter, visit: new matter.

What is the difference between a divisional and a continuation application?

While both divisional and continuation applications are types of continuing applications, they serve different purposes:

  • Divisional Application: Filed to pursue claims to an invention that was not elected in a previous application due to a restriction requirement.
  • Continuation Application: Filed to pursue additional claims to the same invention disclosed in the parent application.

The MPEP 201.06(c) states: “A divisional application is often filed as a result of a restriction requirement made by the examiner.” In contrast, a continuation typically doesn’t arise from a restriction requirement but rather from the applicant’s desire to pursue additional aspects of the same invention.

Both types can be filed under 37 CFR 1.53(b) and may claim the benefit of the parent application’s filing date.

For more information on continuation application, visit: continuation application.

For more information on Divisional application, visit: Divisional application.

A continuation application and a divisional application are both types of continuing applications, but they serve different purposes:

  • Continuation application: Used to pursue additional claims to an invention disclosed in a prior application. It contains the same disclosure as the parent application.
  • Divisional application: Used to pursue claims to an invention that was disclosed but not claimed in a prior application. It typically results from a restriction requirement in the parent application.

As stated in MPEP 201.07: “A continuation application is an application for the invention(s) disclosed in a prior-filed copending nonprovisional application, international application designating the United States, or international design application designating the United States.” In contrast, a divisional application focuses on a distinct invention that was not claimed in the parent application.

To learn more:

A divisional application is defined in MPEP 201.06 as follows:

‘A later application for a distinct or independent invention, carved out of a pending application and disclosing and claiming only subject matter disclosed in the earlier or parent application, is known as a divisional application or ‘division.’

This definition highlights that a divisional application:

  • Is filed after an initial application
  • Focuses on a distinct invention
  • Only includes subject matter from the original application
  • Is ‘carved out’ of the pending parent application

For more information on Divisional application, visit: Divisional application.

For more information on patent application types, visit: patent application types.

Filing a divisional application allows an applicant to pursue protection for distinct inventions disclosed in a parent application. Key benefits include:

  • Ability to claim different aspects or embodiments of the invention
  • Potential for broader patent coverage
  • Preservation of priority date for the disclosed subject matter
  • Opportunity to pursue claims that may have been restricted in the parent application

The MPEP indicates: A divisional application is often filed as a result of a restriction requirement made by the examiner. This suggests that divisional applications provide a way to pursue protection for inventions that were not elected in the parent application due to a restriction requirement.

For more information on Divisional application, visit: Divisional application.

For more information on patent strategy, visit: patent strategy.

What is the benefit of filing a divisional application under 37 CFR 1.53(b)?

Filing a divisional application under 37 CFR 1.53(b) offers several benefits:

  • It allows you to pursue claims to an invention that was not elected in a previous application due to a restriction requirement.
  • You can file the divisional application without a new oath or declaration if you meet certain conditions.
  • The divisional application can claim the benefit of the filing date of the prior application.

As stated in the MPEP: “A divisional application is often filed as a result of a restriction requirement made by the examiner.” This means you can protect additional aspects of your invention that weren’t covered in the original application.

For more information on Divisional application, visit: Divisional application.

For more information on patent filing, visit: patent filing.

A divisional application is a later application for an independent or distinct invention, carved out of a nonprovisional application. It discloses and claims only subject matter disclosed in the earlier or parent application. As stated in MPEP 201.06:

A later application for an independent or distinct invention, carved out of a nonprovisional application (including a nonprovisional application resulting from an international application or international design application), an international application designating the United States, or an international design application designating the United States and disclosing and claiming only subject matter disclosed in the earlier or parent application, is known as a divisional application.

To learn more:

A divisional application under 37 CFR 1.53(b) is a type of continuation application that is filed to pursue a distinct invention that was disclosed but not claimed in a prior nonprovisional application. According to the MPEP:

‘A divisional application is often filed as a result of a restriction requirement made by the examiner.’

Divisional applications allow inventors to protect multiple inventions disclosed in a single parent application by filing separate applications for each invention.

For more information on continuation application, visit: continuation application.

For more information on Divisional application, visit: Divisional application.

A divisional application is a type of patent application that claims subject matter that is independent and distinct from that claimed in a prior application. It typically arises when an original application contains multiple inventions, and the Patent Office requires the applicant to restrict the claims to a single invention.

According to MPEP ยถ 2.01, a divisional application:

“appears to claim only subject matter directed to an invention that is independent and distinct from that claimed in the prior application, and names the inventor or at least one joint inventor named in the prior application.”

To claim the benefit of the filing date of the prior application, applicants must comply with 35 U.S.C. 120, 37 CFR 1.78, and MPEP ยง 211 et seq.

For more information on Divisional application, visit: Divisional application.

For more information on patent law, visit: patent law.

For more information on USPTO, visit: USPTO.

A continuation or divisional application is a type of continuing application filed under 37 CFR 1.53(b). According to the MPEP, An application filed under 37 CFR 1.53(b) may be an original or a reissue, a continuation, a divisional, a continuation-in-part, or a substitute. These applications are filed under 35 U.S.C. 111(a) and can be for utility, design, plant, or reissue patents.

To learn more:

To learn more:

What happens to the priority date in a divisional application?

The priority date of a divisional application remains the same as its parent application, provided that the proper procedures are followed. According to MPEP 201.06:

‘A later-filed application which is filed before the patenting or abandonment of or termination of proceedings on an earlier-filed application and which is an application for an invention which is disclosed and claimed in the earlier-filed application is an application for a ‘divisional’ of the earlier-filed application.’

This means that:

  • The divisional application retains the priority date of the parent application for the subject matter disclosed in the parent.
  • This is crucial for establishing the effective filing date for prior art purposes under 35 U.S.C. 100(i).
  • The applicant must ensure that a proper reference to the parent application is made in the divisional to maintain this priority.

Maintaining the original priority date is one of the key advantages of filing a divisional application, as it allows the applicant to protect different aspects of their invention without losing the benefit of the earlier filing date.

For more information on Divisional application, visit: Divisional application.

For more information on effective filing date, visit: effective filing date.

What happens to the parent application when a divisional application is filed?

When a divisional application is filed, the parent application continues to be prosecuted independently. The filing of a divisional application does not automatically affect the status or prosecution of the parent application.

As stated in MPEP 201.06: “The divisional application may be filed either while the original application is still pending or after it has been patented.” This means that:

  • If the parent application is pending, it remains pending and continues through the examination process.
  • If the parent application has already been granted as a patent, it remains a valid patent.

It’s important to note that while the divisional application is independent, it must contain claims to a different invention than the parent application. The examiner will ensure that there is no double patenting between the parent and divisional applications.

Additionally, if the divisional application is filed in response to a restriction requirement in the parent application, the applicant should consider canceling the claims in the parent application that are now being pursued in the divisional to avoid potential issues with duplicate claims.

For more information on Divisional application, visit: Divisional application.

For more information on parent application, visit: parent application.

For more information on patent prosecution, visit: patent prosecution.

Terminal disclaimers filed in a parent application do not automatically carry over to a divisional application. The MPEP 201.06 states:

‘A terminal disclaimer filed to obviate a nonstatutory double patenting rejection in a parent application does not carry over to a divisional application. The applicant filing the divisional application must determine whether a new terminal disclaimer needs to be filed in the divisional application.’

This means:

  • Terminal disclaimers from the parent application are not automatically applied to the divisional
  • The applicant must assess whether a new terminal disclaimer is necessary for the divisional application
  • If required, a new terminal disclaimer must be filed specifically for the divisional application

It’s important for applicants to review potential double patenting issues in the divisional application and file new terminal disclaimers if needed to overcome any rejections.

To learn more:

What happens to continuation or divisional applications filed after a CPA in a design application?

Continuation or divisional applications filed after a Continued Prosecution Application (CPA) in a design application are treated differently than those filed after a CPA in a utility or plant application. According to MPEP 201.06(d):

Any continuation or divisional application filed under 37 CFR 1.53(b) after the filing of a CPA of a design application will be assigned a new application number and will reflect the filing date of the 37 CFR 1.53(b) application.

This means that unlike utility or plant applications, where a continuation or divisional application would get the benefit of the CPA’s filing date, in design applications, these subsequent applications are treated as new applications with their own filing dates.

For more information on continuation application, visit: continuation application.

For more information on CPA, visit: CPA.

For more information on Divisional application, visit: Divisional application.

Affidavits or declarations from a prior application do not automatically become part of a continuation or divisional application filed under 37 CFR 1.53(b). If an applicant wants to rely on these documents, they must take specific actions:

  1. Make remarks of record in the new application referring to the affidavit or declaration.
  2. Include a copy of the original affidavit or declaration filed in the prior application.

The MPEP states: Affidavits or declarations, such as those submitted under 37 CFR 1.130, 1.131 and 1.132 filed during the prosecution of the prior nonprovisional application do not automatically become a part of a continuation or divisional application filed under 37 CFR 1.53(b). Where it is desired to rely on an earlier filed affidavit or declaration, the applicant should make such remarks of record in the 37 CFR 1.53(b) application and include a copy of the original affidavit or declaration filed in the prior nonprovisional application.

It’s important to note that this rule doesn’t apply to continued prosecution applications (CPAs) filed under 37 CFR 1.53(d), where affidavits and declarations do automatically become part of the new application.

For more information on continuation application, visit: continuation application.

For more information on Divisional application, visit: Divisional application.

Introducing new matter in a divisional application is not allowed and can have serious consequences. According to MPEP 201.06:

“A divisional application is a later application for an independent or distinct invention, carved out of a pending application and disclosing and claiming only subject matter disclosed in the earlier or parent application.”

This means that a divisional application must not contain any new matter beyond what was disclosed in the parent application. If new matter is introduced:

  • The new matter will not be entitled to the benefit of the parent application’s filing date.
  • Claims relying on the new matter may be rejected under 35 U.S.C. 112(a) for lack of written description or enablement.
  • The application may lose its status as a proper divisional, potentially affecting its ability to overcome certain types of double patenting rejections.

To avoid these issues:

  1. Carefully review the divisional application before filing to ensure it only contains subject matter from the parent application.
  2. If new subject matter is necessary, consider filing it as a continuation-in-part (CIP) application instead of a divisional.
  3. Consult with a patent attorney or agent to ensure compliance with MPEP guidelines.

For more information on Divisional application, visit: Divisional application.

For more information on new matter, visit: new matter.

For more information on patent filing, visit: patent filing.

If you submit an amendment to refer to a Continued Prosecution Application (CPA) as a continuation or divisional application, it will be denied entry by the patent examiner. The MPEP ยถ 2.34 states:

“Thus, there is no need to amend the first sentence(s) of the specification to refer back to the prior application and any such amendment shall be denied entry.”

This means that the patent examiner will not accept or incorporate such amendments into the application. The CPA itself serves as the necessary reference to the prior application, making additional amendments unnecessary and inappropriate.

For more information on continuation application, visit: continuation application.

For more information on CPA, visit: CPA.

For more information on Divisional application, visit: Divisional application.

For more information on patent examination, visit: patent examination.

What happens if a divisional application is filed after the parent application is patented or abandoned?

If a divisional application is filed after the parent application has been patented or abandoned, it will not be entitled to the benefit of the parent application’s filing date. The MPEP states:

A divisional application is often filed as a result of a restriction requirement made by the examiner. The divisional application may be filed under 37 CFR 1.53(b) (or 1.53(d) if the application is a design application) before the patenting or abandonment of or termination of proceedings on the original application or on an earlier-filed divisional application directed to the same elected invention.

Filing a divisional application after the parent application has been patented or abandoned means:

  • The application will not be considered a proper divisional application
  • It will not receive the benefit of the parent application’s filing date
  • It may be treated as a new, independent application
  • Prior art that was not applicable to the parent application may now be applicable
  • The application may face potential statutory bars under 35 U.S.C. 102

To avoid these issues, it’s crucial to file divisional applications before the patenting, abandonment, or termination of proceedings on the parent application.

For more information on Divisional application, visit: Divisional application.

For more information on late filing, visit: late filing.

For more information on patent procedure, visit: patent procedure.

If a continuation or divisional application contains new matter not disclosed in the prior-filed application, the applicant will be required to either delete the benefit claim or change the relationship to a continuation-in-part application. This is because continuation and divisional applications cannot include new matter.

As stated in MPEP ยถ 2.10.01:

“Applicant is required to delete the benefit claim or change the relationship (continuation or divisional application) to continuation-in-part because this application contains the following matter not disclosed in the prior-filed application:”

This form paragraph is used by examiners when they identify new matter in an application claiming benefit under 35 U.S.C. 120, 121, 365(c), or 386(c).

For more information on continuation application, visit: continuation application.

For more information on Divisional application, visit: Divisional application.

For more information on new matter, visit: new matter.

For more information on patent examination, visit: patent examination.

The MPEP 201.06(c) outlines the requirements for filing a divisional or continuation application without an executed oath or declaration:

’37 CFR 1.63(d) provides that a newly executed oath or declaration is not required in a continuation or divisional application filed by all or by fewer than all of the inventors named in a prior nonprovisional application containing an oath or declaration as required by 37 CFR 1.63, provided that a copy of the executed oath or declaration filed in the prior application is submitted for the continuation or divisional application and the specification and drawings filed in the continuation or divisional application contain no matter that would have been new matter in the prior application.’

To file without a new oath or declaration:

  • Submit a copy of the executed oath or declaration from the prior application
  • Ensure the new application contains no new matter
  • File a copy of the specification and drawings from the prior application

This procedure allows for expedited filing while maintaining the integrity of the oath or declaration requirements.

To learn more:

To learn more:

To file a Continued Prosecution Application (CPA) in a design application, the following requirements must be met:

  • The application must be a design application.
  • The prior application must be a nonprovisional application.
  • The application must be filed before the earliest of:
    • Payment of the issue fee on the prior application, unless a petition under ยง 1.313(c) is granted in the prior application;
    • Abandonment of the prior application; or
    • Termination of proceedings on the prior application.

As stated in the MPEP 201.06(d): “A continuation or divisional application that is a nonprovisional application may be filed as a CPA under 37 CFR 1.53(d) if the prior nonprovisional application is a design application that is complete as defined by 37 CFR 1.51(b).”

For more information on continuation application, visit: continuation application.

For more information on CPA, visit: CPA.

For more information on Divisional application, visit: Divisional application.

For more information on filing requirements, visit: filing requirements.

For an application to be considered a divisional, it must meet several requirements as outlined in MPEP ยถ 2.01:

  1. It must disclose and claim only subject matter disclosed in the prior application.
  2. It must claim subject matter that is independent and distinct from that claimed in the prior application.
  3. It must name the inventor or at least one joint inventor named in the prior application.

The MPEP states:

This application, which discloses and claims only subject matter disclosed in prior Application No.[1], filed [2], appears to claim only subject matter directed to an invention that is independent and distinct from that claimed in the prior application, and names the inventor or at least one joint inventor named in the prior application.

Meeting these requirements helps ensure that the divisional application is properly derived from the prior application while claiming a distinct invention.

For more information on Divisional application, visit: Divisional application.

For more information on USPTO, visit: USPTO.

A divisional application has several key characteristics as outlined in MPEP ยถ 2.01:

  1. Independent and Distinct Subject Matter: It ‘appears to claim only subject matter directed to an invention that is independent and distinct from that claimed in the prior application.’
  2. Disclosure in Prior Application: It ‘discloses and claims only subject matter disclosed in prior Application No.[1],filed[2].’
  3. Inventor Continuity: It ‘names the inventor or at least one joint inventor named in the prior application.’
  4. Potential Benefit Claim: It may claim the benefit of the filing date of the prior application, subject to meeting certain requirements.

These characteristics ensure that divisional applications properly separate distinct inventions while maintaining a connection to the original disclosure and inventorship.

For more information on Divisional application, visit: Divisional application.

The filing requirements for a divisional application under 37 CFR 1.53(b) are similar to those for a new nonprovisional application. According to the MPEP:

‘A continuation or divisional application may be filed under 35 U.S.C. 111(a) using the procedures set forth in 37 CFR 1.53(b), by providing: (1) a specification complying with 35 U.S.C. 112, including a claim or claims; (2) drawings, where necessary; and (3) the inventorship named on filing.’

Additionally, you must include:

  • The appropriate filing fees
  • An Application Data Sheet (ADS) specifying that the application is a divisional
  • A proper benefit claim to the parent application

It’s important to note that while a new oath or declaration may not be required under 37 CFR 1.63(d), you must still comply with all other filing requirements.

For more information on Divisional application, visit: Divisional application.

For more information on drawings, visit: drawings.

For more information on filing requirements, visit: filing requirements.

For more information on inventorship, visit: inventorship.

To file a CPA for a design patent application, the following requirements must be met:

  • The prior application must be a design application that is complete under 37 CFR 1.51(b)
  • The CPA must be filed before the earliest of: payment of the issue fee, abandonment, or termination of proceedings in the prior application
  • The CPA must be filed with a request on a separate paper
  • The proper filing fee, search fee, and examination fee must be paid
  • The CPA must only disclose and claim subject matter disclosed in the prior application

As stated in the MPEP: “The filing date of a CPA is the date on which a request on a separate paper for an application under this paragraph is filed.”

For more information on continuation application, visit: continuation application.

For more information on design patents, visit: design patents.

For more information on Divisional application, visit: Divisional application.

Filing a CPA for a design patent application offers several benefits:

  • Minimal filing requirements compared to a regular continuation application
  • No new filing receipt is normally issued
  • Faster processing time, as it uses the same application number and file wrapper
  • For examination priority, CPAs are treated as “amended” applications rather than “new” applications, potentially resulting in faster examination
  • The prior application is automatically abandoned, simplifying the process

As stated in the MPEP: “A CPA has a number of advantages compared to a continuation or divisional application filed under 37 CFR 1.53(b). For example, the papers required to be filed in the U.S. Patent and Trademark Office in order to secure a filing date under 37 CFR 1.53(d) are minimal compared to 37 CFR 1.53(b).”

For more information on continuation application, visit: continuation application.

For more information on design patents, visit: design patents.

For more information on Divisional application, visit: Divisional application.

The method for making a specific reference to a prior-filed application depends on when the application was filed:

  • For applications filed before September 16, 2012: The specific reference must be included in the first sentence(s) of the specification following the title or in an application data sheet (ADS) in compliance with pre-AIA 37 CFR 1.76.
  • For applications filed on or after September 16, 2012: The specific reference must be included in an ADS in compliance with 37 CFR 1.76.

Additionally, for benefit claims under 35 U.S.C. 120, 121, 365(c), or 386(c), the reference must include the relationship (i.e., continuation, divisional, or continuation-in-part) of the applications.

It’s important to note that for continued prosecution applications (CPAs) filed under 37 CFR 1.53(d) (design applications under 35 U.S.C. chapter 16 only), a specific reference in the specification or ADS is not required and may not be made. For these applications, The specific reference requirement of 35 U.S.C. 120 is met by the transmittal request for the CPA which is considered to be part of the CPA.

For more information on application data sheet, visit: application data sheet.

For more information on continuation application, visit: continuation application.

For more information on Divisional application, visit: Divisional application.

For more information on specific reference, visit: specific reference.

The presence of new matter in a continuation or divisional application can significantly affect its filing date. If new matter is found, the application may lose the benefit of the earlier filing date for the new subject matter.

According to MPEP ยถ 2.10.01:

“Applicant is required to delete the benefit claim or change the relationship (continuation or divisional application) to continuation-in-part because this application contains the following matter not disclosed in the prior-filed application:”

If the application is changed to a continuation-in-part, the new matter will have the filing date of the current application, while the previously disclosed matter retains the earlier filing date. This can have important implications for determining prior art and patent term.

For more information on continuation application, visit: continuation application.

For more information on continuation-in-part application, visit: continuation-in-part application.

For more information on Divisional application, visit: Divisional application.

For more information on new matter, visit: new matter.

How does the oath or declaration work in divisional applications?

In divisional applications, the oath or declaration process is simplified to reduce the burden on applicants. According to MPEP 201.06(c):

37 CFR 1.63(d) provides that a newly executed oath or declaration is not required in a divisional application filed under 37 CFR 1.53(b) that contains the subject matter described in the prior nonprovisional application, provided a copy of the executed oath or declaration filed in the prior nonprovisional application is submitted.

Key points about the oath or declaration in divisional applications:

  • A new oath or declaration is not required if the divisional application contains the same subject matter as the parent application.
  • A copy of the executed oath or declaration from the parent application can be submitted.
  • The copy must show the USPTO seal or stamp indicating it was received.
  • If the inventorship changes, a new oath or declaration may be required.

This provision under 37 CFR 1.63(d) streamlines the filing process for divisional applications while ensuring proper inventorship documentation.

For more information on declaration, visit: declaration.

For more information on Divisional application, visit: Divisional application.

For more information on oath, visit: oath.

How does the oath or declaration requirement differ for divisional applications?

For divisional applications, the oath or declaration requirement can be simplified under certain conditions. According to MPEP 201.06(c):

“The divisional application may be filed under 37 CFR 1.53(b) using all or part of a copy of the oath or declaration filed in the prior nonprovisional application under 37 CFR 1.63(d), as long as the oath or declaration is still applicable to the subject matter of the divisional application.”

This means that if the original oath or declaration from the parent application still covers the subject matter in the divisional application, it can be reused. However, if new matter is introduced or the inventors change, a new oath or declaration may be required.

For more information on Divisional application, visit: Divisional application.

For more information on oath or declaration, visit: oath or declaration.

For more information on patent filing, visit: patent filing.

Micro entity status does not automatically carry over to continuation or divisional applications. A new certification is required for each new application:

  1. A new assertion of entitlement to micro entity status is required for any continuation, divisional, or continuation-in-part application.
  2. This applies even if micro entity status was properly claimed in the parent application.
  3. The new certification must be filed in the continuing application.

The MPEP states: The refiling of an application under 37 CFR 1.53 as a continuation, divisional, or continuation-in-part application (including a continued prosecution application under 37 CFR 1.53(d) (design applications only)), requires a new certification of entitlement to micro entity status in the continuing application.

It’s important to note that this requirement for a new certification applies specifically to micro entity status. For small entity status, a new assertion is also required, but the rules are slightly different.

For more information on continuation application, visit: continuation application.

For more information on continuation-in-part application, visit: continuation-in-part application.

For more information on Divisional application, visit: Divisional application.

The inventorship of a divisional application must include at least one inventor named in the parent application, but it doesn’t have to be identical. The MPEP states:

The inventorship in the divisional application must include at least one inventor named in the prior-filed application, and the divisional application must claim the benefit of the prior-filed application under 35 U.S.C. 120, 121, 365(c), or 386(c).

If the divisional application is filed with fewer inventors than the parent application, a statement may be filed requesting deletion of the names of the additional inventors. For applications filed on or after September 16, 2012, the statement must be filed with the divisional application and signed by a party set forth in 37 CFR 1.33(b).

To learn more:

The filing date of a divisional application can affect the patent term in several ways:

  1. 20-year term: The 20-year patent term is calculated from the earliest U.S. filing date to which the divisional application claims priority. This means that the term of a patent issued on a divisional application will generally be shorter than that of its parent application.
  2. Patent Term Adjustment (PTA): The divisional application may be eligible for PTA based on delays in its own prosecution, but not for delays in the prosecution of its parent application.
  3. Terminal Disclaimer: If there’s a risk of obviousness-type double patenting, a terminal disclaimer may be required, potentially limiting the term of the divisional patent to that of the parent patent.

The MPEP 201.06(c) does not directly address patent term, but it’s important to consider these factors when filing a divisional application. For more details on patent term, refer to MPEP 2701.

For more information on Divisional application, visit: Divisional application.

For more information on patent term, visit: patent term.

For more information on patent term adjustment, visit: patent term adjustment.

The disclosure requirements for claiming the benefit of provisional and nonprovisional applications differ in some aspects:

Provisional Applications (35 U.S.C. 119(e)):

For a nonprovisional application to claim the benefit of a provisional application:

  • The written description and drawings (if any) of the provisional application must adequately support and enable the subject matter claimed in the nonprovisional application.
  • A claim is not required in the provisional application.

Nonprovisional Applications (35 U.S.C. 120, 121, 365(c), or 386(c)):

For continuation and divisional applications:

  • The disclosure must be the same as the prior-filed application.
  • No new matter can be added.

For continuation-in-part applications:

  • New matter may be included.
  • Only claims supported by the prior-filed application are entitled to the earlier filing date.

As stated in MPEP 211.05:

“Under 35 U.S.C. 120, a claim in a U.S. application is entitled to the benefit of the filing date of an earlier filed U.S. application if the subject matter of the claim is disclosed in the manner provided by 35 U.S.C. 112(a) except for the best mode requirement, in the earlier filed application.”

This requirement applies to both provisional and nonprovisional applications, ensuring that the earlier application provides proper support for the claimed invention.

To learn more:

The treatment of new matter varies depending on the type of continuing application:

  • Continuation and Divisional Applications: Must not contain any new matter. The disclosure must be the same as the prior application.
  • Continuation-in-Part (CIP) Applications: Can include new matter not disclosed in the parent application.
  • Continued Prosecution Applications (CPAs): Cannot contain new matter.

For continuation and divisional applications, the MPEP states: “The disclosure presented in the continuation must not include any subject matter which would constitute new matter if submitted as an amendment to the parent application.”

For CIPs, the MPEP notes: “A continuation-in-part is an application… repeating some substantial portion or all of the prior-filed application and adding matter not disclosed in the prior-filed application.”

If new matter is found in a continuation or divisional application, the examiner will require the applicant to delete the benefit claim or change the application to a continuation-in-part. For CPAs, any new matter introduced will be required to be canceled.

It’s important to note that claims in a CIP application that rely on new matter will only be entitled to the filing date of the CIP, not the parent application’s filing date.

For more information on continuation application, visit: continuation application.

For more information on continuation-in-part application, visit: continuation-in-part application.

For more information on CPA, visit: CPA.

For more information on Divisional application, visit: Divisional application.

For more information on new matter, visit: new matter.

Filing a continuation or divisional application under 37 CFR 1.53(b) does not automatically abandon the prior application. The status of the prior application depends on the actions taken by the applicant:

  1. The prior application remains pending unless expressly abandoned.
  2. To expressly abandon the prior application, a separate paper must be filed and signed in accordance with 37 CFR 1.138.
  3. If a notice of allowance has been issued in the prior application, it can become abandoned by nonpayment of the issue fee.

The MPEP states: Under 37 CFR 1.53(b) practice, the prior nonprovisional application is not automatically abandoned upon filing of the continuing application. If the prior nonprovisional application is to be expressly abandoned, such a paper must be signed in accordance with 37 CFR 1.138.

It’s important to note that if the issue fee has been paid in the prior application, a petition to withdraw the application from issue must be filed before it can be abandoned, even if a continuing application has been filed.

For more information on continuation application, visit: continuation application.

For more information on Divisional application, visit: Divisional application.

Patent examiners carefully compare the content of the continuation or divisional application with the prior-filed application to identify any new matter. If they find information not disclosed in the prior application, they will issue a notice to the applicant.

The MPEP ยถ 2.10.01 instructs examiners:

“In bracket 1, provide an example of the matter not disclosed in the prior-filed application.”

This means the examiner must specifically point out the new matter found in the application, allowing the applicant to understand and address the issue.

For more information on continuation application, visit: continuation application.

For more information on Divisional application, visit: Divisional application.

For more information on new matter, visit: new matter.

For more information on patent examination, visit: patent examination.

Patent examiners use specific criteria to determine if an application may qualify as a divisional. According to MPEP ยถ 2.01, an examiner looks for the following indicators:

  1. The application claims only subject matter disclosed in a prior application.
  2. The claimed subject matter appears to be independent and distinct from that claimed in the prior application.
  3. At least one inventor from the prior application is named in the current application.

The MPEP provides guidance to examiners:

“This form paragraph should only be used if it appears that the application may be a divisional, but a benefit claim has not been properly established.”

If these conditions are met, but a proper benefit claim hasn’t been made, the examiner may use this paragraph to notify the applicant of the possible divisional status and the need to establish a benefit claim if desired.

For more information on Divisional application, visit: Divisional application.

For more information on patent examination, visit: patent examination.

For more information on USPTO, visit: USPTO.

To claim the benefit of a prior application’s filing date for a divisional application, applicants must follow specific procedures outlined in patent law and regulations. The MPEP ยถ 2.01 states:

Should applicant desire to claim the benefit of the filing date of the prior application, attention is directed to 35 U.S.C. 120, 37 CFR 1.78, and MPEP ยง 211 et seq.

This means applicants should:

  • Ensure compliance with 35 U.S.C. 120 requirements for continuity and disclosure
  • Follow the procedures outlined in 37 CFR 1.78 for claiming benefit
  • Consult MPEP ยง 211 for detailed guidance on benefit claims

It’s crucial to properly establish the benefit claim to secure the earlier filing date for the divisional application.

For more information on Divisional application, visit: Divisional application.

A terminal disclaimer filed in a parent application can affect a divisional application in specific ways. According to MPEP 201.06:

‘The divisional application must be directed to subject matter described and claimed in the prior nonprovisional application.’

This means that if a terminal disclaimer was filed in the parent application, it may impact the divisional application as follows:

  • Inheritance of Terminal Disclaimer: Generally, a terminal disclaimer filed in the parent application does not automatically apply to the divisional application.
  • Potential Double Patenting: If the claims in the divisional application are not patentably distinct from those in the parent application, a new terminal disclaimer may be required in the divisional to overcome a double patenting rejection.
  • Examination Considerations: Examiners will consider the relationship between the parent and divisional applications when determining if a terminal disclaimer is necessary.

It’s important to consult with a patent attorney or agent to understand the specific implications of terminal disclaimers on your divisional application, as each case can have unique considerations.

To learn more:

How does a restriction requirement lead to a divisional application?

A restriction requirement often leads to the filing of a divisional application. This process occurs as follows:

  1. An examiner issues a restriction requirement when they believe the original application contains claims to two or more independent or distinct inventions.
  2. The applicant must then elect one invention to pursue in the original application.
  3. The non-elected inventions can be pursued in one or more divisional applications.

As stated in MPEP 201.06:

“The inventions embodied in the claims of a divisional application are generally further characterized as (1) independent inventions or (2) related inventions which are distinct.”

This means that the divisional application allows the applicant to protect inventions that were disclosed but not pursued in the original application due to the restriction requirement.

For more information on Divisional application, visit: Divisional application.

For more information on patent examination, visit: patent examination.

How does a divisional application differ from other continuing applications?

A divisional application is a unique type of continuing application that focuses on a distinct invention separated from a parent application. According to the MPEP:

A divisional application is one filed for an invention which has been disclosed and claimed, but not allowed, in a prior nonprovisional application or international application designating the United States. It is a later application for an independent and distinct invention carved out of a pending application. (MPEP 201.02)

Key differences of a divisional application include:

  • It is filed in response to a restriction requirement in the parent application
  • It claims only subject matter that was disclosed but not claimed in the parent application
  • It does not add new matter
  • It allows the applicant to pursue protection for inventions that were deemed distinct in the original application

Unlike continuations or CIPs, divisional applications are specifically for pursuing separate inventions identified during the examination of the parent application.

To learn more:

Divisional applications and provisional applications are distinct types of patent applications with different purposes and characteristics:

  • Divisional Application: Claims subject matter from a prior non-provisional application that is independent and distinct from the original claims.
  • Provisional Application: A temporary application that establishes a priority date but does not mature into an issued patent.

The MPEP explicitly states in MPEP ยถ 2.01:

“An application claiming the benefit of a provisional application under 35 U.S.C. 119(e) should not be called a ‘divisional’ of the prior application.”

This distinction is important because divisional applications claim the benefit under 35 U.S.C. 120, while provisional applications are claimed under 35 U.S.C. 119(e).

For more information on Divisional application, visit: Divisional application.

For more information on patent law, visit: patent law.

For more information on provisional application, visit: provisional application.

For more information on USPTO, visit: USPTO.

A divisional application discloses and claims only subject matter disclosed in the parent application, while a continuation-in-part (CIP) application may include new matter. The MPEP states:

A continuation-in-part application should not be designated as a divisional application. The Court of Appeals for the Federal Circuit has concluded that the protection of the third sentence of 35 U.S.C. 121 does not extend to continuation-in-part applications, stating that ‘the protection afforded by section 121 to applications (or patents issued therefrom) filed as a result of a restriction requirement is limited to divisional applications.’

This means that divisional applications have certain protections against double patenting rejections that CIPs do not have.

To learn more:

A divisional application does not extend the patent term beyond that of the parent application. The term of a patent issuing from a divisional application is calculated from the filing date of the earliest application for which a benefit is claimed under 35 U.S.C. 120, 121, 365(c), or 386(c), excluding any benefit claims to provisional applications.

This means that while you can pursue additional patent protection through a divisional application, you don’t get extra time beyond the original 20-year term from the earliest non-provisional filing date.

It’s important to note that patent term adjustments may apply independently to divisional applications based on delays in their own prosecution.

To learn more:

A divisional application’s patent term adjustment (PTA) is calculated independently from its parent application. The MPEP 201.06 provides guidance on this matter:

‘A divisional application will receive its own patent term adjustment under 35 U.S.C. 154(b), which will be calculated based on the prosecution of the divisional application itself.’

Key points to understand:

  • The divisional application’s PTA is based solely on its own prosecution history
  • Any delays or adjustments from the parent application do not affect the divisional’s PTA
  • The PTA calculation starts from the filing date of the divisional application
  • Applicants should monitor the prosecution of the divisional application to ensure accurate PTA calculation

For more information on patent term adjustment, refer to MPEP 2730.

To learn more:

To learn more:

37 CFR 1.63(d) provides a simplified process for submitting an oath or declaration in a divisional application. The MPEP states:

’37 CFR 1.63(d) provides that a newly executed oath or declaration is not required in a continuation or divisional application filed by all or by fewer than all of the inventors named in a prior nonprovisional application containing a signed oath or declaration.’

This means that if the divisional application is filed by the same inventor(s) as the parent application, and there’s no new matter added, the oath or declaration from the parent application can be used for the divisional application. This simplifies the filing process and reduces paperwork.

For more information on continuation application, visit: continuation application.

For more information on Divisional application, visit: Divisional application.

For more information on oath or declaration, visit: oath or declaration.

Continuation, divisional, and continuation-in-part (CIP) applications are all types of continuing applications, but they have key differences:

  • Continuation application: Discloses and claims only subject matter disclosed in the prior application. No new matter is added.
  • Divisional application: Results from a restriction requirement in the parent application. Claims an independent and distinct invention carved out of the parent application. No new matter is added.
  • Continuation-in-part (CIP) application: Repeats a substantial portion of the prior application and adds matter not disclosed in the prior application (i.e., new matter is added).

As stated in the MPEP regarding continuation-in-part applications:

A continuation-in-part is an application filed during the lifetime of a prior-filed nonprovisional application, international application designating the United States, or international design application designating the United States repeating some substantial portion or all of the prior-filed application and adding matter not disclosed in the prior-filed application.

All three types of applications must claim the benefit of the prior application under 35 U.S.C. 120, 121, 365(c), or 386(c) and comply with other requirements set forth in 37 CFR 1.78.

For more information on continuation application, visit: continuation application.

For more information on continuation-in-part application, visit: continuation-in-part application.

For more information on Divisional application, visit: Divisional application.

For more information on new matter, visit: new matter.

For more information on patent application types, visit: patent application types.

For applications filed on or after September 16, 2012, an applicant can file a continuation or divisional application using a copy of an oath or declaration from a prior application if:

  • The oath or declaration complies with 35 U.S.C. 115 as revised effective September 16, 2012
  • It was executed by or with respect to the inventor in the earlier-filed application
  • A copy of the oath, declaration, or substitute statement showing the signature or an indication it was executed is submitted in the continuing application

As stated in the MPEP: For applications filed on or after September 16, 2012, the filing of a continuing application by all or by fewer than all of the inventors named in a prior application without a newly executed oath or declaration is permitted provided that an oath or declaration in compliance with 37 CFR 1.63, or a substitute statement under 37 CFR 1.64, was executed by or with respect to such inventor and was filed in the earlier-filed application, and a copy of such oath, declaration, or substitute statement showing the signature or an indication thereon that it was executed, is submitted in the continuing application.

It’s important to note that the oath or declaration must meet the current requirements, even if the prior application was filed before September 16, 2012.

For more information on continuation application, visit: continuation application.

For more information on declaration, visit: declaration.

For more information on Divisional application, visit: Divisional application.

For more information on oath, visit: oath.

The treatment of continuation or divisional applications filed under former 37 CFR 1.62 depends on the filing date of the original application:

  • For applications filed on or after June 8, 1995: Requests filed on or after December 1, 1997, are treated as requests for continued examination (RCE) under 37 CFR 1.114.
  • For applications filed before June 8, 1995: Requests filed on or after December 1, 1997, are treated as improper applications.

As stated in the MPEP: A request for a continuation or divisional application filed under former 37 CFR 1.62 on or after December 1, 1997, in an application that was filed on or after June 8, 1995, will be treated as a request for continued examination (RCE) under 37 CFR 1.114, see MPEP 706.07(h), paragraph IV.

To learn more:

The treatment of continuation or divisional applications filed under former 37 CFR 1.62 depends on the filing date of the original application:

  • For applications filed on or after June 8, 1995: “A request for a continuation or divisional application filed under former 37 CFR 1.62 on or after December 1, 1997, in an application that was filed on or after June 8, 1995, will be treated as a request for continued examination (RCE) under 37 CFR 1.114.” This means these applications are now processed under the RCE procedure.
  • For applications filed before June 8, 1995: Requests for continuation or divisional applications filed on or after December 1, 1997, will be treated as improper applications.

For more details on RCE procedures, refer to MPEP 706.07(h), paragraph IV.

For more information on continuation application, visit: continuation application.

For more information on Divisional application, visit: Divisional application.

For more information on RCE, visit: RCE.

For applications filed on or after June 8, 1995, continuation or divisional applications filed under former 37 CFR 1.62 on or after December 1, 1997, are treated differently. The MPEP states: “A request for a continuation or divisional application filed under former 37 CFR 1.62 on or after December 1, 1997, in an application that was filed on or after June 8, 1995, will be treated as a request for continued examination (RCE) under 37 CFR 1.114.” This means such applications are now processed under the RCE procedure rather than the former File Wrapper Continuing Procedure.

For more information on continuation application, visit: continuation application.

For more information on Divisional application, visit: Divisional application.

For more information on RCE, visit: RCE.

According to the MPEP, A continuation or divisional application filed under 37 CFR 1.60 on or after December 1, 1997, will automatically be treated as an application filed under 37 CFR 1.53(b). This means that any continuation or divisional application filed after this date is processed under the new regulations, specifically 37 CFR 1.53(b), regardless of whether the applicant intended to file under the old 37 CFR 1.60 procedure.

To learn more:

To learn more:

Can the specification be omitted when filing a divisional or continuation application?

No, the specification cannot be omitted when filing a divisional or continuation application. However, the process is simplified. According to MPEP 201.06(c):

“A new specification (including the claims) may be submitted in a divisional application filed under 37 CFR 1.53(b). In such a case, the applicant must file a copy of the prior application as filed or a new specification.”

This means that while you must include a specification, you have two options:

  • Submit a copy of the prior application’s specification
  • Submit a new specification tailored to the divisional or continuation application

If you choose to submit a copy of the prior application, it’s important to ensure that it properly supports the claims in the new application. Any new matter introduced in a continuation application may result in a rejection under 35 U.S.C. 112(a).

Remember, the claims are considered part of the specification, so you must include claims with your application, whether you’re using the prior specification or submitting a new one.

For more information on continuation application, visit: continuation application.

For more information on Divisional application, visit: Divisional application.

For more information on patent application requirements, visit: patent application requirements.

For more information on USPTO, visit: USPTO.

No, new matter cannot be added to a divisional application. The divisional application must only contain subject matter disclosed in the parent application. The MPEP clearly states:

Thus the disclosure presented in a divisional application must not include any subject matter which would constitute new matter if submitted as an amendment to the parent application.

Any attempt to add new matter to a divisional application will be rejected under 35 U.S.C. 112(a) as lacking written description support in the parent application.

To learn more:

Can I file a priority claim in a continuing application?

Yes, you can file a priority claim in a continuing application, but there are specific requirements and deadlines to consider. According to MPEP 214.01, “In the case of a continuation or divisional application filed under 35 U.S.C. 111(a), the claim for priority must be made during the pendency of the application and within the later of four months from the actual filing date of the application or sixteen months from the filing date of the prior foreign application.” This means that even in a continuing application, you must adhere to the time limits for filing the priority claim. It’s important to note that the priority claim must be made in each continuing application; a priority claim in a parent application does not automatically carry over to child applications.

Yes, you can file a priority claim in a continuation or divisional application, but there are specific rules you must follow. According to MPEP 214.01:

‘In the case of a continuation or divisional application filed under 35 U.S.C. 111(a), the claim for priority must be made during the pendency of the application and within the four-month period set forth in 37 CFR 1.55(d)(1).’

This means:

  • The priority claim must be made while the continuation or divisional application is still pending (not abandoned or issued).
  • You have a four-month window from the actual filing date of the continuation or divisional application to make the priority claim.
  • The priority claim should refer back to the foreign application through the parent application(s).

It’s important to note that the continuation or divisional application must have copendency with the parent application to maintain the priority chain. Always consult the latest version of the MPEP and consider seeking advice from a patent attorney to ensure compliance with current USPTO rules.

To learn more:

Can I file a divisional application without a new oath or declaration?

Yes, you can file a divisional application without a new oath or declaration under certain conditions. According to MPEP 201.06(c):

“Under 37 CFR 1.63(d), a newly executed oath or declaration is not required in a divisional application filed under 37 CFR 1.53(b) if: (1) the prior nonprovisional application contained an oath or declaration as prescribed by 37 CFR 1.63 that was executed by or with respect to the inventor and was signed in compliance with 37 CFR 1.64; and (2) the divisional application was filed by all or by fewer than all of the inventors named in the prior application.”

This provision simplifies the filing process for divisional applications, allowing inventors to use the oath or declaration from the original application, provided the specified conditions are met.

For more information on declaration, visit: declaration.

For more information on Divisional application, visit: Divisional application.

For more information on oath, visit: oath.

No, you cannot amend the specification of a Continued Prosecution Application (CPA) to refer to it as a continuation or divisional application. According to MPEP ยง 201.06(d):

“The amendment filed [date] requesting that the specification be amended to refer to the present Continued Prosecution Application (CPA) as a [continuation/divisional] application of Application No. [X] has not been entered. As set forth in 37 CFR 1.53(d)(7), a request for a CPA is the specific reference required by 35 U.S.C. 120 to every application assigned the application number identified in such request. Thus, there is no need to amend the first sentence(s) of the specification to refer back to the prior application and any such amendment shall be denied entry.”

This means that the CPA request itself serves as the required reference to the prior application, and no additional amendment to the specification is necessary or allowed.

For more information on continuation application, visit: continuation application.

For more information on CPA, visit: CPA.

For more information on Divisional application, visit: Divisional application.

Can an inventor’s oath or declaration from a parent application be used in a divisional application?

Yes, an inventor’s oath or declaration from a parent application can often be used in a divisional application. According to MPEP 201.06(c):

‘In accordance with 37 CFR 1.63(d), a newly executed oath or declaration is not required in a divisional application filed under 37 CFR 1.53(b) if: (1) the divisional application claims are directed to subject matter disclosed in the prior-filed application; (2) a copy of the inventor’s oath or declaration from the prior-filed application is provided for the divisional application; and (3) the prior-filed application contains a statement by the applicant that any required oath or declaration, including any substitute statement under 37 CFR 1.64 or assignment-statement under pre-AIA 37 CFR 1.63(e), has been filed.’

This provision simplifies the filing process for divisional applications by allowing the reuse of the original oath or declaration, provided certain conditions are met. However, it’s important to ensure that all inventors named in the divisional application are covered by the oath or declaration from the parent application.

For more information on declaration, visit: declaration.

For more information on Divisional application, visit: Divisional application.

For more information on inventor’s oath, visit: inventor’s oath.

For more information on parent application, visit: parent application.

Yes, according to MPEP ยถ 2.05, an application can potentially be classified as both a continuation and a division. The MPEP states:

“Accordingly, this application may constitute a continuation or division.” (MPEP ยถ 2.05)

This dual classification possibility arises because:

  • A continuation application discloses and claims only subject matter from the prior application
  • A divisional application is filed as a result of a restriction requirement in the prior application
  • In some cases, an application may meet both criteria

The final determination of whether the application is a continuation, division, or both will depend on the specific circumstances of the application and its relationship to the prior application. The examiner will make this determination during the examination process.

For more information on continuation application, visit: continuation application.

For more information on Divisional application, visit: Divisional application.

No, a provisional application cannot be called a ‘divisional’ application. The MPEP ยถ 2.01 explicitly states:

An application claiming the benefit of a provisional application under 35 U.S.C. 119(e) should not be called a ‘divisional’ of the prior application.

This means that while an application can claim the benefit of a provisional application’s filing date under 35 U.S.C. 119(e), it is not considered a divisional application. Divisional applications are specifically related to non-provisional applications and involve claiming independent and distinct inventions from a parent application.

For more information on 35 USC 119(e), visit: 35 USC 119(e).

For more information on Divisional application, visit: Divisional application.

For more information on provisional application, visit: provisional application.

Can a priority claim be made in a continuation or divisional application?

Yes, a priority claim can be made in a continuation or divisional application. However, there are specific requirements and time limits to consider. According to MPEP 214.01:

“In the case of a continuation or divisional application, the claim for priority must be made during the pendency of the application.”

This means:

  • The priority claim must be filed within the later of four months from the actual filing date of the continuation or divisional application or sixteen months from the filing date of the prior foreign application.
  • If the claim is filed outside this period, it will be considered untimely and require a petition under 37 CFR 1.55(e).
  • The priority claim should reference the parent application if it properly claimed the benefit of the foreign application.

It’s important to note that the continuation or divisional application must be filed within 12 months of the foreign application’s filing date (or 6 months for design applications) to be eligible for priority.

Yes, a divisional or continuation application can claim benefit from multiple prior applications. The MPEP 201.06(c) states:

‘A continuation or divisional application may be filed under 35 U.S.C. 111(a) using the procedures set forth in 37 CFR 1.53(b), by providing: (A) a copy of the prior application as filed; and (B) a new specification, claim or claims and drawings, where applicable.’

This provision allows for claiming benefit from multiple prior applications by following the correct procedures. It’s crucial to ensure that the chain of priority is properly maintained and that all necessary documents and declarations are filed.

To learn more:

To learn more:

No, a divisional application cannot include new matter that was not disclosed in the parent application. The MPEP 201.06 clearly states:

‘A divisional application is a later application for an independent or distinct invention, carved out of a prior application and disclosing and claiming only subject matter disclosed in the prior application.’

This means:

  • The divisional application must be limited to the subject matter disclosed in the parent application
  • No new matter can be added to the divisional application
  • The claims in the divisional must be supported by the disclosure of the parent application

If an applicant wishes to add new matter, they should consider filing a continuation-in-part (CIP) application instead of a divisional. For more information on new matter, refer to MPEP 2163.

To learn more:

Yes, a divisional application can claim the benefit of the filing date of the prior application. The MPEP ยถ 2.01 states:

Should applicant desire to claim the benefit of the filing date of the prior application, attention is directed to 35 U.S.C. 120, 37 CFR 1.78, and MPEP ยง 211 et seq.

To claim this benefit, the applicant must meet the requirements set forth in 35 U.S.C. 120 and 37 CFR 1.78, which include making a proper benefit claim and ensuring the divisional application is filed before the patenting or abandonment of the prior application.

For more information on Divisional application, visit: Divisional application.

For more information on patent law, visit: patent law.

Yes, a divisional application can claim priority to its parent application. This is one of the key benefits of filing a divisional application. According to MPEP 201.06:

‘A divisional application is entitled to the benefit of the filing date of the prior application.’

This means that the divisional application:

  • Retains the priority date of the parent application for the subject matter disclosed in the parent
  • Can claim the benefit under 35 U.S.C. 120 or 365(c)
  • Must include a specific reference to the parent application in its application data sheet (ADS)

It’s important to note that the divisional application must be filed while the parent application is still pending to claim this benefit.

For more information on 35 U.S.C. 120, visit: 35 U.S.C. 120.

For more information on Divisional application, visit: Divisional application.

For more information on patent law, visit: patent law.

Yes, a divisional application can claim priority to a provisional application under certain conditions. According to MPEP 201.06:

‘A divisional application may claim the benefit of a provisional application under 35 U.S.C. 119(e) if the divisional application is filed within twelve months of the provisional application filing date (or an appropriate later date in accordance with 37 CFR 1.78).’

To claim this benefit, the divisional application must:

  • Be filed within 12 months of the provisional application’s filing date (or later if permitted under 37 CFR 1.78)
  • Include a proper reference to the provisional application
  • Have at least one common inventor with the provisional application

To learn more:

To learn more:

Yes, a divisional application can claim the benefit of its parent application’s filing date. According to MPEP 201.06(c):

“A divisional application is entitled to the benefit of the filing date of the prior-filed application if the prior-filed application discloses the invention claimed in the divisional application in the manner provided by the first paragraph of 35 U.S.C. 112.”

This means that as long as the invention claimed in the divisional application was adequately described in the parent application, it can claim the parent’s filing date. This is crucial for establishing priority and avoiding potential prior art issues.

For more information on Divisional application, visit: Divisional application.

Can a divisional application claim benefit from a provisional application?

Yes, a divisional application can claim benefit from a provisional application through its parent application. The MPEP 201.06 states: “A later-filed application may be filed as a continuation, divisional, or continuation-in-part of a prior nonprovisional application or international application designating the United States. Unless the filing date of the earlier nonprovisional application is to be claimed for prior art purposes, there is no need for the later-filed application to be copending with it.” This means that if the parent application claimed benefit from a provisional application, the divisional application can also claim that benefit, extending its effective filing date back to the provisional application’s filing date.

To learn more:

Can a divisional application be filed without a restriction requirement?

While divisional applications are typically filed in response to a restriction requirement, they can be filed voluntarily without one. The MPEP 201.06 states:

“A divisional application is often filed as a result of a restriction requirement made by the examiner.”

However, the use of “often” implies that this is not always the case. Applicants may choose to file a divisional application voluntarily if they:

  • Recognize distinct inventions in their application
  • Want to pursue different claim scopes separately
  • Need to address potential unity of invention issues proactively

It’s important to note that voluntarily filing a divisional application without a restriction requirement may affect the application of the safe harbor provision under 35 U.S.C. 121, which protects against double patenting rejections in certain cases.

For more information on Divisional application, visit: Divisional application.

Can a divisional application be filed after the parent application is abandoned?

Yes, a divisional application can be filed after the parent application is abandoned, as long as certain conditions are met. The MPEP 201.06 states:

“A divisional application may be filed after abandonment of the parent application, provided the parent application is not relied upon for benefit claims under 35 U.S.C. 120, 121, 365(c), or 386(c) in an application whose effective filing date is on or after March 16, 2013.”

This means that you can file a divisional application even if the parent application has been abandoned. However, it’s crucial to note that if you want to claim the benefit of the parent application’s filing date, and the effective filing date of your divisional application is on or after March 16, 2013, the parent application must not have been abandoned at the time of filing the divisional.

It’s important to consult with a patent attorney to ensure compliance with all requirements when filing a divisional application, especially in cases involving abandoned parent applications.

For more information on Divisional application, visit: Divisional application.

For more information on effective filing date, visit: effective filing date.

Yes, certain applications are still processed under former 37 CFR 1.62. Specifically, all continuation, divisional, and continuation-in-part (CIP) applications filed under former 37 CFR 1.62 prior to December 1, 1997, continue to be processed and examined under the procedures set forth in that regulation.

The MPEP clarifies this point: All continuation, divisional and CIP applications filed under former 37 CFR 1.62 prior to December 1, 1997, will continue to be processed and examined under the procedures set forth in former 37 CFR 1.62.

For more detailed information on the practice and procedure under former 37 CFR 1.62, the MPEP directs readers to consult MPEP ยง 201.06(b) in the MPEP 8th Edition, Rev. 1 (February 2003), which is available on the USPTO website.

To learn more:

To learn more:

MPEP 201 - Types of Applications (60)

Patent examiners should use the MPEP ยถ 2.01 form paragraph under specific circumstances. The MPEP provides the following guidance:

Examiner Note: […] 2. This form paragraph should only be used if it appears that the application may be a divisional, but a benefit claim has not been properly established.

This means examiners should use this paragraph when:

  • The application appears to contain subject matter that is independent and distinct from a prior application
  • The application seems to qualify as a divisional
  • The applicant has not yet properly claimed the benefit of the prior application’s filing date

The form paragraph serves as a prompt for the applicant to properly establish the benefit claim if they intend the application to be a divisional.

For more information on Divisional application, visit: Divisional application.

Continued Prosecution Applications (CPAs) are specifically for design patent applications. The MPEP ยถ 2.34 refers to CPAs in the context of design applications.

The MPEP ยถ 2.34 states: “In bracket 2, insert either –continuation– or –divisional–.”

This indicates that a CPA can be filed as either a continuation or a divisional of a prior design application. It’s important to note that CPAs are no longer available for utility or plant patent applications.

For more information on continuation application, visit: continuation application.

For more information on design patents, visit: design patents.

For more information on Divisional application, visit: Divisional application.

To claim the benefit of a prior application’s filing date for a divisional application, applicants must follow specific procedures. The MPEP ยถ 2.01 provides guidance:

Should applicant desire to claim the benefit of the filing date of the prior application, attention is directed to 35 U.S.C. 120, 37 CFR 1.78, and MPEP ยง 211 et seq.

This means applicants should:

  • Review 35 U.S.C. 120 for statutory requirements
  • Consult MPEP ยง 211 for detailed guidance on claiming benefit
  • Follow the procedures outlined in 37 CFR 1.78 for claiming benefit of an earlier-filed application

It’s crucial to properly establish the benefit claim to secure the earlier filing date for the divisional application.

For more information on Divisional application, visit: Divisional application.

For more information on USPTO procedures, visit: USPTO procedures.

When a patent examiner encounters an application that appears to be a divisional but lacks a proper benefit claim, they should follow the guidance provided in MPEP ยถ 2.01. The MPEP states:

“This form paragraph should only be used if it appears that the application may be a divisional, but a benefit claim has not been properly established.”

In such cases, the examiner should:

  1. Recognize that the application may potentially be a divisional based on its content and relationship to a prior application.
  2. Note that the applicant has not properly established a benefit claim to the prior application.
  3. Use the appropriate form paragraph (as mentioned in MPEP ยถ 2.01) to inform the applicant about the potential divisional status and the need to establish a proper benefit claim.
  4. Direct the applicant’s attention to the relevant laws and regulations for claiming benefit, specifically: 35 U.S.C. 120, 37 CFR 1.78, and MPEP ยง 211 et seq.

By following these steps, the examiner ensures that the applicant is aware of the potential divisional status and has the opportunity to properly establish the benefit claim if desired.

For more information on Divisional application, visit: Divisional application.

What is the time limit for filing a divisional application?

There is no specific time limit for filing a divisional application, but there are important considerations that effectively create a practical time frame:

  1. Parent Application Status: The divisional application must be filed while the parent application is still pending. Once the parent application is either abandoned or issued as a patent, it’s no longer possible to file a divisional from it.
  2. Priority Claim: To claim the benefit of the parent application’s filing date, the divisional must be filed within the time periods specified in 35 U.S.C. 120, 121, 365(c), or 386(c).
  3. Patent Term: The 20-year patent term is calculated from the earliest non-provisional U.S. filing date. Filing a divisional later in the parent’s pendency will result in a shorter potential patent term for the divisional.

The MPEP 201.06 states:

A divisional application is often filed as a result of a restriction requirement made by the examiner.

While this doesn’t specify a time limit, it implies that divisional applications are typically filed in response to office actions during the examination process. Applicants should consider filing a divisional promptly after receiving a restriction requirement to ensure they don’t miss any deadlines and to maximize the potential patent term.

For more information on Divisional application, visit: Divisional application.

For more information on filing deadline, visit: filing deadline.

What is the significance of the term ‘carved out’ in relation to divisional applications?

The term ‘carved out’ is significant in understanding the nature of divisional applications. According to MPEP 201.06: ‘A later application for an independent or distinct invention, carved out of a pending application and disclosing and claiming only subject matter disclosed in the earlier or parent application, is known as a divisional application or ‘division.”

The significance of ‘carved out’ includes:

  • It implies that the divisional application is derived from a portion of the parent application.
  • The divisional application focuses on a specific invention or set of claims that are distinct from those in the parent application.
  • The subject matter in the divisional is entirely based on the disclosure in the parent application.
  • No new matter can be added to the divisional application beyond what was disclosed in the parent.

This term emphasizes that while the divisional application is separate, it is intrinsically linked to the parent application’s original disclosure, ensuring that the applicant doesn’t gain an unfair advantage by introducing new subject matter.

For more information on Divisional application, visit: Divisional application.

For more information on invention disclosure, visit: invention disclosure.

For more information on patent filing, visit: patent filing.

What is the significance of the filing date in a divisional application?

The filing date of a divisional application is crucial for several reasons:

  • Benefit of earlier filing date: A divisional application is entitled to the benefit of the filing date of the prior-filed application, as stated in MPEP 201.06(c): ‘A divisional application is entitled to the benefit of the filing date of the prior-filed application if filed before the patenting or abandonment of or termination of proceedings on the prior-filed application.’
  • Prior art considerations: The earlier filing date can be used to overcome prior art references that might otherwise be applicable.
  • Patent term calculations: The filing date may affect the calculation of the patent term.
  • Priority claims: It establishes the basis for any priority claims to foreign applications.

It’s important to note that the divisional application must be filed while the parent application is still pending to claim the benefit of the earlier filing date.

For more information on Divisional application, visit: Divisional application.

For more information on patent term, visit: patent term.

For more information on prior art, visit: prior art.

What is the significance of the ‘original disclosure’ in a divisional application?

The ‘original disclosure’ is crucial in a divisional application because it defines the scope of what can be included in the divisional. According to MPEP 201.06:

‘The disclosure of a divisional application must be the same as the disclosure of the prior-filed application, i.e., the original disclosure of the parent or original nonprovisional application or provisional application.’

This means that no new matter can be added to a divisional application. The divisional must rely entirely on the content disclosed in the original parent application. This ensures that the divisional application doesn’t extend beyond the scope of what was initially invented and disclosed.

For more information on Divisional application, visit: Divisional application.

For more information on new matter, visit: new matter.

Claiming benefit to a prior application is crucial for divisional applications as it allows the divisional to retain the earlier filing date of the parent application. This can be significant for:

  • Establishing priority over potential prior art
  • Extending the potential patent term
  • Maintaining continuity in the patent family

The MPEP ยถ 2.01 states:

“Should applicant desire to claim the benefit of the filing date of the prior application, attention is directed to 35 U.S.C. 120, 37 CFR 1.78, and MPEP ยง 211 et seq.”

To properly claim this benefit, applicants must comply with the requirements set forth in 35 U.S.C. 120, 37 CFR 1.78, and MPEP ยง 211, which include making a proper benefit claim and ensuring continuity of disclosure.

For more information on Divisional application, visit: Divisional application.

For more information on patent law, visit: patent law.

What is the relationship between a divisional application and a continuation application?

Divisional applications and continuation applications are both types of continuing applications, but they serve different purposes. The MPEP 201.06 provides insight into the nature of divisional applications:

“A later application for an independent or distinct invention, carved out of a pending application and disclosing and claiming only subject matter disclosed in the earlier or parent application, is known as a divisional application or ‘division.'”

In contrast, a continuation application typically contains the same disclosure as the parent application but with different claims. The key differences are:

  • Divisional Application: Filed to pursue distinct inventions that were subject to a restriction requirement in the parent application.
  • Continuation Application: Filed to pursue additional aspects of the same invention disclosed in the parent application.

Both types of applications claim the benefit of the parent application’s filing date, but they differ in their purpose and the scope of claims they can contain. It’s important to choose the appropriate type of continuing application based on the specific circumstances and inventive concepts you wish to protect.

For more information on continuation application, visit: continuation application.

For more information on Divisional application, visit: Divisional application.

What is the purpose of a divisional application under 37 CFR 1.53(b)?

A divisional application under 37 CFR 1.53(b) serves to pursue claims to a distinct invention that was disclosed but not claimed in a parent application. According to the MPEP:

“A divisional application is often filed as a result of a restriction requirement made by the examiner.”

This allows inventors to protect multiple inventions that may have been included in a single original application but were deemed to be separate and distinct by the USPTO. Divisional applications maintain the benefit of the filing date of the parent application for the common subject matter.

For more information on Divisional application, visit: Divisional application.

For more information on patent filing, visit: patent filing.

What is the purpose of a divisional application in patent law?

A divisional application is a type of patent application that allows an inventor to pursue protection for different aspects of an invention that were disclosed but not claimed in a parent application. According to MPEP 201.06:

A later application for an independent or distinct invention, carved out of a pending application and disclosing and claiming only subject matter disclosed in the earlier or parent application, is known as a divisional application or ‘division.’

The main purposes of a divisional application are:

  • To protect multiple inventions disclosed in a single parent application
  • To respond to a restriction requirement issued by the USPTO
  • To pursue broader or different claim scope than what was allowed in the parent application

Divisional applications are filed under 37 CFR 1.53(b) and can benefit from the filing date of the parent application for the common subject matter.

For more information on Divisional application, visit: Divisional application.

For more information on patent law, visit: patent law.

What is the filing date requirement for a divisional application?

A divisional application must be filed before the patenting or abandonment of or termination of proceedings on the original application or on an earlier-filed divisional application directed to the same elected invention. The MPEP states:

A divisional application is often filed as a result of a restriction requirement made by the examiner. The divisional application may be filed under 37 CFR 1.53(b) (or 1.53(d) if the application is a design application) before the patenting or abandonment of or termination of proceedings on the original application or on an earlier-filed divisional application directed to the same elected invention.”

This means that the divisional application must be filed while the parent application is still pending, ensuring continuity in the examination process.

For more information on Divisional application, visit: Divisional application.

For more information on patent procedure, visit: patent procedure.

Incorporating by reference a prior application in a continuation or divisional application can provide important benefits:

  1. It allows the applicant to amend the continuing application to include subject matter from the prior application without the need for a petition, provided the continuing application is entitled to a filing date.
  2. For applications filed on or after September 21, 2004, it can help recover inadvertently omitted material.

The MPEP states: An applicant may incorporate by reference the prior application by including, in the continuing application-as-filed, an explicit statement that such specifically enumerated prior application or applications are “hereby incorporated by reference.” The statement must appear in the specification.

It’s important to note that an incorporation by reference statement added after an application’s filing date is not effective because no new matter can be added to an application after its filing date (see 35 U.S.C. 132(a)).

For more information on 37 cfr 1.57, visit: 37 cfr 1.57.

For more information on continuation application, visit: continuation application.

For more information on Divisional application, visit: Divisional application.

For more information on incorporation by reference, visit: incorporation by reference.

A preliminary amendment filed with a continuation or divisional application can have significant effects:

  1. If present on the filing date, it is considered part of the original disclosure.
  2. It must comply with the requirements of 37 CFR 1.121 for amendments.
  3. The Office may require a substitute specification for extensive preliminary amendments.
  4. The application will be classified and assigned based on the claims that will be before the examiner after entry of the preliminary amendment.

The MPEP states: Any preliminary amendment that is present on the filing date of an application filed under 37 CFR 1.53(b) is part of the original disclosure.

However, applicants should be cautious about introducing new matter through a preliminary amendment. The MPEP advises: Where a copy of the oath or declaration from a prior application was filed in a continuation or divisional application, if the examiner determines that new matter is present relative to the prior application, the examiner should so notify the applicant in the next Office action (preferably the first Office action). The examiner should require applicant to delete the benefit claim or redesignate the application as a continuation-in-part.

For more information on continuation application, visit: continuation application.

For more information on Divisional application, visit: Divisional application.

For more information on new matter, visit: new matter.

What is the difference between a divisional and a continuation application?

While both divisional and continuation applications are types of continuing applications, they serve different purposes:

  • Divisional Application: Filed to pursue claims to an invention that was not elected in a previous application due to a restriction requirement.
  • Continuation Application: Filed to pursue additional claims to the same invention disclosed in the parent application.

The MPEP 201.06(c) states: “A divisional application is often filed as a result of a restriction requirement made by the examiner.” In contrast, a continuation typically doesn’t arise from a restriction requirement but rather from the applicant’s desire to pursue additional aspects of the same invention.

Both types can be filed under 37 CFR 1.53(b) and may claim the benefit of the parent application’s filing date.

For more information on continuation application, visit: continuation application.

For more information on Divisional application, visit: Divisional application.

A divisional application is defined in MPEP 201.06 as follows:

‘A later application for a distinct or independent invention, carved out of a pending application and disclosing and claiming only subject matter disclosed in the earlier or parent application, is known as a divisional application or ‘division.’

This definition highlights that a divisional application:

  • Is filed after an initial application
  • Focuses on a distinct invention
  • Only includes subject matter from the original application
  • Is ‘carved out’ of the pending parent application

For more information on Divisional application, visit: Divisional application.

For more information on patent application types, visit: patent application types.

Filing a divisional application allows an applicant to pursue protection for distinct inventions disclosed in a parent application. Key benefits include:

  • Ability to claim different aspects or embodiments of the invention
  • Potential for broader patent coverage
  • Preservation of priority date for the disclosed subject matter
  • Opportunity to pursue claims that may have been restricted in the parent application

The MPEP indicates: A divisional application is often filed as a result of a restriction requirement made by the examiner. This suggests that divisional applications provide a way to pursue protection for inventions that were not elected in the parent application due to a restriction requirement.

For more information on Divisional application, visit: Divisional application.

For more information on patent strategy, visit: patent strategy.

What is the benefit of filing a divisional application under 37 CFR 1.53(b)?

Filing a divisional application under 37 CFR 1.53(b) offers several benefits:

  • It allows you to pursue claims to an invention that was not elected in a previous application due to a restriction requirement.
  • You can file the divisional application without a new oath or declaration if you meet certain conditions.
  • The divisional application can claim the benefit of the filing date of the prior application.

As stated in the MPEP: “A divisional application is often filed as a result of a restriction requirement made by the examiner.” This means you can protect additional aspects of your invention that weren’t covered in the original application.

For more information on Divisional application, visit: Divisional application.

For more information on patent filing, visit: patent filing.

A divisional application under 37 CFR 1.53(b) is a type of continuation application that is filed to pursue a distinct invention that was disclosed but not claimed in a prior nonprovisional application. According to the MPEP:

‘A divisional application is often filed as a result of a restriction requirement made by the examiner.’

Divisional applications allow inventors to protect multiple inventions disclosed in a single parent application by filing separate applications for each invention.

For more information on continuation application, visit: continuation application.

For more information on Divisional application, visit: Divisional application.

A divisional application is a type of patent application that claims subject matter that is independent and distinct from that claimed in a prior application. It typically arises when an original application contains multiple inventions, and the Patent Office requires the applicant to restrict the claims to a single invention.

According to MPEP ยถ 2.01, a divisional application:

“appears to claim only subject matter directed to an invention that is independent and distinct from that claimed in the prior application, and names the inventor or at least one joint inventor named in the prior application.”

To claim the benefit of the filing date of the prior application, applicants must comply with 35 U.S.C. 120, 37 CFR 1.78, and MPEP ยง 211 et seq.

For more information on Divisional application, visit: Divisional application.

For more information on patent law, visit: patent law.

For more information on USPTO, visit: USPTO.

What happens to the priority date in a divisional application?

The priority date of a divisional application remains the same as its parent application, provided that the proper procedures are followed. According to MPEP 201.06:

‘A later-filed application which is filed before the patenting or abandonment of or termination of proceedings on an earlier-filed application and which is an application for an invention which is disclosed and claimed in the earlier-filed application is an application for a ‘divisional’ of the earlier-filed application.’

This means that:

  • The divisional application retains the priority date of the parent application for the subject matter disclosed in the parent.
  • This is crucial for establishing the effective filing date for prior art purposes under 35 U.S.C. 100(i).
  • The applicant must ensure that a proper reference to the parent application is made in the divisional to maintain this priority.

Maintaining the original priority date is one of the key advantages of filing a divisional application, as it allows the applicant to protect different aspects of their invention without losing the benefit of the earlier filing date.

For more information on Divisional application, visit: Divisional application.

For more information on effective filing date, visit: effective filing date.

What happens to the parent application when a divisional application is filed?

When a divisional application is filed, the parent application continues to be prosecuted independently. The filing of a divisional application does not automatically affect the status or prosecution of the parent application.

As stated in MPEP 201.06: “The divisional application may be filed either while the original application is still pending or after it has been patented.” This means that:

  • If the parent application is pending, it remains pending and continues through the examination process.
  • If the parent application has already been granted as a patent, it remains a valid patent.

It’s important to note that while the divisional application is independent, it must contain claims to a different invention than the parent application. The examiner will ensure that there is no double patenting between the parent and divisional applications.

Additionally, if the divisional application is filed in response to a restriction requirement in the parent application, the applicant should consider canceling the claims in the parent application that are now being pursued in the divisional to avoid potential issues with duplicate claims.

For more information on Divisional application, visit: Divisional application.

For more information on parent application, visit: parent application.

For more information on patent prosecution, visit: patent prosecution.

What happens to continuation or divisional applications filed after a CPA in a design application?

Continuation or divisional applications filed after a Continued Prosecution Application (CPA) in a design application are treated differently than those filed after a CPA in a utility or plant application. According to MPEP 201.06(d):

Any continuation or divisional application filed under 37 CFR 1.53(b) after the filing of a CPA of a design application will be assigned a new application number and will reflect the filing date of the 37 CFR 1.53(b) application.

This means that unlike utility or plant applications, where a continuation or divisional application would get the benefit of the CPA’s filing date, in design applications, these subsequent applications are treated as new applications with their own filing dates.

For more information on continuation application, visit: continuation application.

For more information on CPA, visit: CPA.

For more information on Divisional application, visit: Divisional application.

Affidavits or declarations from a prior application do not automatically become part of a continuation or divisional application filed under 37 CFR 1.53(b). If an applicant wants to rely on these documents, they must take specific actions:

  1. Make remarks of record in the new application referring to the affidavit or declaration.
  2. Include a copy of the original affidavit or declaration filed in the prior application.

The MPEP states: Affidavits or declarations, such as those submitted under 37 CFR 1.130, 1.131 and 1.132 filed during the prosecution of the prior nonprovisional application do not automatically become a part of a continuation or divisional application filed under 37 CFR 1.53(b). Where it is desired to rely on an earlier filed affidavit or declaration, the applicant should make such remarks of record in the 37 CFR 1.53(b) application and include a copy of the original affidavit or declaration filed in the prior nonprovisional application.

It’s important to note that this rule doesn’t apply to continued prosecution applications (CPAs) filed under 37 CFR 1.53(d), where affidavits and declarations do automatically become part of the new application.

For more information on continuation application, visit: continuation application.

For more information on Divisional application, visit: Divisional application.

Introducing new matter in a divisional application is not allowed and can have serious consequences. According to MPEP 201.06:

“A divisional application is a later application for an independent or distinct invention, carved out of a pending application and disclosing and claiming only subject matter disclosed in the earlier or parent application.”

This means that a divisional application must not contain any new matter beyond what was disclosed in the parent application. If new matter is introduced:

  • The new matter will not be entitled to the benefit of the parent application’s filing date.
  • Claims relying on the new matter may be rejected under 35 U.S.C. 112(a) for lack of written description or enablement.
  • The application may lose its status as a proper divisional, potentially affecting its ability to overcome certain types of double patenting rejections.

To avoid these issues:

  1. Carefully review the divisional application before filing to ensure it only contains subject matter from the parent application.
  2. If new subject matter is necessary, consider filing it as a continuation-in-part (CIP) application instead of a divisional.
  3. Consult with a patent attorney or agent to ensure compliance with MPEP guidelines.

For more information on Divisional application, visit: Divisional application.

For more information on new matter, visit: new matter.

For more information on patent filing, visit: patent filing.

If you submit an amendment to refer to a Continued Prosecution Application (CPA) as a continuation or divisional application, it will be denied entry by the patent examiner. The MPEP ยถ 2.34 states:

“Thus, there is no need to amend the first sentence(s) of the specification to refer back to the prior application and any such amendment shall be denied entry.”

This means that the patent examiner will not accept or incorporate such amendments into the application. The CPA itself serves as the necessary reference to the prior application, making additional amendments unnecessary and inappropriate.

For more information on continuation application, visit: continuation application.

For more information on CPA, visit: CPA.

For more information on Divisional application, visit: Divisional application.

For more information on patent examination, visit: patent examination.

What happens if a divisional application is filed after the parent application is patented or abandoned?

If a divisional application is filed after the parent application has been patented or abandoned, it will not be entitled to the benefit of the parent application’s filing date. The MPEP states:

A divisional application is often filed as a result of a restriction requirement made by the examiner. The divisional application may be filed under 37 CFR 1.53(b) (or 1.53(d) if the application is a design application) before the patenting or abandonment of or termination of proceedings on the original application or on an earlier-filed divisional application directed to the same elected invention.

Filing a divisional application after the parent application has been patented or abandoned means:

  • The application will not be considered a proper divisional application
  • It will not receive the benefit of the parent application’s filing date
  • It may be treated as a new, independent application
  • Prior art that was not applicable to the parent application may now be applicable
  • The application may face potential statutory bars under 35 U.S.C. 102

To avoid these issues, it’s crucial to file divisional applications before the patenting, abandonment, or termination of proceedings on the parent application.

For more information on Divisional application, visit: Divisional application.

For more information on late filing, visit: late filing.

For more information on patent procedure, visit: patent procedure.

To file a Continued Prosecution Application (CPA) in a design application, the following requirements must be met:

  • The application must be a design application.
  • The prior application must be a nonprovisional application.
  • The application must be filed before the earliest of:
    • Payment of the issue fee on the prior application, unless a petition under ยง 1.313(c) is granted in the prior application;
    • Abandonment of the prior application; or
    • Termination of proceedings on the prior application.

As stated in the MPEP 201.06(d): “A continuation or divisional application that is a nonprovisional application may be filed as a CPA under 37 CFR 1.53(d) if the prior nonprovisional application is a design application that is complete as defined by 37 CFR 1.51(b).”

For more information on continuation application, visit: continuation application.

For more information on CPA, visit: CPA.

For more information on Divisional application, visit: Divisional application.

For more information on filing requirements, visit: filing requirements.

For an application to be considered a divisional, it must meet several requirements as outlined in MPEP ยถ 2.01:

  1. It must disclose and claim only subject matter disclosed in the prior application.
  2. It must claim subject matter that is independent and distinct from that claimed in the prior application.
  3. It must name the inventor or at least one joint inventor named in the prior application.

The MPEP states:

This application, which discloses and claims only subject matter disclosed in prior Application No.[1], filed [2], appears to claim only subject matter directed to an invention that is independent and distinct from that claimed in the prior application, and names the inventor or at least one joint inventor named in the prior application.

Meeting these requirements helps ensure that the divisional application is properly derived from the prior application while claiming a distinct invention.

For more information on Divisional application, visit: Divisional application.

For more information on USPTO, visit: USPTO.

A divisional application has several key characteristics as outlined in MPEP ยถ 2.01:

  1. Independent and Distinct Subject Matter: It ‘appears to claim only subject matter directed to an invention that is independent and distinct from that claimed in the prior application.’
  2. Disclosure in Prior Application: It ‘discloses and claims only subject matter disclosed in prior Application No.[1],filed[2].’
  3. Inventor Continuity: It ‘names the inventor or at least one joint inventor named in the prior application.’
  4. Potential Benefit Claim: It may claim the benefit of the filing date of the prior application, subject to meeting certain requirements.

These characteristics ensure that divisional applications properly separate distinct inventions while maintaining a connection to the original disclosure and inventorship.

For more information on Divisional application, visit: Divisional application.

The filing requirements for a divisional application under 37 CFR 1.53(b) are similar to those for a new nonprovisional application. According to the MPEP:

‘A continuation or divisional application may be filed under 35 U.S.C. 111(a) using the procedures set forth in 37 CFR 1.53(b), by providing: (1) a specification complying with 35 U.S.C. 112, including a claim or claims; (2) drawings, where necessary; and (3) the inventorship named on filing.’

Additionally, you must include:

  • The appropriate filing fees
  • An Application Data Sheet (ADS) specifying that the application is a divisional
  • A proper benefit claim to the parent application

It’s important to note that while a new oath or declaration may not be required under 37 CFR 1.63(d), you must still comply with all other filing requirements.

For more information on Divisional application, visit: Divisional application.

For more information on drawings, visit: drawings.

For more information on filing requirements, visit: filing requirements.

For more information on inventorship, visit: inventorship.

To file a CPA for a design patent application, the following requirements must be met:

  • The prior application must be a design application that is complete under 37 CFR 1.51(b)
  • The CPA must be filed before the earliest of: payment of the issue fee, abandonment, or termination of proceedings in the prior application
  • The CPA must be filed with a request on a separate paper
  • The proper filing fee, search fee, and examination fee must be paid
  • The CPA must only disclose and claim subject matter disclosed in the prior application

As stated in the MPEP: “The filing date of a CPA is the date on which a request on a separate paper for an application under this paragraph is filed.”

For more information on continuation application, visit: continuation application.

For more information on design patents, visit: design patents.

For more information on Divisional application, visit: Divisional application.

Filing a CPA for a design patent application offers several benefits:

  • Minimal filing requirements compared to a regular continuation application
  • No new filing receipt is normally issued
  • Faster processing time, as it uses the same application number and file wrapper
  • For examination priority, CPAs are treated as “amended” applications rather than “new” applications, potentially resulting in faster examination
  • The prior application is automatically abandoned, simplifying the process

As stated in the MPEP: “A CPA has a number of advantages compared to a continuation or divisional application filed under 37 CFR 1.53(b). For example, the papers required to be filed in the U.S. Patent and Trademark Office in order to secure a filing date under 37 CFR 1.53(d) are minimal compared to 37 CFR 1.53(b).”

For more information on continuation application, visit: continuation application.

For more information on design patents, visit: design patents.

For more information on Divisional application, visit: Divisional application.

How does the oath or declaration work in divisional applications?

In divisional applications, the oath or declaration process is simplified to reduce the burden on applicants. According to MPEP 201.06(c):

37 CFR 1.63(d) provides that a newly executed oath or declaration is not required in a divisional application filed under 37 CFR 1.53(b) that contains the subject matter described in the prior nonprovisional application, provided a copy of the executed oath or declaration filed in the prior nonprovisional application is submitted.

Key points about the oath or declaration in divisional applications:

  • A new oath or declaration is not required if the divisional application contains the same subject matter as the parent application.
  • A copy of the executed oath or declaration from the parent application can be submitted.
  • The copy must show the USPTO seal or stamp indicating it was received.
  • If the inventorship changes, a new oath or declaration may be required.

This provision under 37 CFR 1.63(d) streamlines the filing process for divisional applications while ensuring proper inventorship documentation.

For more information on declaration, visit: declaration.

For more information on Divisional application, visit: Divisional application.

For more information on oath, visit: oath.

How does the oath or declaration requirement differ for divisional applications?

For divisional applications, the oath or declaration requirement can be simplified under certain conditions. According to MPEP 201.06(c):

“The divisional application may be filed under 37 CFR 1.53(b) using all or part of a copy of the oath or declaration filed in the prior nonprovisional application under 37 CFR 1.63(d), as long as the oath or declaration is still applicable to the subject matter of the divisional application.”

This means that if the original oath or declaration from the parent application still covers the subject matter in the divisional application, it can be reused. However, if new matter is introduced or the inventors change, a new oath or declaration may be required.

For more information on Divisional application, visit: Divisional application.

For more information on oath or declaration, visit: oath or declaration.

For more information on patent filing, visit: patent filing.

Micro entity status does not automatically carry over to continuation or divisional applications. A new certification is required for each new application:

  1. A new assertion of entitlement to micro entity status is required for any continuation, divisional, or continuation-in-part application.
  2. This applies even if micro entity status was properly claimed in the parent application.
  3. The new certification must be filed in the continuing application.

The MPEP states: The refiling of an application under 37 CFR 1.53 as a continuation, divisional, or continuation-in-part application (including a continued prosecution application under 37 CFR 1.53(d) (design applications only)), requires a new certification of entitlement to micro entity status in the continuing application.

It’s important to note that this requirement for a new certification applies specifically to micro entity status. For small entity status, a new assertion is also required, but the rules are slightly different.

For more information on continuation application, visit: continuation application.

For more information on continuation-in-part application, visit: continuation-in-part application.

For more information on Divisional application, visit: Divisional application.

The filing date of a divisional application can affect the patent term in several ways:

  1. 20-year term: The 20-year patent term is calculated from the earliest U.S. filing date to which the divisional application claims priority. This means that the term of a patent issued on a divisional application will generally be shorter than that of its parent application.
  2. Patent Term Adjustment (PTA): The divisional application may be eligible for PTA based on delays in its own prosecution, but not for delays in the prosecution of its parent application.
  3. Terminal Disclaimer: If there’s a risk of obviousness-type double patenting, a terminal disclaimer may be required, potentially limiting the term of the divisional patent to that of the parent patent.

The MPEP 201.06(c) does not directly address patent term, but it’s important to consider these factors when filing a divisional application. For more details on patent term, refer to MPEP 2701.

For more information on Divisional application, visit: Divisional application.

For more information on patent term, visit: patent term.

For more information on patent term adjustment, visit: patent term adjustment.

The treatment of new matter varies depending on the type of continuing application:

  • Continuation and Divisional Applications: Must not contain any new matter. The disclosure must be the same as the prior application.
  • Continuation-in-Part (CIP) Applications: Can include new matter not disclosed in the parent application.
  • Continued Prosecution Applications (CPAs): Cannot contain new matter.

For continuation and divisional applications, the MPEP states: “The disclosure presented in the continuation must not include any subject matter which would constitute new matter if submitted as an amendment to the parent application.”

For CIPs, the MPEP notes: “A continuation-in-part is an application… repeating some substantial portion or all of the prior-filed application and adding matter not disclosed in the prior-filed application.”

If new matter is found in a continuation or divisional application, the examiner will require the applicant to delete the benefit claim or change the application to a continuation-in-part. For CPAs, any new matter introduced will be required to be canceled.

It’s important to note that claims in a CIP application that rely on new matter will only be entitled to the filing date of the CIP, not the parent application’s filing date.

For more information on continuation application, visit: continuation application.

For more information on continuation-in-part application, visit: continuation-in-part application.

For more information on CPA, visit: CPA.

For more information on Divisional application, visit: Divisional application.

For more information on new matter, visit: new matter.

Filing a continuation or divisional application under 37 CFR 1.53(b) does not automatically abandon the prior application. The status of the prior application depends on the actions taken by the applicant:

  1. The prior application remains pending unless expressly abandoned.
  2. To expressly abandon the prior application, a separate paper must be filed and signed in accordance with 37 CFR 1.138.
  3. If a notice of allowance has been issued in the prior application, it can become abandoned by nonpayment of the issue fee.

The MPEP states: Under 37 CFR 1.53(b) practice, the prior nonprovisional application is not automatically abandoned upon filing of the continuing application. If the prior nonprovisional application is to be expressly abandoned, such a paper must be signed in accordance with 37 CFR 1.138.

It’s important to note that if the issue fee has been paid in the prior application, a petition to withdraw the application from issue must be filed before it can be abandoned, even if a continuing application has been filed.

For more information on continuation application, visit: continuation application.

For more information on Divisional application, visit: Divisional application.

Patent examiners use specific criteria to determine if an application may qualify as a divisional. According to MPEP ยถ 2.01, an examiner looks for the following indicators:

  1. The application claims only subject matter disclosed in a prior application.
  2. The claimed subject matter appears to be independent and distinct from that claimed in the prior application.
  3. At least one inventor from the prior application is named in the current application.

The MPEP provides guidance to examiners:

“This form paragraph should only be used if it appears that the application may be a divisional, but a benefit claim has not been properly established.”

If these conditions are met, but a proper benefit claim hasn’t been made, the examiner may use this paragraph to notify the applicant of the possible divisional status and the need to establish a benefit claim if desired.

For more information on Divisional application, visit: Divisional application.

For more information on patent examination, visit: patent examination.

For more information on USPTO, visit: USPTO.

To claim the benefit of a prior application’s filing date for a divisional application, applicants must follow specific procedures outlined in patent law and regulations. The MPEP ยถ 2.01 states:

Should applicant desire to claim the benefit of the filing date of the prior application, attention is directed to 35 U.S.C. 120, 37 CFR 1.78, and MPEP ยง 211 et seq.

This means applicants should:

  • Ensure compliance with 35 U.S.C. 120 requirements for continuity and disclosure
  • Follow the procedures outlined in 37 CFR 1.78 for claiming benefit
  • Consult MPEP ยง 211 for detailed guidance on benefit claims

It’s crucial to properly establish the benefit claim to secure the earlier filing date for the divisional application.

For more information on Divisional application, visit: Divisional application.

How does a restriction requirement lead to a divisional application?

A restriction requirement often leads to the filing of a divisional application. This process occurs as follows:

  1. An examiner issues a restriction requirement when they believe the original application contains claims to two or more independent or distinct inventions.
  2. The applicant must then elect one invention to pursue in the original application.
  3. The non-elected inventions can be pursued in one or more divisional applications.

As stated in MPEP 201.06:

“The inventions embodied in the claims of a divisional application are generally further characterized as (1) independent inventions or (2) related inventions which are distinct.”

This means that the divisional application allows the applicant to protect inventions that were disclosed but not pursued in the original application due to the restriction requirement.

For more information on Divisional application, visit: Divisional application.

For more information on patent examination, visit: patent examination.

Divisional applications and provisional applications are distinct types of patent applications with different purposes and characteristics:

  • Divisional Application: Claims subject matter from a prior non-provisional application that is independent and distinct from the original claims.
  • Provisional Application: A temporary application that establishes a priority date but does not mature into an issued patent.

The MPEP explicitly states in MPEP ยถ 2.01:

“An application claiming the benefit of a provisional application under 35 U.S.C. 119(e) should not be called a ‘divisional’ of the prior application.”

This distinction is important because divisional applications claim the benefit under 35 U.S.C. 120, while provisional applications are claimed under 35 U.S.C. 119(e).

For more information on Divisional application, visit: Divisional application.

For more information on patent law, visit: patent law.

For more information on provisional application, visit: provisional application.

For more information on USPTO, visit: USPTO.

37 CFR 1.63(d) provides a simplified process for submitting an oath or declaration in a divisional application. The MPEP states:

’37 CFR 1.63(d) provides that a newly executed oath or declaration is not required in a continuation or divisional application filed by all or by fewer than all of the inventors named in a prior nonprovisional application containing a signed oath or declaration.’

This means that if the divisional application is filed by the same inventor(s) as the parent application, and there’s no new matter added, the oath or declaration from the parent application can be used for the divisional application. This simplifies the filing process and reduces paperwork.

For more information on continuation application, visit: continuation application.

For more information on Divisional application, visit: Divisional application.

For more information on oath or declaration, visit: oath or declaration.

Continuation, divisional, and continuation-in-part (CIP) applications are all types of continuing applications, but they have key differences:

  • Continuation application: Discloses and claims only subject matter disclosed in the prior application. No new matter is added.
  • Divisional application: Results from a restriction requirement in the parent application. Claims an independent and distinct invention carved out of the parent application. No new matter is added.
  • Continuation-in-part (CIP) application: Repeats a substantial portion of the prior application and adds matter not disclosed in the prior application (i.e., new matter is added).

As stated in the MPEP regarding continuation-in-part applications:

A continuation-in-part is an application filed during the lifetime of a prior-filed nonprovisional application, international application designating the United States, or international design application designating the United States repeating some substantial portion or all of the prior-filed application and adding matter not disclosed in the prior-filed application.

All three types of applications must claim the benefit of the prior application under 35 U.S.C. 120, 121, 365(c), or 386(c) and comply with other requirements set forth in 37 CFR 1.78.

For more information on continuation application, visit: continuation application.

For more information on continuation-in-part application, visit: continuation-in-part application.

For more information on Divisional application, visit: Divisional application.

For more information on new matter, visit: new matter.

For more information on patent application types, visit: patent application types.

For applications filed on or after September 16, 2012, an applicant can file a continuation or divisional application using a copy of an oath or declaration from a prior application if:

  • The oath or declaration complies with 35 U.S.C. 115 as revised effective September 16, 2012
  • It was executed by or with respect to the inventor in the earlier-filed application
  • A copy of the oath, declaration, or substitute statement showing the signature or an indication it was executed is submitted in the continuing application

As stated in the MPEP: For applications filed on or after September 16, 2012, the filing of a continuing application by all or by fewer than all of the inventors named in a prior application without a newly executed oath or declaration is permitted provided that an oath or declaration in compliance with 37 CFR 1.63, or a substitute statement under 37 CFR 1.64, was executed by or with respect to such inventor and was filed in the earlier-filed application, and a copy of such oath, declaration, or substitute statement showing the signature or an indication thereon that it was executed, is submitted in the continuing application.

It’s important to note that the oath or declaration must meet the current requirements, even if the prior application was filed before September 16, 2012.

For more information on continuation application, visit: continuation application.

For more information on declaration, visit: declaration.

For more information on Divisional application, visit: Divisional application.

For more information on oath, visit: oath.

The treatment of continuation or divisional applications filed under former 37 CFR 1.62 depends on the filing date of the original application:

  • For applications filed on or after June 8, 1995: “A request for a continuation or divisional application filed under former 37 CFR 1.62 on or after December 1, 1997, in an application that was filed on or after June 8, 1995, will be treated as a request for continued examination (RCE) under 37 CFR 1.114.” This means these applications are now processed under the RCE procedure.
  • For applications filed before June 8, 1995: Requests for continuation or divisional applications filed on or after December 1, 1997, will be treated as improper applications.

For more details on RCE procedures, refer to MPEP 706.07(h), paragraph IV.

For more information on continuation application, visit: continuation application.

For more information on Divisional application, visit: Divisional application.

For more information on RCE, visit: RCE.

For applications filed on or after June 8, 1995, continuation or divisional applications filed under former 37 CFR 1.62 on or after December 1, 1997, are treated differently. The MPEP states: “A request for a continuation or divisional application filed under former 37 CFR 1.62 on or after December 1, 1997, in an application that was filed on or after June 8, 1995, will be treated as a request for continued examination (RCE) under 37 CFR 1.114.” This means such applications are now processed under the RCE procedure rather than the former File Wrapper Continuing Procedure.

For more information on continuation application, visit: continuation application.

For more information on Divisional application, visit: Divisional application.

For more information on RCE, visit: RCE.

Can the specification be omitted when filing a divisional or continuation application?

No, the specification cannot be omitted when filing a divisional or continuation application. However, the process is simplified. According to MPEP 201.06(c):

“A new specification (including the claims) may be submitted in a divisional application filed under 37 CFR 1.53(b). In such a case, the applicant must file a copy of the prior application as filed or a new specification.”

This means that while you must include a specification, you have two options:

  • Submit a copy of the prior application’s specification
  • Submit a new specification tailored to the divisional or continuation application

If you choose to submit a copy of the prior application, it’s important to ensure that it properly supports the claims in the new application. Any new matter introduced in a continuation application may result in a rejection under 35 U.S.C. 112(a).

Remember, the claims are considered part of the specification, so you must include claims with your application, whether you’re using the prior specification or submitting a new one.

For more information on continuation application, visit: continuation application.

For more information on Divisional application, visit: Divisional application.

For more information on patent application requirements, visit: patent application requirements.

For more information on USPTO, visit: USPTO.

Can I file a divisional application without a new oath or declaration?

Yes, you can file a divisional application without a new oath or declaration under certain conditions. According to MPEP 201.06(c):

“Under 37 CFR 1.63(d), a newly executed oath or declaration is not required in a divisional application filed under 37 CFR 1.53(b) if: (1) the prior nonprovisional application contained an oath or declaration as prescribed by 37 CFR 1.63 that was executed by or with respect to the inventor and was signed in compliance with 37 CFR 1.64; and (2) the divisional application was filed by all or by fewer than all of the inventors named in the prior application.”

This provision simplifies the filing process for divisional applications, allowing inventors to use the oath or declaration from the original application, provided the specified conditions are met.

For more information on declaration, visit: declaration.

For more information on Divisional application, visit: Divisional application.

For more information on oath, visit: oath.

No, you cannot amend the specification of a Continued Prosecution Application (CPA) to refer to it as a continuation or divisional application. According to MPEP ยง 201.06(d):

“The amendment filed [date] requesting that the specification be amended to refer to the present Continued Prosecution Application (CPA) as a [continuation/divisional] application of Application No. [X] has not been entered. As set forth in 37 CFR 1.53(d)(7), a request for a CPA is the specific reference required by 35 U.S.C. 120 to every application assigned the application number identified in such request. Thus, there is no need to amend the first sentence(s) of the specification to refer back to the prior application and any such amendment shall be denied entry.”

This means that the CPA request itself serves as the required reference to the prior application, and no additional amendment to the specification is necessary or allowed.

For more information on continuation application, visit: continuation application.

For more information on CPA, visit: CPA.

For more information on Divisional application, visit: Divisional application.

Can an inventor’s oath or declaration from a parent application be used in a divisional application?

Yes, an inventor’s oath or declaration from a parent application can often be used in a divisional application. According to MPEP 201.06(c):

‘In accordance with 37 CFR 1.63(d), a newly executed oath or declaration is not required in a divisional application filed under 37 CFR 1.53(b) if: (1) the divisional application claims are directed to subject matter disclosed in the prior-filed application; (2) a copy of the inventor’s oath or declaration from the prior-filed application is provided for the divisional application; and (3) the prior-filed application contains a statement by the applicant that any required oath or declaration, including any substitute statement under 37 CFR 1.64 or assignment-statement under pre-AIA 37 CFR 1.63(e), has been filed.’

This provision simplifies the filing process for divisional applications by allowing the reuse of the original oath or declaration, provided certain conditions are met. However, it’s important to ensure that all inventors named in the divisional application are covered by the oath or declaration from the parent application.

For more information on declaration, visit: declaration.

For more information on Divisional application, visit: Divisional application.

For more information on inventor’s oath, visit: inventor’s oath.

For more information on parent application, visit: parent application.

Yes, according to MPEP ยถ 2.05, an application can potentially be classified as both a continuation and a division. The MPEP states:

“Accordingly, this application may constitute a continuation or division.” (MPEP ยถ 2.05)

This dual classification possibility arises because:

  • A continuation application discloses and claims only subject matter from the prior application
  • A divisional application is filed as a result of a restriction requirement in the prior application
  • In some cases, an application may meet both criteria

The final determination of whether the application is a continuation, division, or both will depend on the specific circumstances of the application and its relationship to the prior application. The examiner will make this determination during the examination process.

For more information on continuation application, visit: continuation application.

For more information on Divisional application, visit: Divisional application.

No, a provisional application cannot be called a ‘divisional’ application. The MPEP ยถ 2.01 explicitly states:

An application claiming the benefit of a provisional application under 35 U.S.C. 119(e) should not be called a ‘divisional’ of the prior application.

This means that while an application can claim the benefit of a provisional application’s filing date under 35 U.S.C. 119(e), it is not considered a divisional application. Divisional applications are specifically related to non-provisional applications and involve claiming independent and distinct inventions from a parent application.

For more information on 35 USC 119(e), visit: 35 USC 119(e).

For more information on Divisional application, visit: Divisional application.

For more information on provisional application, visit: provisional application.

Yes, a divisional application can claim the benefit of the filing date of the prior application. The MPEP ยถ 2.01 states:

Should applicant desire to claim the benefit of the filing date of the prior application, attention is directed to 35 U.S.C. 120, 37 CFR 1.78, and MPEP ยง 211 et seq.

To claim this benefit, the applicant must meet the requirements set forth in 35 U.S.C. 120 and 37 CFR 1.78, which include making a proper benefit claim and ensuring the divisional application is filed before the patenting or abandonment of the prior application.

For more information on Divisional application, visit: Divisional application.

For more information on patent law, visit: patent law.

Yes, a divisional application can claim priority to its parent application. This is one of the key benefits of filing a divisional application. According to MPEP 201.06:

‘A divisional application is entitled to the benefit of the filing date of the prior application.’

This means that the divisional application:

  • Retains the priority date of the parent application for the subject matter disclosed in the parent
  • Can claim the benefit under 35 U.S.C. 120 or 365(c)
  • Must include a specific reference to the parent application in its application data sheet (ADS)

It’s important to note that the divisional application must be filed while the parent application is still pending to claim this benefit.

For more information on 35 U.S.C. 120, visit: 35 U.S.C. 120.

For more information on Divisional application, visit: Divisional application.

For more information on patent law, visit: patent law.

Yes, a divisional application can claim the benefit of its parent application’s filing date. According to MPEP 201.06(c):

“A divisional application is entitled to the benefit of the filing date of the prior-filed application if the prior-filed application discloses the invention claimed in the divisional application in the manner provided by the first paragraph of 35 U.S.C. 112.”

This means that as long as the invention claimed in the divisional application was adequately described in the parent application, it can claim the parent’s filing date. This is crucial for establishing priority and avoiding potential prior art issues.

For more information on Divisional application, visit: Divisional application.

Can a divisional application be filed without a restriction requirement?

While divisional applications are typically filed in response to a restriction requirement, they can be filed voluntarily without one. The MPEP 201.06 states:

“A divisional application is often filed as a result of a restriction requirement made by the examiner.”

However, the use of “often” implies that this is not always the case. Applicants may choose to file a divisional application voluntarily if they:

  • Recognize distinct inventions in their application
  • Want to pursue different claim scopes separately
  • Need to address potential unity of invention issues proactively

It’s important to note that voluntarily filing a divisional application without a restriction requirement may affect the application of the safe harbor provision under 35 U.S.C. 121, which protects against double patenting rejections in certain cases.

For more information on Divisional application, visit: Divisional application.

Can a divisional application be filed after the parent application is abandoned?

Yes, a divisional application can be filed after the parent application is abandoned, as long as certain conditions are met. The MPEP 201.06 states:

“A divisional application may be filed after abandonment of the parent application, provided the parent application is not relied upon for benefit claims under 35 U.S.C. 120, 121, 365(c), or 386(c) in an application whose effective filing date is on or after March 16, 2013.”

This means that you can file a divisional application even if the parent application has been abandoned. However, it’s crucial to note that if you want to claim the benefit of the parent application’s filing date, and the effective filing date of your divisional application is on or after March 16, 2013, the parent application must not have been abandoned at the time of filing the divisional.

It’s important to consult with a patent attorney to ensure compliance with all requirements when filing a divisional application, especially in cases involving abandoned parent applications.

For more information on Divisional application, visit: Divisional application.

For more information on effective filing date, visit: effective filing date.

MPEP 211 - Claiming the Benefit of an Earlier Filing Date Under 35 U.S.C. 120 and 119(e) (4)

If a continuation or divisional application contains new matter not disclosed in the prior-filed application, the applicant will be required to either delete the benefit claim or change the relationship to a continuation-in-part application. This is because continuation and divisional applications cannot include new matter.

As stated in MPEP ยถ 2.10.01:

“Applicant is required to delete the benefit claim or change the relationship (continuation or divisional application) to continuation-in-part because this application contains the following matter not disclosed in the prior-filed application:”

This form paragraph is used by examiners when they identify new matter in an application claiming benefit under 35 U.S.C. 120, 121, 365(c), or 386(c).

For more information on continuation application, visit: continuation application.

For more information on Divisional application, visit: Divisional application.

For more information on new matter, visit: new matter.

For more information on patent examination, visit: patent examination.

The method for making a specific reference to a prior-filed application depends on when the application was filed:

  • For applications filed before September 16, 2012: The specific reference must be included in the first sentence(s) of the specification following the title or in an application data sheet (ADS) in compliance with pre-AIA 37 CFR 1.76.
  • For applications filed on or after September 16, 2012: The specific reference must be included in an ADS in compliance with 37 CFR 1.76.

Additionally, for benefit claims under 35 U.S.C. 120, 121, 365(c), or 386(c), the reference must include the relationship (i.e., continuation, divisional, or continuation-in-part) of the applications.

It’s important to note that for continued prosecution applications (CPAs) filed under 37 CFR 1.53(d) (design applications under 35 U.S.C. chapter 16 only), a specific reference in the specification or ADS is not required and may not be made. For these applications, The specific reference requirement of 35 U.S.C. 120 is met by the transmittal request for the CPA which is considered to be part of the CPA.

For more information on application data sheet, visit: application data sheet.

For more information on continuation application, visit: continuation application.

For more information on Divisional application, visit: Divisional application.

For more information on specific reference, visit: specific reference.

The presence of new matter in a continuation or divisional application can significantly affect its filing date. If new matter is found, the application may lose the benefit of the earlier filing date for the new subject matter.

According to MPEP ยถ 2.10.01:

“Applicant is required to delete the benefit claim or change the relationship (continuation or divisional application) to continuation-in-part because this application contains the following matter not disclosed in the prior-filed application:”

If the application is changed to a continuation-in-part, the new matter will have the filing date of the current application, while the previously disclosed matter retains the earlier filing date. This can have important implications for determining prior art and patent term.

For more information on continuation application, visit: continuation application.

For more information on continuation-in-part application, visit: continuation-in-part application.

For more information on Divisional application, visit: Divisional application.

For more information on new matter, visit: new matter.

Patent examiners carefully compare the content of the continuation or divisional application with the prior-filed application to identify any new matter. If they find information not disclosed in the prior application, they will issue a notice to the applicant.

The MPEP ยถ 2.10.01 instructs examiners:

“In bracket 1, provide an example of the matter not disclosed in the prior-filed application.”

This means the examiner must specifically point out the new matter found in the application, allowing the applicant to understand and address the issue.

For more information on continuation application, visit: continuation application.

For more information on Divisional application, visit: Divisional application.

For more information on new matter, visit: new matter.

For more information on patent examination, visit: patent examination.

MPEP 214-Formal Requirements of Claim for Foreign Priority (2)

Can I file a priority claim in a continuing application?

Yes, you can file a priority claim in a continuing application, but there are specific requirements and deadlines to consider. According to MPEP 214.01, “In the case of a continuation or divisional application filed under 35 U.S.C. 111(a), the claim for priority must be made during the pendency of the application and within the later of four months from the actual filing date of the application or sixteen months from the filing date of the prior foreign application.” This means that even in a continuing application, you must adhere to the time limits for filing the priority claim. It’s important to note that the priority claim must be made in each continuing application; a priority claim in a parent application does not automatically carry over to child applications.

Can a priority claim be made in a continuation or divisional application?

Yes, a priority claim can be made in a continuation or divisional application. However, there are specific requirements and time limits to consider. According to MPEP 214.01:

“In the case of a continuation or divisional application, the claim for priority must be made during the pendency of the application.”

This means:

  • The priority claim must be filed within the later of four months from the actual filing date of the continuation or divisional application or sixteen months from the filing date of the prior foreign application.
  • If the claim is filed outside this period, it will be considered untimely and require a petition under 37 CFR 1.55(e).
  • The priority claim should reference the parent application if it properly claimed the benefit of the foreign application.

It’s important to note that the continuation or divisional application must be filed within 12 months of the foreign application’s filing date (or 6 months for design applications) to be eligible for priority.

MPEP 400 - Representative of Applicant or Owner (1)

For continuation or divisional applications filed under 37 CFR 1.53(b) of an application previously accorded status under pre-AIA 37 CFR 1.47, the process is simplified. The MPEP states: In a continuation or divisional application filed under 37 CFR 1.53(b) of an application accorded status under pre-AIA 37 CFR 1.47, if a copy of a declaration from a prior application and a copy of a decision according status under pre-AIA 37 CFR 1.47 are filed as permitted by pre-AIA 37 CFR 1.63(d)(3)(i), the notice will not be repeated.

This means that if the proper documentation from the original application is provided, the USPTO won’t repeat the notification process for nonsigning inventors. This streamlines the application process for related applications.

To learn more:

To learn more:

MPEP 500 - Receipt and Handling of Mail and Papers (1)

How does the USPTO assign application numbers to continuation or divisional applications?

The USPTO assigns application numbers to continuation or divisional applications in the same manner as new applications. According to MPEP 503:

A continuation or divisional application (including a continued prosecution application) filed under 37 CFR 1.53(b) is assigned a new application number.

This means:

  • Each continuation or divisional application receives a unique number
  • The number is assigned in the same sequence as other new applications
  • The application number does not indicate its relationship to the parent application

It’s important to reference the parent application in the appropriate sections of the new application to establish the continuity.

For more information on continuation application, visit: continuation application.

For more information on Divisional application, visit: Divisional application.

For more information on USPTO procedures, visit: USPTO procedures.

MPEP 503 - Application Number and Filing Receipt (1)

How does the USPTO assign application numbers to continuation or divisional applications?

The USPTO assigns application numbers to continuation or divisional applications in the same manner as new applications. According to MPEP 503:

A continuation or divisional application (including a continued prosecution application) filed under 37 CFR 1.53(b) is assigned a new application number.

This means:

  • Each continuation or divisional application receives a unique number
  • The number is assigned in the same sequence as other new applications
  • The application number does not indicate its relationship to the parent application

It’s important to reference the parent application in the appropriate sections of the new application to establish the continuity.

For more information on continuation application, visit: continuation application.

For more information on Divisional application, visit: Divisional application.

For more information on USPTO procedures, visit: USPTO procedures.

Patent Law (109)

Patent examiners should use the MPEP ยถ 2.01 form paragraph under specific circumstances. The MPEP provides the following guidance:

Examiner Note: […] 2. This form paragraph should only be used if it appears that the application may be a divisional, but a benefit claim has not been properly established.

This means examiners should use this paragraph when:

  • The application appears to contain subject matter that is independent and distinct from a prior application
  • The application seems to qualify as a divisional
  • The applicant has not yet properly claimed the benefit of the prior application’s filing date

The form paragraph serves as a prompt for the applicant to properly establish the benefit claim if they intend the application to be a divisional.

For more information on Divisional application, visit: Divisional application.

A new oath or declaration is required in a patent application under the following circumstances:

  • When filing a continuation or divisional application
  • When filing a continuation-in-part application
  • To correct inventorship in an application
  • When the original oath or declaration was defective

According to MPEP 602.02: ‘A new oath or declaration is required in a continuation or divisional application filed under 37 CFR 1.53(b) or in a continuation-in-part application.’ Additionally, the MPEP states that ‘A new oath or declaration in compliance with 37 CFR 1.63 or 1.67 is required in a continuation or divisional application filed under 37 CFR 1.53(b), regardless of whether a copy of the oath or declaration from the prior application is submitted.’

To learn more:

A divisional application is often filed in response to a restriction requirement made by the patent examiner. As stated in the MPEP:

A divisional application is often filed as a result of a restriction requirement made by the examiner.

When an examiner determines that an application contains multiple independent and distinct inventions, they may issue a restriction requirement. The applicant can then elect one invention to pursue in the original application and file a divisional application to pursue the non-elected invention(s).

To learn more:

For continuation or divisional applications filed under 37 CFR 1.53(d) (continued prosecution design applications) with changes in inventorship, the requirements were as follows:

“If an inventor named in a prior application is not an inventor in a continuation or divisional application filed under 37 CFR 1.53(d) (continued prosecution design application), the request for filing the continuation or divisional application must be accompanied by a statement requesting the deletion of the name or names of the person or persons who are not inventors of the invention being claimed in the continuation or divisional application (see 37 CFR 1.53(d)(4)).”

This meant that for continued prosecution design applications, a statement requesting the deletion of non-inventors had to be submitted along with the filing request when there were changes in inventorship.

To learn more:

To learn more:

Continued Prosecution Applications (CPAs) are specifically for design patent applications. The MPEP ยถ 2.34 refers to CPAs in the context of design applications.

The MPEP ยถ 2.34 states: “In bracket 2, insert either –continuation– or –divisional–.”

This indicates that a CPA can be filed as either a continuation or a divisional of a prior design application. It’s important to note that CPAs are no longer available for utility or plant patent applications.

For more information on continuation application, visit: continuation application.

For more information on design patents, visit: design patents.

For more information on Divisional application, visit: Divisional application.

If an inventor’s signature was missing from a continuation or divisional application, the Office of Patent Application Processing (OPAP) would typically take the following steps:

  1. Send a “Notice to File Missing Parts” requiring the signature of the nonsigning inventor.
  2. If a copy of the decision according status under pre-AIA 37 CFR 1.47 was not included with the original filing, applicants could respond to the Notice by:
    • Submitting a copy of the decision according status under pre-AIA 37 CFR 1.47, along with a surcharge for late filing.
    • Alternatively, submitting an oath or declaration signed by the previously nonsigning inventor, along with the required surcharge.

This process is described in the MPEP:

“If OPAP mails such a Notice, a copy of the decision according status under pre-AIA 37 CFR 1.47, together with a surcharge under 37 CFR 1.16(f) for its late filing, will be an acceptable reply to the Notice. Alternatively, applicant may submit an oath or declaration signed by the previously nonsigning inventor together with the surcharge set forth in 37 CFR 1.16(f) in reply to the Notice.”

To learn more:

To claim the benefit of a prior application’s filing date for a divisional application, applicants must follow specific procedures. The MPEP ยถ 2.01 provides guidance:

Should applicant desire to claim the benefit of the filing date of the prior application, attention is directed to 35 U.S.C. 120, 37 CFR 1.78, and MPEP ยง 211 et seq.

This means applicants should:

  • Review 35 U.S.C. 120 for statutory requirements
  • Consult MPEP ยง 211 for detailed guidance on claiming benefit
  • Follow the procedures outlined in 37 CFR 1.78 for claiming benefit of an earlier-filed application

It’s crucial to properly establish the benefit claim to secure the earlier filing date for the divisional application.

For more information on Divisional application, visit: Divisional application.

For more information on USPTO procedures, visit: USPTO procedures.

When a patent examiner encounters an application that appears to be a divisional but lacks a proper benefit claim, they should follow the guidance provided in MPEP ยถ 2.01. The MPEP states:

“This form paragraph should only be used if it appears that the application may be a divisional, but a benefit claim has not been properly established.”

In such cases, the examiner should:

  1. Recognize that the application may potentially be a divisional based on its content and relationship to a prior application.
  2. Note that the applicant has not properly established a benefit claim to the prior application.
  3. Use the appropriate form paragraph (as mentioned in MPEP ยถ 2.01) to inform the applicant about the potential divisional status and the need to establish a proper benefit claim.
  4. Direct the applicant’s attention to the relevant laws and regulations for claiming benefit, specifically: 35 U.S.C. 120, 37 CFR 1.78, and MPEP ยง 211 et seq.

By following these steps, the examiner ensures that the applicant is aware of the potential divisional status and has the opportunity to properly establish the benefit claim if desired.

For more information on Divisional application, visit: Divisional application.

What is the time limit for filing a divisional application?

There is no specific time limit for filing a divisional application, but there are important considerations that effectively create a practical time frame:

  1. Parent Application Status: The divisional application must be filed while the parent application is still pending. Once the parent application is either abandoned or issued as a patent, it’s no longer possible to file a divisional from it.
  2. Priority Claim: To claim the benefit of the parent application’s filing date, the divisional must be filed within the time periods specified in 35 U.S.C. 120, 121, 365(c), or 386(c).
  3. Patent Term: The 20-year patent term is calculated from the earliest non-provisional U.S. filing date. Filing a divisional later in the parent’s pendency will result in a shorter potential patent term for the divisional.

The MPEP 201.06 states:

A divisional application is often filed as a result of a restriction requirement made by the examiner.

While this doesn’t specify a time limit, it implies that divisional applications are typically filed in response to office actions during the examination process. Applicants should consider filing a divisional promptly after receiving a restriction requirement to ensure they don’t miss any deadlines and to maximize the potential patent term.

For more information on Divisional application, visit: Divisional application.

For more information on filing deadline, visit: filing deadline.

What is the significance of the term ‘carved out’ in relation to divisional applications?

The term ‘carved out’ is significant in understanding the nature of divisional applications. According to MPEP 201.06: ‘A later application for an independent or distinct invention, carved out of a pending application and disclosing and claiming only subject matter disclosed in the earlier or parent application, is known as a divisional application or ‘division.”

The significance of ‘carved out’ includes:

  • It implies that the divisional application is derived from a portion of the parent application.
  • The divisional application focuses on a specific invention or set of claims that are distinct from those in the parent application.
  • The subject matter in the divisional is entirely based on the disclosure in the parent application.
  • No new matter can be added to the divisional application beyond what was disclosed in the parent.

This term emphasizes that while the divisional application is separate, it is intrinsically linked to the parent application’s original disclosure, ensuring that the applicant doesn’t gain an unfair advantage by introducing new subject matter.

For more information on Divisional application, visit: Divisional application.

For more information on invention disclosure, visit: invention disclosure.

For more information on patent filing, visit: patent filing.

What is the significance of the filing date in a divisional application?

The filing date of a divisional application is crucial for several reasons:

  • Benefit of earlier filing date: A divisional application is entitled to the benefit of the filing date of the prior-filed application, as stated in MPEP 201.06(c): ‘A divisional application is entitled to the benefit of the filing date of the prior-filed application if filed before the patenting or abandonment of or termination of proceedings on the prior-filed application.’
  • Prior art considerations: The earlier filing date can be used to overcome prior art references that might otherwise be applicable.
  • Patent term calculations: The filing date may affect the calculation of the patent term.
  • Priority claims: It establishes the basis for any priority claims to foreign applications.

It’s important to note that the divisional application must be filed while the parent application is still pending to claim the benefit of the earlier filing date.

For more information on Divisional application, visit: Divisional application.

For more information on patent term, visit: patent term.

For more information on prior art, visit: prior art.

What is the significance of the ‘original disclosure’ in a divisional application?

The ‘original disclosure’ is crucial in a divisional application because it defines the scope of what can be included in the divisional. According to MPEP 201.06:

‘The disclosure of a divisional application must be the same as the disclosure of the prior-filed application, i.e., the original disclosure of the parent or original nonprovisional application or provisional application.’

This means that no new matter can be added to a divisional application. The divisional must rely entirely on the content disclosed in the original parent application. This ensures that the divisional application doesn’t extend beyond the scope of what was initially invented and disclosed.

For more information on Divisional application, visit: Divisional application.

For more information on new matter, visit: new matter.

Claiming benefit to a prior application is crucial for divisional applications as it allows the divisional to retain the earlier filing date of the parent application. This can be significant for:

  • Establishing priority over potential prior art
  • Extending the potential patent term
  • Maintaining continuity in the patent family

The MPEP ยถ 2.01 states:

“Should applicant desire to claim the benefit of the filing date of the prior application, attention is directed to 35 U.S.C. 120, 37 CFR 1.78, and MPEP ยง 211 et seq.”

To properly claim this benefit, applicants must comply with the requirements set forth in 35 U.S.C. 120, 37 CFR 1.78, and MPEP ยง 211, which include making a proper benefit claim and ensuring continuity of disclosure.

For more information on Divisional application, visit: Divisional application.

For more information on patent law, visit: patent law.

What is the required oath or declaration for a divisional application?

For a divisional application, a new oath or declaration is generally required. However, there are exceptions:

  • If the application is filed on or after September 16, 2012, a copy of the oath or declaration from the prior nonprovisional application may be used, provided that it complies with 37 CFR 1.63, 1.64, or 1.67 for the divisional application.
  • If the divisional application was filed before September 16, 2012, and the prior application was filed on or after September 16, 2012, a copy of the oath or declaration from the prior application can be used in the divisional application.

As stated in MPEP 201.06: “A new oath or declaration is required in a divisional application filed under 37 CFR 1.53(b), regardless of whether a copy of the oath or declaration from the prior nonprovisional application is used in the divisional application, unless the divisional application is filed on or after September 16, 2012.”

To learn more:

What is the relationship between a divisional application and its parent application?

A divisional application is closely related to its parent application, as it stems from the same invention disclosure. According to MPEP 201.06, “A divisional application is a later application for an independent or distinct invention, carved out of a pending application and disclosing and claiming only subject matter disclosed in the earlier or parent application.” This means that:

  • The divisional application contains a subset of the subject matter from the parent application.
  • It claims an independent or distinct invention from what is claimed in the parent application.
  • It cannot introduce new matter beyond what was disclosed in the parent application.
  • It can claim the benefit of the parent application’s filing date.

The relationship allows for separate prosecution of different aspects of an invention while maintaining the priority date of the original disclosure.

To learn more:

What is the relationship between a divisional application and a restriction requirement?

A divisional application is often filed in response to a restriction requirement issued by the USPTO. The MPEP 201.06 explains:

“A later application for an independent or distinct invention, carved out of a pending application and disclosing and claiming only subject matter disclosed in the earlier or parent application, is known as a divisional application or ‘division.'”

The relationship between a divisional application and a restriction requirement is as follows:

  • When an examiner determines that a single application contains two or more independent or distinct inventions, they may issue a restriction requirement.
  • The applicant must then elect one invention to pursue in the original application.
  • The non-elected invention(s) can be pursued in one or more divisional applications.
  • Divisional applications allow applicants to protect multiple inventions disclosed in a single parent application without violating the principle of double patenting.

It’s important to note that while divisional applications are often filed in response to restriction requirements, they can also be filed voluntarily if the applicant recognizes distinct inventions in their application before receiving a restriction requirement.

To learn more:

What is the relationship between a divisional application and a continuation application?

Divisional applications and continuation applications are both types of continuing applications, but they serve different purposes. The MPEP 201.06 provides insight into the nature of divisional applications:

“A later application for an independent or distinct invention, carved out of a pending application and disclosing and claiming only subject matter disclosed in the earlier or parent application, is known as a divisional application or ‘division.'”

In contrast, a continuation application typically contains the same disclosure as the parent application but with different claims. The key differences are:

  • Divisional Application: Filed to pursue distinct inventions that were subject to a restriction requirement in the parent application.
  • Continuation Application: Filed to pursue additional aspects of the same invention disclosed in the parent application.

Both types of applications claim the benefit of the parent application’s filing date, but they differ in their purpose and the scope of claims they can contain. It’s important to choose the appropriate type of continuing application based on the specific circumstances and inventive concepts you wish to protect.

For more information on continuation application, visit: continuation application.

For more information on Divisional application, visit: Divisional application.

What is the purpose of a divisional application under 37 CFR 1.53(b)?

A divisional application under 37 CFR 1.53(b) serves to pursue claims to a distinct invention that was disclosed but not claimed in a parent application. According to the MPEP:

“A divisional application is often filed as a result of a restriction requirement made by the examiner.”

This allows inventors to protect multiple inventions that may have been included in a single original application but were deemed to be separate and distinct by the USPTO. Divisional applications maintain the benefit of the filing date of the parent application for the common subject matter.

For more information on Divisional application, visit: Divisional application.

For more information on patent filing, visit: patent filing.

What is the purpose of a divisional application in patent law?

A divisional application is a type of patent application that allows an inventor to pursue protection for different aspects of an invention that were disclosed but not claimed in a parent application. According to MPEP 201.06:

A later application for an independent or distinct invention, carved out of a pending application and disclosing and claiming only subject matter disclosed in the earlier or parent application, is known as a divisional application or ‘division.’

The main purposes of a divisional application are:

  • To protect multiple inventions disclosed in a single parent application
  • To respond to a restriction requirement issued by the USPTO
  • To pursue broader or different claim scope than what was allowed in the parent application

Divisional applications are filed under 37 CFR 1.53(b) and can benefit from the filing date of the parent application for the common subject matter.

For more information on Divisional application, visit: Divisional application.

For more information on patent law, visit: patent law.

What is the procedure for filing a continuation or divisional application without a new oath or declaration?

When filing a continuation or divisional application, a new oath or declaration is not always required. According to MPEP 602.05, the following procedure can be followed:

  1. File a copy of the oath or declaration from the prior application.
  2. Include a statement requesting the filing of the continuation or divisional application without a newly executed oath or declaration.
  3. Pay the filing fee for the continuation or divisional application.

The MPEP states: ‘The applicant may file a continuation or divisional application by filing a copy of the oath or declaration from the prior application, accompanied by a statement requesting the filing of the continuation or divisional application without a newly executed oath or declaration and payment of the filing fee.’

This procedure simplifies the filing process for certain types of continuing applications, reducing paperwork and potentially saving time for applicants.

To learn more:

What is the filing date requirement for a divisional application?

A divisional application must be filed before the patenting or abandonment of or termination of proceedings on the original application or on an earlier-filed divisional application directed to the same elected invention. The MPEP states:

A divisional application is often filed as a result of a restriction requirement made by the examiner. The divisional application may be filed under 37 CFR 1.53(b) (or 1.53(d) if the application is a design application) before the patenting or abandonment of or termination of proceedings on the original application or on an earlier-filed divisional application directed to the same elected invention.”

This means that the divisional application must be filed while the parent application is still pending, ensuring continuity in the examination process.

For more information on Divisional application, visit: Divisional application.

For more information on patent procedure, visit: patent procedure.

What is the filing date of a divisional application?

A divisional application is entitled to the filing date of the parent application. According to MPEP 201.06, “A divisional application is entitled to the benefit of the filing date of the prior-filed application if filed as a nonprovisional application that discloses and claims only subject matter disclosed in the prior-filed (parent) application.” This means that the divisional application inherits the filing date of its parent application, provided it meets the necessary requirements.

To learn more:

According to MPEP 201.06(c), the filing date for a divisional or continuation application filed under 37 CFR 1.53(b) is determined as follows:

‘The filing date of an application filed under 37 CFR 1.53(b) is the date on which a specification, with or without claims, is received in the Office.’

This means that the filing date is established when the USPTO receives the specification, regardless of whether claims are included. It’s important to note that other application components, such as drawings or fees, are not required to secure the filing date.

To learn more:

To learn more:

Incorporating by reference a prior application in a continuation or divisional application can provide important benefits:

  1. It allows the applicant to amend the continuing application to include subject matter from the prior application without the need for a petition, provided the continuing application is entitled to a filing date.
  2. For applications filed on or after September 21, 2004, it can help recover inadvertently omitted material.

The MPEP states: An applicant may incorporate by reference the prior application by including, in the continuing application-as-filed, an explicit statement that such specifically enumerated prior application or applications are “hereby incorporated by reference.” The statement must appear in the specification.

It’s important to note that an incorporation by reference statement added after an application’s filing date is not effective because no new matter can be added to an application after its filing date (see 35 U.S.C. 132(a)).

For more information on 37 cfr 1.57, visit: 37 cfr 1.57.

For more information on continuation application, visit: continuation application.

For more information on Divisional application, visit: Divisional application.

For more information on incorporation by reference, visit: incorporation by reference.

A preliminary amendment filed with a continuation or divisional application can have significant effects:

  1. If present on the filing date, it is considered part of the original disclosure.
  2. It must comply with the requirements of 37 CFR 1.121 for amendments.
  3. The Office may require a substitute specification for extensive preliminary amendments.
  4. The application will be classified and assigned based on the claims that will be before the examiner after entry of the preliminary amendment.

The MPEP states: Any preliminary amendment that is present on the filing date of an application filed under 37 CFR 1.53(b) is part of the original disclosure.

However, applicants should be cautious about introducing new matter through a preliminary amendment. The MPEP advises: Where a copy of the oath or declaration from a prior application was filed in a continuation or divisional application, if the examiner determines that new matter is present relative to the prior application, the examiner should so notify the applicant in the next Office action (preferably the first Office action). The examiner should require applicant to delete the benefit claim or redesignate the application as a continuation-in-part.

For more information on continuation application, visit: continuation application.

For more information on Divisional application, visit: Divisional application.

For more information on new matter, visit: new matter.

What is the difference between a divisional and a continuation application?

While both divisional and continuation applications are types of continuing applications, they serve different purposes:

  • Divisional Application: Filed to pursue claims to an invention that was not elected in a previous application due to a restriction requirement.
  • Continuation Application: Filed to pursue additional claims to the same invention disclosed in the parent application.

The MPEP 201.06(c) states: “A divisional application is often filed as a result of a restriction requirement made by the examiner.” In contrast, a continuation typically doesn’t arise from a restriction requirement but rather from the applicant’s desire to pursue additional aspects of the same invention.

Both types can be filed under 37 CFR 1.53(b) and may claim the benefit of the parent application’s filing date.

For more information on continuation application, visit: continuation application.

For more information on Divisional application, visit: Divisional application.

A continuation application and a divisional application are both types of continuing applications, but they serve different purposes:

  • Continuation application: Used to pursue additional claims to an invention disclosed in a prior application. It contains the same disclosure as the parent application.
  • Divisional application: Used to pursue claims to an invention that was disclosed but not claimed in a prior application. It typically results from a restriction requirement in the parent application.

As stated in MPEP 201.07: “A continuation application is an application for the invention(s) disclosed in a prior-filed copending nonprovisional application, international application designating the United States, or international design application designating the United States.” In contrast, a divisional application focuses on a distinct invention that was not claimed in the parent application.

To learn more:

A divisional application is defined in MPEP 201.06 as follows:

‘A later application for a distinct or independent invention, carved out of a pending application and disclosing and claiming only subject matter disclosed in the earlier or parent application, is known as a divisional application or ‘division.’

This definition highlights that a divisional application:

  • Is filed after an initial application
  • Focuses on a distinct invention
  • Only includes subject matter from the original application
  • Is ‘carved out’ of the pending parent application

For more information on Divisional application, visit: Divisional application.

For more information on patent application types, visit: patent application types.

Filing a divisional application allows an applicant to pursue protection for distinct inventions disclosed in a parent application. Key benefits include:

  • Ability to claim different aspects or embodiments of the invention
  • Potential for broader patent coverage
  • Preservation of priority date for the disclosed subject matter
  • Opportunity to pursue claims that may have been restricted in the parent application

The MPEP indicates: A divisional application is often filed as a result of a restriction requirement made by the examiner. This suggests that divisional applications provide a way to pursue protection for inventions that were not elected in the parent application due to a restriction requirement.

For more information on Divisional application, visit: Divisional application.

For more information on patent strategy, visit: patent strategy.

What is the benefit of filing a divisional application under 37 CFR 1.53(b)?

Filing a divisional application under 37 CFR 1.53(b) offers several benefits:

  • It allows you to pursue claims to an invention that was not elected in a previous application due to a restriction requirement.
  • You can file the divisional application without a new oath or declaration if you meet certain conditions.
  • The divisional application can claim the benefit of the filing date of the prior application.

As stated in the MPEP: “A divisional application is often filed as a result of a restriction requirement made by the examiner.” This means you can protect additional aspects of your invention that weren’t covered in the original application.

For more information on Divisional application, visit: Divisional application.

For more information on patent filing, visit: patent filing.

A divisional application is a later application for an independent or distinct invention, carved out of a nonprovisional application. It discloses and claims only subject matter disclosed in the earlier or parent application. As stated in MPEP 201.06:

A later application for an independent or distinct invention, carved out of a nonprovisional application (including a nonprovisional application resulting from an international application or international design application), an international application designating the United States, or an international design application designating the United States and disclosing and claiming only subject matter disclosed in the earlier or parent application, is known as a divisional application.

To learn more:

A divisional application under 37 CFR 1.53(b) is a type of continuation application that is filed to pursue a distinct invention that was disclosed but not claimed in a prior nonprovisional application. According to the MPEP:

‘A divisional application is often filed as a result of a restriction requirement made by the examiner.’

Divisional applications allow inventors to protect multiple inventions disclosed in a single parent application by filing separate applications for each invention.

For more information on continuation application, visit: continuation application.

For more information on Divisional application, visit: Divisional application.

A divisional application is a type of patent application that claims subject matter that is independent and distinct from that claimed in a prior application. It typically arises when an original application contains multiple inventions, and the Patent Office requires the applicant to restrict the claims to a single invention.

According to MPEP ยถ 2.01, a divisional application:

“appears to claim only subject matter directed to an invention that is independent and distinct from that claimed in the prior application, and names the inventor or at least one joint inventor named in the prior application.”

To claim the benefit of the filing date of the prior application, applicants must comply with 35 U.S.C. 120, 37 CFR 1.78, and MPEP ยง 211 et seq.

For more information on Divisional application, visit: Divisional application.

For more information on patent law, visit: patent law.

For more information on USPTO, visit: USPTO.

A continuation or divisional application is a type of continuing application filed under 37 CFR 1.53(b). According to the MPEP, An application filed under 37 CFR 1.53(b) may be an original or a reissue, a continuation, a divisional, a continuation-in-part, or a substitute. These applications are filed under 35 U.S.C. 111(a) and can be for utility, design, plant, or reissue patents.

To learn more:

To learn more:

What happens to the priority date in a divisional application?

The priority date of a divisional application remains the same as its parent application, provided that the proper procedures are followed. According to MPEP 201.06:

‘A later-filed application which is filed before the patenting or abandonment of or termination of proceedings on an earlier-filed application and which is an application for an invention which is disclosed and claimed in the earlier-filed application is an application for a ‘divisional’ of the earlier-filed application.’

This means that:

  • The divisional application retains the priority date of the parent application for the subject matter disclosed in the parent.
  • This is crucial for establishing the effective filing date for prior art purposes under 35 U.S.C. 100(i).
  • The applicant must ensure that a proper reference to the parent application is made in the divisional to maintain this priority.

Maintaining the original priority date is one of the key advantages of filing a divisional application, as it allows the applicant to protect different aspects of their invention without losing the benefit of the earlier filing date.

For more information on Divisional application, visit: Divisional application.

For more information on effective filing date, visit: effective filing date.

What happens to the parent application when a divisional application is filed?

When a divisional application is filed, the parent application continues to be prosecuted independently. The filing of a divisional application does not automatically affect the status or prosecution of the parent application.

As stated in MPEP 201.06: “The divisional application may be filed either while the original application is still pending or after it has been patented.” This means that:

  • If the parent application is pending, it remains pending and continues through the examination process.
  • If the parent application has already been granted as a patent, it remains a valid patent.

It’s important to note that while the divisional application is independent, it must contain claims to a different invention than the parent application. The examiner will ensure that there is no double patenting between the parent and divisional applications.

Additionally, if the divisional application is filed in response to a restriction requirement in the parent application, the applicant should consider canceling the claims in the parent application that are now being pursued in the divisional to avoid potential issues with duplicate claims.

For more information on Divisional application, visit: Divisional application.

For more information on parent application, visit: parent application.

For more information on patent prosecution, visit: patent prosecution.

Terminal disclaimers filed in a parent application do not automatically carry over to a divisional application. The MPEP 201.06 states:

‘A terminal disclaimer filed to obviate a nonstatutory double patenting rejection in a parent application does not carry over to a divisional application. The applicant filing the divisional application must determine whether a new terminal disclaimer needs to be filed in the divisional application.’

This means:

  • Terminal disclaimers from the parent application are not automatically applied to the divisional
  • The applicant must assess whether a new terminal disclaimer is necessary for the divisional application
  • If required, a new terminal disclaimer must be filed specifically for the divisional application

It’s important for applicants to review potential double patenting issues in the divisional application and file new terminal disclaimers if needed to overcome any rejections.

To learn more:

What happens to continuation or divisional applications filed after a CPA in a design application?

Continuation or divisional applications filed after a Continued Prosecution Application (CPA) in a design application are treated differently than those filed after a CPA in a utility or plant application. According to MPEP 201.06(d):

Any continuation or divisional application filed under 37 CFR 1.53(b) after the filing of a CPA of a design application will be assigned a new application number and will reflect the filing date of the 37 CFR 1.53(b) application.

This means that unlike utility or plant applications, where a continuation or divisional application would get the benefit of the CPA’s filing date, in design applications, these subsequent applications are treated as new applications with their own filing dates.

For more information on continuation application, visit: continuation application.

For more information on CPA, visit: CPA.

For more information on Divisional application, visit: Divisional application.

Affidavits or declarations from a prior application do not automatically become part of a continuation or divisional application filed under 37 CFR 1.53(b). If an applicant wants to rely on these documents, they must take specific actions:

  1. Make remarks of record in the new application referring to the affidavit or declaration.
  2. Include a copy of the original affidavit or declaration filed in the prior application.

The MPEP states: Affidavits or declarations, such as those submitted under 37 CFR 1.130, 1.131 and 1.132 filed during the prosecution of the prior nonprovisional application do not automatically become a part of a continuation or divisional application filed under 37 CFR 1.53(b). Where it is desired to rely on an earlier filed affidavit or declaration, the applicant should make such remarks of record in the 37 CFR 1.53(b) application and include a copy of the original affidavit or declaration filed in the prior nonprovisional application.

It’s important to note that this rule doesn’t apply to continued prosecution applications (CPAs) filed under 37 CFR 1.53(d), where affidavits and declarations do automatically become part of the new application.

For more information on continuation application, visit: continuation application.

For more information on Divisional application, visit: Divisional application.

Introducing new matter in a divisional application is not allowed and can have serious consequences. According to MPEP 201.06:

“A divisional application is a later application for an independent or distinct invention, carved out of a pending application and disclosing and claiming only subject matter disclosed in the earlier or parent application.”

This means that a divisional application must not contain any new matter beyond what was disclosed in the parent application. If new matter is introduced:

  • The new matter will not be entitled to the benefit of the parent application’s filing date.
  • Claims relying on the new matter may be rejected under 35 U.S.C. 112(a) for lack of written description or enablement.
  • The application may lose its status as a proper divisional, potentially affecting its ability to overcome certain types of double patenting rejections.

To avoid these issues:

  1. Carefully review the divisional application before filing to ensure it only contains subject matter from the parent application.
  2. If new subject matter is necessary, consider filing it as a continuation-in-part (CIP) application instead of a divisional.
  3. Consult with a patent attorney or agent to ensure compliance with MPEP guidelines.

For more information on Divisional application, visit: Divisional application.

For more information on new matter, visit: new matter.

For more information on patent filing, visit: patent filing.

If you submit an amendment to refer to a Continued Prosecution Application (CPA) as a continuation or divisional application, it will be denied entry by the patent examiner. The MPEP ยถ 2.34 states:

“Thus, there is no need to amend the first sentence(s) of the specification to refer back to the prior application and any such amendment shall be denied entry.”

This means that the patent examiner will not accept or incorporate such amendments into the application. The CPA itself serves as the necessary reference to the prior application, making additional amendments unnecessary and inappropriate.

For more information on continuation application, visit: continuation application.

For more information on CPA, visit: CPA.

For more information on Divisional application, visit: Divisional application.

For more information on patent examination, visit: patent examination.

What happens if a divisional application is filed after the parent application is patented or abandoned?

If a divisional application is filed after the parent application has been patented or abandoned, it will not be entitled to the benefit of the parent application’s filing date. The MPEP states:

A divisional application is often filed as a result of a restriction requirement made by the examiner. The divisional application may be filed under 37 CFR 1.53(b) (or 1.53(d) if the application is a design application) before the patenting or abandonment of or termination of proceedings on the original application or on an earlier-filed divisional application directed to the same elected invention.

Filing a divisional application after the parent application has been patented or abandoned means:

  • The application will not be considered a proper divisional application
  • It will not receive the benefit of the parent application’s filing date
  • It may be treated as a new, independent application
  • Prior art that was not applicable to the parent application may now be applicable
  • The application may face potential statutory bars under 35 U.S.C. 102

To avoid these issues, it’s crucial to file divisional applications before the patenting, abandonment, or termination of proceedings on the parent application.

For more information on Divisional application, visit: Divisional application.

For more information on late filing, visit: late filing.

For more information on patent procedure, visit: patent procedure.

If a continuation or divisional application contains new matter not disclosed in the prior-filed application, the applicant will be required to either delete the benefit claim or change the relationship to a continuation-in-part application. This is because continuation and divisional applications cannot include new matter.

As stated in MPEP ยถ 2.10.01:

“Applicant is required to delete the benefit claim or change the relationship (continuation or divisional application) to continuation-in-part because this application contains the following matter not disclosed in the prior-filed application:”

This form paragraph is used by examiners when they identify new matter in an application claiming benefit under 35 U.S.C. 120, 121, 365(c), or 386(c).

For more information on continuation application, visit: continuation application.

For more information on Divisional application, visit: Divisional application.

For more information on new matter, visit: new matter.

For more information on patent examination, visit: patent examination.

For continuation, divisional, or continuation-in-part applications filed under 37 CFR 1.53(b), the requirements for submitting an Information Disclosure Statement (IDS) depend on whether the information was previously considered in the parent application. The MPEP provides the following guidance:

  • If the information was considered in the parent application, it need not be resubmitted unless the applicant wants it printed on the patent.
  • If the information was not considered in the parent application, it must be resubmitted in compliance with 37 CFR 1.97 and 37 CFR 1.98.

The MPEP further states:

“Pursuant to 37 CFR 1.98(d), if the IDS submitted in the parent application complies with 37 CFR 1.98(a) to (c), copies of the patents, publications, pending U.S. applications, or other information submitted in the parent application need not be resubmitted in the continuing application.”

When resubmitting information, applicants should use a new listing that complies with the format requirements in 37 CFR 1.98(a)(1) and avoid submitting copies of PTO/SB/08 or PTO-892 forms from other applications to prevent confusion in the record.

To learn more:

The MPEP 201.06(c) outlines the requirements for filing a divisional or continuation application without an executed oath or declaration:

’37 CFR 1.63(d) provides that a newly executed oath or declaration is not required in a continuation or divisional application filed by all or by fewer than all of the inventors named in a prior nonprovisional application containing an oath or declaration as required by 37 CFR 1.63, provided that a copy of the executed oath or declaration filed in the prior application is submitted for the continuation or divisional application and the specification and drawings filed in the continuation or divisional application contain no matter that would have been new matter in the prior application.’

To file without a new oath or declaration:

  • Submit a copy of the executed oath or declaration from the prior application
  • Ensure the new application contains no new matter
  • File a copy of the specification and drawings from the prior application

This procedure allows for expedited filing while maintaining the integrity of the oath or declaration requirements.

To learn more:

To learn more:

To file a Continued Prosecution Application (CPA) in a design application, the following requirements must be met:

  • The application must be a design application.
  • The prior application must be a nonprovisional application.
  • The application must be filed before the earliest of:
    • Payment of the issue fee on the prior application, unless a petition under ยง 1.313(c) is granted in the prior application;
    • Abandonment of the prior application; or
    • Termination of proceedings on the prior application.

As stated in the MPEP 201.06(d): “A continuation or divisional application that is a nonprovisional application may be filed as a CPA under 37 CFR 1.53(d) if the prior nonprovisional application is a design application that is complete as defined by 37 CFR 1.51(b).”

For more information on continuation application, visit: continuation application.

For more information on CPA, visit: CPA.

For more information on Divisional application, visit: Divisional application.

For more information on filing requirements, visit: filing requirements.

For continuation or divisional applications filed before September 16, 2012 under 37 CFR 1.53(b) (excluding continuation-in-part applications), applicants could file using a copy of the oath or declaration from the prior nonprovisional application. This is supported by the MPEP, which states:

“A continuation or divisional application filed before September 16, 2012 under 37 CFR 1.53(b) (other than a continuation-in-part (CIP)) may be filed with a copy of the oath or declaration from the prior nonprovisional application.”

This provision simplified the filing process for certain types of continuing applications by allowing the use of previously submitted documents.

To learn more:

For an application to be considered a divisional, it must meet several requirements as outlined in MPEP ยถ 2.01:

  1. It must disclose and claim only subject matter disclosed in the prior application.
  2. It must claim subject matter that is independent and distinct from that claimed in the prior application.
  3. It must name the inventor or at least one joint inventor named in the prior application.

The MPEP states:

This application, which discloses and claims only subject matter disclosed in prior Application No.[1], filed [2], appears to claim only subject matter directed to an invention that is independent and distinct from that claimed in the prior application, and names the inventor or at least one joint inventor named in the prior application.

Meeting these requirements helps ensure that the divisional application is properly derived from the prior application while claiming a distinct invention.

For more information on Divisional application, visit: Divisional application.

For more information on USPTO, visit: USPTO.

A divisional application has several key characteristics as outlined in MPEP ยถ 2.01:

  1. Independent and Distinct Subject Matter: It ‘appears to claim only subject matter directed to an invention that is independent and distinct from that claimed in the prior application.’
  2. Disclosure in Prior Application: It ‘discloses and claims only subject matter disclosed in prior Application No.[1],filed[2].’
  3. Inventor Continuity: It ‘names the inventor or at least one joint inventor named in the prior application.’
  4. Potential Benefit Claim: It may claim the benefit of the filing date of the prior application, subject to meeting certain requirements.

These characteristics ensure that divisional applications properly separate distinct inventions while maintaining a connection to the original disclosure and inventorship.

For more information on Divisional application, visit: Divisional application.

The filing requirements for a divisional application under 37 CFR 1.53(b) are similar to those for a new nonprovisional application. According to the MPEP:

‘A continuation or divisional application may be filed under 35 U.S.C. 111(a) using the procedures set forth in 37 CFR 1.53(b), by providing: (1) a specification complying with 35 U.S.C. 112, including a claim or claims; (2) drawings, where necessary; and (3) the inventorship named on filing.’

Additionally, you must include:

  • The appropriate filing fees
  • An Application Data Sheet (ADS) specifying that the application is a divisional
  • A proper benefit claim to the parent application

It’s important to note that while a new oath or declaration may not be required under 37 CFR 1.63(d), you must still comply with all other filing requirements.

For more information on Divisional application, visit: Divisional application.

For more information on drawings, visit: drawings.

For more information on filing requirements, visit: filing requirements.

For more information on inventorship, visit: inventorship.

To file a CPA for a design patent application, the following requirements must be met:

  • The prior application must be a design application that is complete under 37 CFR 1.51(b)
  • The CPA must be filed before the earliest of: payment of the issue fee, abandonment, or termination of proceedings in the prior application
  • The CPA must be filed with a request on a separate paper
  • The proper filing fee, search fee, and examination fee must be paid
  • The CPA must only disclose and claim subject matter disclosed in the prior application

As stated in the MPEP: “The filing date of a CPA is the date on which a request on a separate paper for an application under this paragraph is filed.”

For more information on continuation application, visit: continuation application.

For more information on design patents, visit: design patents.

For more information on Divisional application, visit: Divisional application.

Filing a CPA for a design patent application offers several benefits:

  • Minimal filing requirements compared to a regular continuation application
  • No new filing receipt is normally issued
  • Faster processing time, as it uses the same application number and file wrapper
  • For examination priority, CPAs are treated as “amended” applications rather than “new” applications, potentially resulting in faster examination
  • The prior application is automatically abandoned, simplifying the process

As stated in the MPEP: “A CPA has a number of advantages compared to a continuation or divisional application filed under 37 CFR 1.53(b). For example, the papers required to be filed in the U.S. Patent and Trademark Office in order to secure a filing date under 37 CFR 1.53(d) are minimal compared to 37 CFR 1.53(b).”

For more information on continuation application, visit: continuation application.

For more information on design patents, visit: design patents.

For more information on Divisional application, visit: Divisional application.

When a petition under 37 CFR 1.48 to add an inventor was filed in a prior application, the continuation or divisional application could be handled in one of two ways:

  1. Filed with a copy of the executed declaration naming the correct inventive entity from the prior application.
  2. Filed with a newly executed declaration naming the correct inventive entity.

The MPEP states:

“A continuation or divisional application filed under 37 CFR 1.53(b) of a prior application in which a petition (or request) under 37 CFR 1.48 to add an inventor was filed should be filed with a copy of the executed declaration naming the correct inventive entity from the prior application or a newly executed declaration naming the correct inventive entity.”

Importantly, the MPEP also notes that “A copy of any decision under 37 CFR 1.48 from the prior application is not required to be filed in the continuation or divisional application.” This simplifies the filing process by not requiring additional documentation related to the inventorship change in the prior application.

To learn more:

To learn more:

Changes in inventorship for continuation or divisional applications filed under 37 CFR 1.53(b) could be handled in two ways:

  1. Filing with a copy of an oath or declaration from a prior application and a statement requesting the deletion of non-inventors.
  2. Filing with a newly executed oath or declaration naming the correct inventive entity.

The MPEP states:

“If an inventor named in a prior application is not an inventor in a continuation or divisional application filed under 37 CFR 1.53(b), the continuation or divisional application may either be filed (A) with a copy of an oath or declaration from a prior application and a statement requesting the deletion of the name or names of the person or persons who are not inventors of the invention being claimed in the continuation or divisional application (see pre-AIA 37 CFR 1.63(d)), or (B) with a newly executed oath or declaration naming the correct inventive entity.”

This provision allowed applicants to address changes in inventorship efficiently when filing continuation or divisional applications.

To learn more:

Applications with pre-AIA 37 CFR 1.47 status were given special consideration in continuation or divisional filings. The MPEP states:

“A continuation or divisional application of a prior application accorded status under pre-AIA 37 CFR 1.47 will be accorded status under pre-AIA 37 CFR 1.47 if a copy of the decision according pre-AIA 37 CFR 1.47 status in the prior application is filed in the continuation or divisional application, unless an oath or declaration signed by all of the inventors is included upon filing the continuation or divisional application.”

This means that the special status could be carried over to the new application by submitting a copy of the previous decision, unless a complete oath or declaration was provided.

To learn more:

To learn more:

The method for making a specific reference to a prior-filed application depends on when the application was filed:

  • For applications filed before September 16, 2012: The specific reference must be included in the first sentence(s) of the specification following the title or in an application data sheet (ADS) in compliance with pre-AIA 37 CFR 1.76.
  • For applications filed on or after September 16, 2012: The specific reference must be included in an ADS in compliance with 37 CFR 1.76.

Additionally, for benefit claims under 35 U.S.C. 120, 121, 365(c), or 386(c), the reference must include the relationship (i.e., continuation, divisional, or continuation-in-part) of the applications.

It’s important to note that for continued prosecution applications (CPAs) filed under 37 CFR 1.53(d) (design applications under 35 U.S.C. chapter 16 only), a specific reference in the specification or ADS is not required and may not be made. For these applications, The specific reference requirement of 35 U.S.C. 120 is met by the transmittal request for the CPA which is considered to be part of the CPA.

For more information on application data sheet, visit: application data sheet.

For more information on continuation application, visit: continuation application.

For more information on Divisional application, visit: Divisional application.

For more information on specific reference, visit: specific reference.

Special attention is required when dealing with the correspondence address in continuation or divisional applications. According to MPEP 601.03(b):

“Special care should be taken in continuation or divisional applications to ensure that any change of correspondence address in a prior application is reflected in the continuation or divisional application.”

The MPEP further explains that when a copy of the oath or declaration from the prior application is submitted for a continuation or divisional application filed under pre-AIA 37 CFR 1.53(b), and this copy designates an old correspondence address, the USPTO may not recognize the change of correspondence address made during the prosecution of the prior application.

To address this, the MPEP advises: “Applicant is required to identify the change of correspondence address in the continuation or divisional application to ensure that communications from the Office are mailed to the current correspondence address.”

To learn more:

To learn more:

How does the USPTO assign application numbers to continuation or divisional applications?

The USPTO assigns application numbers to continuation or divisional applications in the same manner as new applications. According to MPEP 503:

A continuation or divisional application (including a continued prosecution application) filed under 37 CFR 1.53(b) is assigned a new application number.

This means:

  • Each continuation or divisional application receives a unique number
  • The number is assigned in the same sequence as other new applications
  • The application number does not indicate its relationship to the parent application

It’s important to reference the parent application in the appropriate sections of the new application to establish the continuity.

For more information on continuation application, visit: continuation application.

For more information on Divisional application, visit: Divisional application.

For more information on USPTO procedures, visit: USPTO procedures.

The presence of new matter in a continuation or divisional application can significantly affect its filing date. If new matter is found, the application may lose the benefit of the earlier filing date for the new subject matter.

According to MPEP ยถ 2.10.01:

“Applicant is required to delete the benefit claim or change the relationship (continuation or divisional application) to continuation-in-part because this application contains the following matter not disclosed in the prior-filed application:”

If the application is changed to a continuation-in-part, the new matter will have the filing date of the current application, while the previously disclosed matter retains the earlier filing date. This can have important implications for determining prior art and patent term.

For more information on continuation application, visit: continuation application.

For more information on continuation-in-part application, visit: continuation-in-part application.

For more information on Divisional application, visit: Divisional application.

For more information on new matter, visit: new matter.

How does the oath or declaration work in divisional applications?

In divisional applications, the oath or declaration process is simplified to reduce the burden on applicants. According to MPEP 201.06(c):

37 CFR 1.63(d) provides that a newly executed oath or declaration is not required in a divisional application filed under 37 CFR 1.53(b) that contains the subject matter described in the prior nonprovisional application, provided a copy of the executed oath or declaration filed in the prior nonprovisional application is submitted.

Key points about the oath or declaration in divisional applications:

  • A new oath or declaration is not required if the divisional application contains the same subject matter as the parent application.
  • A copy of the executed oath or declaration from the parent application can be submitted.
  • The copy must show the USPTO seal or stamp indicating it was received.
  • If the inventorship changes, a new oath or declaration may be required.

This provision under 37 CFR 1.63(d) streamlines the filing process for divisional applications while ensuring proper inventorship documentation.

For more information on declaration, visit: declaration.

For more information on Divisional application, visit: Divisional application.

For more information on oath, visit: oath.

How does the oath or declaration requirement differ for divisional applications?

For divisional applications, the oath or declaration requirement can be simplified under certain conditions. According to MPEP 201.06(c):

“The divisional application may be filed under 37 CFR 1.53(b) using all or part of a copy of the oath or declaration filed in the prior nonprovisional application under 37 CFR 1.63(d), as long as the oath or declaration is still applicable to the subject matter of the divisional application.”

This means that if the original oath or declaration from the parent application still covers the subject matter in the divisional application, it can be reused. However, if new matter is introduced or the inventors change, a new oath or declaration may be required.

For more information on Divisional application, visit: Divisional application.

For more information on oath or declaration, visit: oath or declaration.

For more information on patent filing, visit: patent filing.

Micro entity status does not automatically carry over to continuation or divisional applications. A new certification is required for each new application:

  1. A new assertion of entitlement to micro entity status is required for any continuation, divisional, or continuation-in-part application.
  2. This applies even if micro entity status was properly claimed in the parent application.
  3. The new certification must be filed in the continuing application.

The MPEP states: The refiling of an application under 37 CFR 1.53 as a continuation, divisional, or continuation-in-part application (including a continued prosecution application under 37 CFR 1.53(d) (design applications only)), requires a new certification of entitlement to micro entity status in the continuing application.

It’s important to note that this requirement for a new certification applies specifically to micro entity status. For small entity status, a new assertion is also required, but the rules are slightly different.

For more information on continuation application, visit: continuation application.

For more information on continuation-in-part application, visit: continuation-in-part application.

For more information on Divisional application, visit: Divisional application.

The inventorship of a divisional application must include at least one inventor named in the parent application, but it doesn’t have to be identical. The MPEP states:

The inventorship in the divisional application must include at least one inventor named in the prior-filed application, and the divisional application must claim the benefit of the prior-filed application under 35 U.S.C. 120, 121, 365(c), or 386(c).

If the divisional application is filed with fewer inventors than the parent application, a statement may be filed requesting deletion of the names of the additional inventors. For applications filed on or after September 16, 2012, the statement must be filed with the divisional application and signed by a party set forth in 37 CFR 1.33(b).

To learn more:

The filing date of a divisional application can affect the patent term in several ways:

  1. 20-year term: The 20-year patent term is calculated from the earliest U.S. filing date to which the divisional application claims priority. This means that the term of a patent issued on a divisional application will generally be shorter than that of its parent application.
  2. Patent Term Adjustment (PTA): The divisional application may be eligible for PTA based on delays in its own prosecution, but not for delays in the prosecution of its parent application.
  3. Terminal Disclaimer: If there’s a risk of obviousness-type double patenting, a terminal disclaimer may be required, potentially limiting the term of the divisional patent to that of the parent patent.

The MPEP 201.06(c) does not directly address patent term, but it’s important to consider these factors when filing a divisional application. For more details on patent term, refer to MPEP 2701.

For more information on Divisional application, visit: Divisional application.

For more information on patent term, visit: patent term.

For more information on patent term adjustment, visit: patent term adjustment.

The disclosure requirements for claiming the benefit of provisional and nonprovisional applications differ in some aspects:

Provisional Applications (35 U.S.C. 119(e)):

For a nonprovisional application to claim the benefit of a provisional application:

  • The written description and drawings (if any) of the provisional application must adequately support and enable the subject matter claimed in the nonprovisional application.
  • A claim is not required in the provisional application.

Nonprovisional Applications (35 U.S.C. 120, 121, 365(c), or 386(c)):

For continuation and divisional applications:

  • The disclosure must be the same as the prior-filed application.
  • No new matter can be added.

For continuation-in-part applications:

  • New matter may be included.
  • Only claims supported by the prior-filed application are entitled to the earlier filing date.

As stated in MPEP 211.05:

“Under 35 U.S.C. 120, a claim in a U.S. application is entitled to the benefit of the filing date of an earlier filed U.S. application if the subject matter of the claim is disclosed in the manner provided by 35 U.S.C. 112(a) except for the best mode requirement, in the earlier filed application.”

This requirement applies to both provisional and nonprovisional applications, ensuring that the earlier application provides proper support for the claimed invention.

To learn more:

Restriction requirements can affect joint inventorship in patent applications. The MPEP explains:

If an application by joint inventors includes more than one independent and distinct invention, restriction may be required with the possible result of a necessity to change the inventorship named in the application if the elected invention was not the invention of all the originally named inventors.

In such cases:

  • A ‘divisional’ application complying with 35 U.S.C. 120 would be entitled to the benefit of the earlier filing date of the original application.
  • Examiners should remind applicants to correct inventorship under 37 CFR 1.48 if an invention is elected and claims to the invention of one or more inventors are canceled.

This process ensures that the correct inventors are named for each distinct invention in the case of a restriction requirement.

To learn more:

To learn more:

The treatment of new matter varies depending on the type of continuing application:

  • Continuation and Divisional Applications: Must not contain any new matter. The disclosure must be the same as the prior application.
  • Continuation-in-Part (CIP) Applications: Can include new matter not disclosed in the parent application.
  • Continued Prosecution Applications (CPAs): Cannot contain new matter.

For continuation and divisional applications, the MPEP states: “The disclosure presented in the continuation must not include any subject matter which would constitute new matter if submitted as an amendment to the parent application.”

For CIPs, the MPEP notes: “A continuation-in-part is an application… repeating some substantial portion or all of the prior-filed application and adding matter not disclosed in the prior-filed application.”

If new matter is found in a continuation or divisional application, the examiner will require the applicant to delete the benefit claim or change the application to a continuation-in-part. For CPAs, any new matter introduced will be required to be canceled.

It’s important to note that claims in a CIP application that rely on new matter will only be entitled to the filing date of the CIP, not the parent application’s filing date.

For more information on continuation application, visit: continuation application.

For more information on continuation-in-part application, visit: continuation-in-part application.

For more information on CPA, visit: CPA.

For more information on Divisional application, visit: Divisional application.

For more information on new matter, visit: new matter.

Filing a continuation or divisional application under 37 CFR 1.53(b) does not automatically abandon the prior application. The status of the prior application depends on the actions taken by the applicant:

  1. The prior application remains pending unless expressly abandoned.
  2. To expressly abandon the prior application, a separate paper must be filed and signed in accordance with 37 CFR 1.138.
  3. If a notice of allowance has been issued in the prior application, it can become abandoned by nonpayment of the issue fee.

The MPEP states: Under 37 CFR 1.53(b) practice, the prior nonprovisional application is not automatically abandoned upon filing of the continuing application. If the prior nonprovisional application is to be expressly abandoned, such a paper must be signed in accordance with 37 CFR 1.138.

It’s important to note that if the issue fee has been paid in the prior application, a petition to withdraw the application from issue must be filed before it can be abandoned, even if a continuing application has been filed.

For more information on continuation application, visit: continuation application.

For more information on Divisional application, visit: Divisional application.

Patent examiners carefully compare the content of the continuation or divisional application with the prior-filed application to identify any new matter. If they find information not disclosed in the prior application, they will issue a notice to the applicant.

The MPEP ยถ 2.10.01 instructs examiners:

“In bracket 1, provide an example of the matter not disclosed in the prior-filed application.”

This means the examiner must specifically point out the new matter found in the application, allowing the applicant to understand and address the issue.

For more information on continuation application, visit: continuation application.

For more information on Divisional application, visit: Divisional application.

For more information on new matter, visit: new matter.

For more information on patent examination, visit: patent examination.

Patent examiners use specific criteria to determine if an application may qualify as a divisional. According to MPEP ยถ 2.01, an examiner looks for the following indicators:

  1. The application claims only subject matter disclosed in a prior application.
  2. The claimed subject matter appears to be independent and distinct from that claimed in the prior application.
  3. At least one inventor from the prior application is named in the current application.

The MPEP provides guidance to examiners:

“This form paragraph should only be used if it appears that the application may be a divisional, but a benefit claim has not been properly established.”

If these conditions are met, but a proper benefit claim hasn’t been made, the examiner may use this paragraph to notify the applicant of the possible divisional status and the need to establish a benefit claim if desired.

For more information on Divisional application, visit: Divisional application.

For more information on patent examination, visit: patent examination.

For more information on USPTO, visit: USPTO.

To claim the benefit of a prior application’s filing date for a divisional application, applicants must follow specific procedures outlined in patent law and regulations. The MPEP ยถ 2.01 states:

Should applicant desire to claim the benefit of the filing date of the prior application, attention is directed to 35 U.S.C. 120, 37 CFR 1.78, and MPEP ยง 211 et seq.

This means applicants should:

  • Ensure compliance with 35 U.S.C. 120 requirements for continuity and disclosure
  • Follow the procedures outlined in 37 CFR 1.78 for claiming benefit
  • Consult MPEP ยง 211 for detailed guidance on benefit claims

It’s crucial to properly establish the benefit claim to secure the earlier filing date for the divisional application.

For more information on Divisional application, visit: Divisional application.

A terminal disclaimer filed in a parent application can affect a divisional application in specific ways. According to MPEP 201.06:

‘The divisional application must be directed to subject matter described and claimed in the prior nonprovisional application.’

This means that if a terminal disclaimer was filed in the parent application, it may impact the divisional application as follows:

  • Inheritance of Terminal Disclaimer: Generally, a terminal disclaimer filed in the parent application does not automatically apply to the divisional application.
  • Potential Double Patenting: If the claims in the divisional application are not patentably distinct from those in the parent application, a new terminal disclaimer may be required in the divisional to overcome a double patenting rejection.
  • Examination Considerations: Examiners will consider the relationship between the parent and divisional applications when determining if a terminal disclaimer is necessary.

It’s important to consult with a patent attorney or agent to understand the specific implications of terminal disclaimers on your divisional application, as each case can have unique considerations.

To learn more:

How does a restriction requirement lead to a divisional application?

A restriction requirement often leads to the filing of a divisional application. This process occurs as follows:

  1. An examiner issues a restriction requirement when they believe the original application contains claims to two or more independent or distinct inventions.
  2. The applicant must then elect one invention to pursue in the original application.
  3. The non-elected inventions can be pursued in one or more divisional applications.

As stated in MPEP 201.06:

“The inventions embodied in the claims of a divisional application are generally further characterized as (1) independent inventions or (2) related inventions which are distinct.”

This means that the divisional application allows the applicant to protect inventions that were disclosed but not pursued in the original application due to the restriction requirement.

For more information on Divisional application, visit: Divisional application.

For more information on patent examination, visit: patent examination.

How does a divisional application differ from other continuing applications?

A divisional application is a unique type of continuing application that focuses on a distinct invention separated from a parent application. According to the MPEP:

A divisional application is one filed for an invention which has been disclosed and claimed, but not allowed, in a prior nonprovisional application or international application designating the United States. It is a later application for an independent and distinct invention carved out of a pending application. (MPEP 201.02)

Key differences of a divisional application include:

  • It is filed in response to a restriction requirement in the parent application
  • It claims only subject matter that was disclosed but not claimed in the parent application
  • It does not add new matter
  • It allows the applicant to pursue protection for inventions that were deemed distinct in the original application

Unlike continuations or CIPs, divisional applications are specifically for pursuing separate inventions identified during the examination of the parent application.

To learn more:

Divisional applications and provisional applications are distinct types of patent applications with different purposes and characteristics:

  • Divisional Application: Claims subject matter from a prior non-provisional application that is independent and distinct from the original claims.
  • Provisional Application: A temporary application that establishes a priority date but does not mature into an issued patent.

The MPEP explicitly states in MPEP ยถ 2.01:

“An application claiming the benefit of a provisional application under 35 U.S.C. 119(e) should not be called a ‘divisional’ of the prior application.”

This distinction is important because divisional applications claim the benefit under 35 U.S.C. 120, while provisional applications are claimed under 35 U.S.C. 119(e).

For more information on Divisional application, visit: Divisional application.

For more information on patent law, visit: patent law.

For more information on provisional application, visit: provisional application.

For more information on USPTO, visit: USPTO.

A divisional application discloses and claims only subject matter disclosed in the parent application, while a continuation-in-part (CIP) application may include new matter. The MPEP states:

A continuation-in-part application should not be designated as a divisional application. The Court of Appeals for the Federal Circuit has concluded that the protection of the third sentence of 35 U.S.C. 121 does not extend to continuation-in-part applications, stating that ‘the protection afforded by section 121 to applications (or patents issued therefrom) filed as a result of a restriction requirement is limited to divisional applications.’

This means that divisional applications have certain protections against double patenting rejections that CIPs do not have.

To learn more:

A divisional application does not extend the patent term beyond that of the parent application. The term of a patent issuing from a divisional application is calculated from the filing date of the earliest application for which a benefit is claimed under 35 U.S.C. 120, 121, 365(c), or 386(c), excluding any benefit claims to provisional applications.

This means that while you can pursue additional patent protection through a divisional application, you don’t get extra time beyond the original 20-year term from the earliest non-provisional filing date.

It’s important to note that patent term adjustments may apply independently to divisional applications based on delays in their own prosecution.

To learn more:

A divisional application’s patent term adjustment (PTA) is calculated independently from its parent application. The MPEP 201.06 provides guidance on this matter:

‘A divisional application will receive its own patent term adjustment under 35 U.S.C. 154(b), which will be calculated based on the prosecution of the divisional application itself.’

Key points to understand:

  • The divisional application’s PTA is based solely on its own prosecution history
  • Any delays or adjustments from the parent application do not affect the divisional’s PTA
  • The PTA calculation starts from the filing date of the divisional application
  • Applicants should monitor the prosecution of the divisional application to ensure accurate PTA calculation

For more information on patent term adjustment, refer to MPEP 2730.

To learn more:

To learn more:

37 CFR 1.63(d) provides a simplified process for submitting an oath or declaration in a divisional application. The MPEP states:

’37 CFR 1.63(d) provides that a newly executed oath or declaration is not required in a continuation or divisional application filed by all or by fewer than all of the inventors named in a prior nonprovisional application containing a signed oath or declaration.’

This means that if the divisional application is filed by the same inventor(s) as the parent application, and there’s no new matter added, the oath or declaration from the parent application can be used for the divisional application. This simplifies the filing process and reduces paperwork.

For more information on continuation application, visit: continuation application.

For more information on Divisional application, visit: Divisional application.

For more information on oath or declaration, visit: oath or declaration.

Continuation, divisional, and continuation-in-part (CIP) applications are all types of continuing applications, but they have key differences:

  • Continuation application: Discloses and claims only subject matter disclosed in the prior application. No new matter is added.
  • Divisional application: Results from a restriction requirement in the parent application. Claims an independent and distinct invention carved out of the parent application. No new matter is added.
  • Continuation-in-part (CIP) application: Repeats a substantial portion of the prior application and adds matter not disclosed in the prior application (i.e., new matter is added).

As stated in the MPEP regarding continuation-in-part applications:

A continuation-in-part is an application filed during the lifetime of a prior-filed nonprovisional application, international application designating the United States, or international design application designating the United States repeating some substantial portion or all of the prior-filed application and adding matter not disclosed in the prior-filed application.

All three types of applications must claim the benefit of the prior application under 35 U.S.C. 120, 121, 365(c), or 386(c) and comply with other requirements set forth in 37 CFR 1.78.

For more information on continuation application, visit: continuation application.

For more information on continuation-in-part application, visit: continuation-in-part application.

For more information on Divisional application, visit: Divisional application.

For more information on new matter, visit: new matter.

For more information on patent application types, visit: patent application types.

Applicants can avoid the need for preliminary amendments in patent applications by following the guidance provided in MPEP 608.04(b):

Applicants can avoid the need to file a preliminary amendment by incorporating any desired amendments into the text of the specification, even where the application is a continuation or divisional application of a prior-filed application.

This approach involves directly including any necessary changes or additions in the original specification text when preparing the application. By doing so, applicants can ensure that all desired content is present from the initial filing, eliminating the need for separate preliminary amendments. This method is recommended for all types of applications, including continuations and divisionals, to streamline the application process and reduce potential processing delays.

To learn more:

For applications filed on or after September 16, 2012, an applicant can file a continuation or divisional application using a copy of an oath or declaration from a prior application if:

  • The oath or declaration complies with 35 U.S.C. 115 as revised effective September 16, 2012
  • It was executed by or with respect to the inventor in the earlier-filed application
  • A copy of the oath, declaration, or substitute statement showing the signature or an indication it was executed is submitted in the continuing application

As stated in the MPEP: For applications filed on or after September 16, 2012, the filing of a continuing application by all or by fewer than all of the inventors named in a prior application without a newly executed oath or declaration is permitted provided that an oath or declaration in compliance with 37 CFR 1.63, or a substitute statement under 37 CFR 1.64, was executed by or with respect to such inventor and was filed in the earlier-filed application, and a copy of such oath, declaration, or substitute statement showing the signature or an indication thereon that it was executed, is submitted in the continuing application.

It’s important to note that the oath or declaration must meet the current requirements, even if the prior application was filed before September 16, 2012.

For more information on continuation application, visit: continuation application.

For more information on declaration, visit: declaration.

For more information on Divisional application, visit: Divisional application.

For more information on oath, visit: oath.

For continuation or divisional applications filed under 37 CFR 1.53(b) of an application previously accorded status under pre-AIA 37 CFR 1.47, the process is simplified. The MPEP states: In a continuation or divisional application filed under 37 CFR 1.53(b) of an application accorded status under pre-AIA 37 CFR 1.47, if a copy of a declaration from a prior application and a copy of a decision according status under pre-AIA 37 CFR 1.47 are filed as permitted by pre-AIA 37 CFR 1.63(d)(3)(i), the notice will not be repeated.

This means that if the proper documentation from the original application is provided, the USPTO won’t repeat the notification process for nonsigning inventors. This streamlines the application process for related applications.

To learn more:

To learn more:

The treatment of continuation or divisional applications filed under former 37 CFR 1.62 depends on the filing date of the original application:

  • For applications filed on or after June 8, 1995: Requests filed on or after December 1, 1997, are treated as requests for continued examination (RCE) under 37 CFR 1.114.
  • For applications filed before June 8, 1995: Requests filed on or after December 1, 1997, are treated as improper applications.

As stated in the MPEP: A request for a continuation or divisional application filed under former 37 CFR 1.62 on or after December 1, 1997, in an application that was filed on or after June 8, 1995, will be treated as a request for continued examination (RCE) under 37 CFR 1.114, see MPEP 706.07(h), paragraph IV.

To learn more:

The treatment of continuation or divisional applications filed under former 37 CFR 1.62 depends on the filing date of the original application:

  • For applications filed on or after June 8, 1995: “A request for a continuation or divisional application filed under former 37 CFR 1.62 on or after December 1, 1997, in an application that was filed on or after June 8, 1995, will be treated as a request for continued examination (RCE) under 37 CFR 1.114.” This means these applications are now processed under the RCE procedure.
  • For applications filed before June 8, 1995: Requests for continuation or divisional applications filed on or after December 1, 1997, will be treated as improper applications.

For more details on RCE procedures, refer to MPEP 706.07(h), paragraph IV.

For more information on continuation application, visit: continuation application.

For more information on Divisional application, visit: Divisional application.

For more information on RCE, visit: RCE.

For applications filed on or after June 8, 1995, continuation or divisional applications filed under former 37 CFR 1.62 on or after December 1, 1997, are treated differently. The MPEP states: “A request for a continuation or divisional application filed under former 37 CFR 1.62 on or after December 1, 1997, in an application that was filed on or after June 8, 1995, will be treated as a request for continued examination (RCE) under 37 CFR 1.114.” This means such applications are now processed under the RCE procedure rather than the former File Wrapper Continuing Procedure.

For more information on continuation application, visit: continuation application.

For more information on Divisional application, visit: Divisional application.

For more information on RCE, visit: RCE.

According to the MPEP, A continuation or divisional application filed under 37 CFR 1.60 on or after December 1, 1997, will automatically be treated as an application filed under 37 CFR 1.53(b). This means that any continuation or divisional application filed after this date is processed under the new regulations, specifically 37 CFR 1.53(b), regardless of whether the applicant intended to file under the old 37 CFR 1.60 procedure.

To learn more:

To learn more:

Can the specification be omitted when filing a divisional or continuation application?

No, the specification cannot be omitted when filing a divisional or continuation application. However, the process is simplified. According to MPEP 201.06(c):

“A new specification (including the claims) may be submitted in a divisional application filed under 37 CFR 1.53(b). In such a case, the applicant must file a copy of the prior application as filed or a new specification.”

This means that while you must include a specification, you have two options:

  • Submit a copy of the prior application’s specification
  • Submit a new specification tailored to the divisional or continuation application

If you choose to submit a copy of the prior application, it’s important to ensure that it properly supports the claims in the new application. Any new matter introduced in a continuation application may result in a rejection under 35 U.S.C. 112(a).

Remember, the claims are considered part of the specification, so you must include claims with your application, whether you’re using the prior specification or submitting a new one.

For more information on continuation application, visit: continuation application.

For more information on Divisional application, visit: Divisional application.

For more information on patent application requirements, visit: patent application requirements.

For more information on USPTO, visit: USPTO.

No, new matter cannot be added to a divisional application. The divisional application must only contain subject matter disclosed in the parent application. The MPEP clearly states:

Thus the disclosure presented in a divisional application must not include any subject matter which would constitute new matter if submitted as an amendment to the parent application.

Any attempt to add new matter to a divisional application will be rejected under 35 U.S.C. 112(a) as lacking written description support in the parent application.

To learn more:

Can I file a priority claim in a continuing application?

Yes, you can file a priority claim in a continuing application, but there are specific requirements and deadlines to consider. According to MPEP 214.01, “In the case of a continuation or divisional application filed under 35 U.S.C. 111(a), the claim for priority must be made during the pendency of the application and within the later of four months from the actual filing date of the application or sixteen months from the filing date of the prior foreign application.” This means that even in a continuing application, you must adhere to the time limits for filing the priority claim. It’s important to note that the priority claim must be made in each continuing application; a priority claim in a parent application does not automatically carry over to child applications.

Yes, you can file a priority claim in a continuation or divisional application, but there are specific rules you must follow. According to MPEP 214.01:

‘In the case of a continuation or divisional application filed under 35 U.S.C. 111(a), the claim for priority must be made during the pendency of the application and within the four-month period set forth in 37 CFR 1.55(d)(1).’

This means:

  • The priority claim must be made while the continuation or divisional application is still pending (not abandoned or issued).
  • You have a four-month window from the actual filing date of the continuation or divisional application to make the priority claim.
  • The priority claim should refer back to the foreign application through the parent application(s).

It’s important to note that the continuation or divisional application must have copendency with the parent application to maintain the priority chain. Always consult the latest version of the MPEP and consider seeking advice from a patent attorney to ensure compliance with current USPTO rules.

To learn more:

Can I file a divisional application without a new oath or declaration?

Yes, you can file a divisional application without a new oath or declaration under certain conditions. According to MPEP 201.06(c):

“Under 37 CFR 1.63(d), a newly executed oath or declaration is not required in a divisional application filed under 37 CFR 1.53(b) if: (1) the prior nonprovisional application contained an oath or declaration as prescribed by 37 CFR 1.63 that was executed by or with respect to the inventor and was signed in compliance with 37 CFR 1.64; and (2) the divisional application was filed by all or by fewer than all of the inventors named in the prior application.”

This provision simplifies the filing process for divisional applications, allowing inventors to use the oath or declaration from the original application, provided the specified conditions are met.

For more information on declaration, visit: declaration.

For more information on Divisional application, visit: Divisional application.

For more information on oath, visit: oath.

No, you cannot amend the specification of a Continued Prosecution Application (CPA) to refer to it as a continuation or divisional application. According to MPEP ยง 201.06(d):

“The amendment filed [date] requesting that the specification be amended to refer to the present Continued Prosecution Application (CPA) as a [continuation/divisional] application of Application No. [X] has not been entered. As set forth in 37 CFR 1.53(d)(7), a request for a CPA is the specific reference required by 35 U.S.C. 120 to every application assigned the application number identified in such request. Thus, there is no need to amend the first sentence(s) of the specification to refer back to the prior application and any such amendment shall be denied entry.”

This means that the CPA request itself serves as the required reference to the prior application, and no additional amendment to the specification is necessary or allowed.

For more information on continuation application, visit: continuation application.

For more information on CPA, visit: CPA.

For more information on Divisional application, visit: Divisional application.

Can an inventor’s oath or declaration from a parent application be used in a divisional application?

Yes, an inventor’s oath or declaration from a parent application can often be used in a divisional application. According to MPEP 201.06(c):

‘In accordance with 37 CFR 1.63(d), a newly executed oath or declaration is not required in a divisional application filed under 37 CFR 1.53(b) if: (1) the divisional application claims are directed to subject matter disclosed in the prior-filed application; (2) a copy of the inventor’s oath or declaration from the prior-filed application is provided for the divisional application; and (3) the prior-filed application contains a statement by the applicant that any required oath or declaration, including any substitute statement under 37 CFR 1.64 or assignment-statement under pre-AIA 37 CFR 1.63(e), has been filed.’

This provision simplifies the filing process for divisional applications by allowing the reuse of the original oath or declaration, provided certain conditions are met. However, it’s important to ensure that all inventors named in the divisional application are covered by the oath or declaration from the parent application.

For more information on declaration, visit: declaration.

For more information on Divisional application, visit: Divisional application.

For more information on inventor’s oath, visit: inventor’s oath.

For more information on parent application, visit: parent application.

Yes, according to MPEP ยถ 2.05, an application can potentially be classified as both a continuation and a division. The MPEP states:

“Accordingly, this application may constitute a continuation or division.” (MPEP ยถ 2.05)

This dual classification possibility arises because:

  • A continuation application discloses and claims only subject matter from the prior application
  • A divisional application is filed as a result of a restriction requirement in the prior application
  • In some cases, an application may meet both criteria

The final determination of whether the application is a continuation, division, or both will depend on the specific circumstances of the application and its relationship to the prior application. The examiner will make this determination during the examination process.

For more information on continuation application, visit: continuation application.

For more information on Divisional application, visit: Divisional application.

No, a provisional application cannot be called a ‘divisional’ application. The MPEP ยถ 2.01 explicitly states:

An application claiming the benefit of a provisional application under 35 U.S.C. 119(e) should not be called a ‘divisional’ of the prior application.

This means that while an application can claim the benefit of a provisional application’s filing date under 35 U.S.C. 119(e), it is not considered a divisional application. Divisional applications are specifically related to non-provisional applications and involve claiming independent and distinct inventions from a parent application.

For more information on 35 USC 119(e), visit: 35 USC 119(e).

For more information on Divisional application, visit: Divisional application.

For more information on provisional application, visit: provisional application.

Can a priority claim be made in a continuation or divisional application?

Yes, a priority claim can be made in a continuation or divisional application. However, there are specific requirements and time limits to consider. According to MPEP 214.01:

“In the case of a continuation or divisional application, the claim for priority must be made during the pendency of the application.”

This means:

  • The priority claim must be filed within the later of four months from the actual filing date of the continuation or divisional application or sixteen months from the filing date of the prior foreign application.
  • If the claim is filed outside this period, it will be considered untimely and require a petition under 37 CFR 1.55(e).
  • The priority claim should reference the parent application if it properly claimed the benefit of the foreign application.

It’s important to note that the continuation or divisional application must be filed within 12 months of the foreign application’s filing date (or 6 months for design applications) to be eligible for priority.

Yes, a divisional or continuation application can claim benefit from multiple prior applications. The MPEP 201.06(c) states:

‘A continuation or divisional application may be filed under 35 U.S.C. 111(a) using the procedures set forth in 37 CFR 1.53(b), by providing: (A) a copy of the prior application as filed; and (B) a new specification, claim or claims and drawings, where applicable.’

This provision allows for claiming benefit from multiple prior applications by following the correct procedures. It’s crucial to ensure that the chain of priority is properly maintained and that all necessary documents and declarations are filed.

To learn more:

To learn more:

No, a divisional application cannot include new matter that was not disclosed in the parent application. The MPEP 201.06 clearly states:

‘A divisional application is a later application for an independent or distinct invention, carved out of a prior application and disclosing and claiming only subject matter disclosed in the prior application.’

This means:

  • The divisional application must be limited to the subject matter disclosed in the parent application
  • No new matter can be added to the divisional application
  • The claims in the divisional must be supported by the disclosure of the parent application

If an applicant wishes to add new matter, they should consider filing a continuation-in-part (CIP) application instead of a divisional. For more information on new matter, refer to MPEP 2163.

To learn more:

Yes, a divisional application can claim the benefit of the filing date of the prior application. The MPEP ยถ 2.01 states:

Should applicant desire to claim the benefit of the filing date of the prior application, attention is directed to 35 U.S.C. 120, 37 CFR 1.78, and MPEP ยง 211 et seq.

To claim this benefit, the applicant must meet the requirements set forth in 35 U.S.C. 120 and 37 CFR 1.78, which include making a proper benefit claim and ensuring the divisional application is filed before the patenting or abandonment of the prior application.

For more information on Divisional application, visit: Divisional application.

For more information on patent law, visit: patent law.

Yes, a divisional application can claim priority to its parent application. This is one of the key benefits of filing a divisional application. According to MPEP 201.06:

‘A divisional application is entitled to the benefit of the filing date of the prior application.’

This means that the divisional application:

  • Retains the priority date of the parent application for the subject matter disclosed in the parent
  • Can claim the benefit under 35 U.S.C. 120 or 365(c)
  • Must include a specific reference to the parent application in its application data sheet (ADS)

It’s important to note that the divisional application must be filed while the parent application is still pending to claim this benefit.

For more information on 35 U.S.C. 120, visit: 35 U.S.C. 120.

For more information on Divisional application, visit: Divisional application.

For more information on patent law, visit: patent law.

Yes, a divisional application can claim priority to a provisional application under certain conditions. According to MPEP 201.06:

‘A divisional application may claim the benefit of a provisional application under 35 U.S.C. 119(e) if the divisional application is filed within twelve months of the provisional application filing date (or an appropriate later date in accordance with 37 CFR 1.78).’

To claim this benefit, the divisional application must:

  • Be filed within 12 months of the provisional application’s filing date (or later if permitted under 37 CFR 1.78)
  • Include a proper reference to the provisional application
  • Have at least one common inventor with the provisional application

To learn more:

To learn more:

Yes, a divisional application can claim the benefit of its parent application’s filing date. According to MPEP 201.06(c):

“A divisional application is entitled to the benefit of the filing date of the prior-filed application if the prior-filed application discloses the invention claimed in the divisional application in the manner provided by the first paragraph of 35 U.S.C. 112.”

This means that as long as the invention claimed in the divisional application was adequately described in the parent application, it can claim the parent’s filing date. This is crucial for establishing priority and avoiding potential prior art issues.

For more information on Divisional application, visit: Divisional application.

Can a divisional application claim benefit from a provisional application?

Yes, a divisional application can claim benefit from a provisional application through its parent application. The MPEP 201.06 states: “A later-filed application may be filed as a continuation, divisional, or continuation-in-part of a prior nonprovisional application or international application designating the United States. Unless the filing date of the earlier nonprovisional application is to be claimed for prior art purposes, there is no need for the later-filed application to be copending with it.” This means that if the parent application claimed benefit from a provisional application, the divisional application can also claim that benefit, extending its effective filing date back to the provisional application’s filing date.

To learn more:

Can a divisional application be filed without a restriction requirement?

While divisional applications are typically filed in response to a restriction requirement, they can be filed voluntarily without one. The MPEP 201.06 states:

“A divisional application is often filed as a result of a restriction requirement made by the examiner.”

However, the use of “often” implies that this is not always the case. Applicants may choose to file a divisional application voluntarily if they:

  • Recognize distinct inventions in their application
  • Want to pursue different claim scopes separately
  • Need to address potential unity of invention issues proactively

It’s important to note that voluntarily filing a divisional application without a restriction requirement may affect the application of the safe harbor provision under 35 U.S.C. 121, which protects against double patenting rejections in certain cases.

For more information on Divisional application, visit: Divisional application.

Can a divisional application be filed after the parent application is abandoned?

Yes, a divisional application can be filed after the parent application is abandoned, as long as certain conditions are met. The MPEP 201.06 states:

“A divisional application may be filed after abandonment of the parent application, provided the parent application is not relied upon for benefit claims under 35 U.S.C. 120, 121, 365(c), or 386(c) in an application whose effective filing date is on or after March 16, 2013.”

This means that you can file a divisional application even if the parent application has been abandoned. However, it’s crucial to note that if you want to claim the benefit of the parent application’s filing date, and the effective filing date of your divisional application is on or after March 16, 2013, the parent application must not have been abandoned at the time of filing the divisional.

It’s important to consult with a patent attorney to ensure compliance with all requirements when filing a divisional application, especially in cases involving abandoned parent applications.

For more information on Divisional application, visit: Divisional application.

For more information on effective filing date, visit: effective filing date.

No, a copy of the original oath or declaration cannot be used in a continuation or divisional application filed under 37 CFR 1.53(b). A new oath or declaration must be filed.

According to MPEP 602.02: ‘A new oath or declaration in compliance with 37 CFR 1.63 or 1.67 is required in a continuation or divisional application filed under 37 CFR 1.53(b), regardless of whether a copy of the oath or declaration from the prior application is submitted.’

This requirement ensures that the inventors reaffirm their commitment to the claims in the new application, which may differ from those in the parent application.

To learn more:

Yes, certain applications are still processed under former 37 CFR 1.62. Specifically, all continuation, divisional, and continuation-in-part (CIP) applications filed under former 37 CFR 1.62 prior to December 1, 1997, continue to be processed and examined under the procedures set forth in that regulation.

The MPEP clarifies this point: All continuation, divisional and CIP applications filed under former 37 CFR 1.62 prior to December 1, 1997, will continue to be processed and examined under the procedures set forth in former 37 CFR 1.62.

For more detailed information on the practice and procedure under former 37 CFR 1.62, the MPEP directs readers to consult MPEP ยง 201.06(b) in the MPEP 8th Edition, Rev. 1 (February 2003), which is available on the USPTO website.

To learn more:

To learn more:

For continuation or divisional applications filed before September 16, 2012, a new oath or declaration is generally not required if certain conditions are met. According to pre-AIA 37 CFR 1.63(d):

A newly executed oath or declaration is not required under ยง 1.51(b)(2) and ยง 1.53(f) in a continuation or divisional application, provided that:

  • The prior nonprovisional application contained a proper oath or declaration
  • The continuation or divisional application was filed by all or fewer than all inventors named in the prior application
  • The specification and drawings contain no new matter
  • A copy of the executed oath or declaration from the prior application is submitted

However, a new oath or declaration is required if naming an inventor not named in the prior application. The MPEP states: A newly executed oath or declaration must be filed in a continuation or divisional application naming an inventor not named in the prior application.

To learn more:

Patent Procedure (109)

Patent examiners should use the MPEP ยถ 2.01 form paragraph under specific circumstances. The MPEP provides the following guidance:

Examiner Note: […] 2. This form paragraph should only be used if it appears that the application may be a divisional, but a benefit claim has not been properly established.

This means examiners should use this paragraph when:

  • The application appears to contain subject matter that is independent and distinct from a prior application
  • The application seems to qualify as a divisional
  • The applicant has not yet properly claimed the benefit of the prior application’s filing date

The form paragraph serves as a prompt for the applicant to properly establish the benefit claim if they intend the application to be a divisional.

For more information on Divisional application, visit: Divisional application.

A new oath or declaration is required in a patent application under the following circumstances:

  • When filing a continuation or divisional application
  • When filing a continuation-in-part application
  • To correct inventorship in an application
  • When the original oath or declaration was defective

According to MPEP 602.02: ‘A new oath or declaration is required in a continuation or divisional application filed under 37 CFR 1.53(b) or in a continuation-in-part application.’ Additionally, the MPEP states that ‘A new oath or declaration in compliance with 37 CFR 1.63 or 1.67 is required in a continuation or divisional application filed under 37 CFR 1.53(b), regardless of whether a copy of the oath or declaration from the prior application is submitted.’

To learn more:

A divisional application is often filed in response to a restriction requirement made by the patent examiner. As stated in the MPEP:

A divisional application is often filed as a result of a restriction requirement made by the examiner.

When an examiner determines that an application contains multiple independent and distinct inventions, they may issue a restriction requirement. The applicant can then elect one invention to pursue in the original application and file a divisional application to pursue the non-elected invention(s).

To learn more:

For continuation or divisional applications filed under 37 CFR 1.53(d) (continued prosecution design applications) with changes in inventorship, the requirements were as follows:

“If an inventor named in a prior application is not an inventor in a continuation or divisional application filed under 37 CFR 1.53(d) (continued prosecution design application), the request for filing the continuation or divisional application must be accompanied by a statement requesting the deletion of the name or names of the person or persons who are not inventors of the invention being claimed in the continuation or divisional application (see 37 CFR 1.53(d)(4)).”

This meant that for continued prosecution design applications, a statement requesting the deletion of non-inventors had to be submitted along with the filing request when there were changes in inventorship.

To learn more:

To learn more:

Continued Prosecution Applications (CPAs) are specifically for design patent applications. The MPEP ยถ 2.34 refers to CPAs in the context of design applications.

The MPEP ยถ 2.34 states: “In bracket 2, insert either –continuation– or –divisional–.”

This indicates that a CPA can be filed as either a continuation or a divisional of a prior design application. It’s important to note that CPAs are no longer available for utility or plant patent applications.

For more information on continuation application, visit: continuation application.

For more information on design patents, visit: design patents.

For more information on Divisional application, visit: Divisional application.

If an inventor’s signature was missing from a continuation or divisional application, the Office of Patent Application Processing (OPAP) would typically take the following steps:

  1. Send a “Notice to File Missing Parts” requiring the signature of the nonsigning inventor.
  2. If a copy of the decision according status under pre-AIA 37 CFR 1.47 was not included with the original filing, applicants could respond to the Notice by:
    • Submitting a copy of the decision according status under pre-AIA 37 CFR 1.47, along with a surcharge for late filing.
    • Alternatively, submitting an oath or declaration signed by the previously nonsigning inventor, along with the required surcharge.

This process is described in the MPEP:

“If OPAP mails such a Notice, a copy of the decision according status under pre-AIA 37 CFR 1.47, together with a surcharge under 37 CFR 1.16(f) for its late filing, will be an acceptable reply to the Notice. Alternatively, applicant may submit an oath or declaration signed by the previously nonsigning inventor together with the surcharge set forth in 37 CFR 1.16(f) in reply to the Notice.”

To learn more:

To claim the benefit of a prior application’s filing date for a divisional application, applicants must follow specific procedures. The MPEP ยถ 2.01 provides guidance:

Should applicant desire to claim the benefit of the filing date of the prior application, attention is directed to 35 U.S.C. 120, 37 CFR 1.78, and MPEP ยง 211 et seq.

This means applicants should:

  • Review 35 U.S.C. 120 for statutory requirements
  • Consult MPEP ยง 211 for detailed guidance on claiming benefit
  • Follow the procedures outlined in 37 CFR 1.78 for claiming benefit of an earlier-filed application

It’s crucial to properly establish the benefit claim to secure the earlier filing date for the divisional application.

For more information on Divisional application, visit: Divisional application.

For more information on USPTO procedures, visit: USPTO procedures.

When a patent examiner encounters an application that appears to be a divisional but lacks a proper benefit claim, they should follow the guidance provided in MPEP ยถ 2.01. The MPEP states:

“This form paragraph should only be used if it appears that the application may be a divisional, but a benefit claim has not been properly established.”

In such cases, the examiner should:

  1. Recognize that the application may potentially be a divisional based on its content and relationship to a prior application.
  2. Note that the applicant has not properly established a benefit claim to the prior application.
  3. Use the appropriate form paragraph (as mentioned in MPEP ยถ 2.01) to inform the applicant about the potential divisional status and the need to establish a proper benefit claim.
  4. Direct the applicant’s attention to the relevant laws and regulations for claiming benefit, specifically: 35 U.S.C. 120, 37 CFR 1.78, and MPEP ยง 211 et seq.

By following these steps, the examiner ensures that the applicant is aware of the potential divisional status and has the opportunity to properly establish the benefit claim if desired.

For more information on Divisional application, visit: Divisional application.

What is the time limit for filing a divisional application?

There is no specific time limit for filing a divisional application, but there are important considerations that effectively create a practical time frame:

  1. Parent Application Status: The divisional application must be filed while the parent application is still pending. Once the parent application is either abandoned or issued as a patent, it’s no longer possible to file a divisional from it.
  2. Priority Claim: To claim the benefit of the parent application’s filing date, the divisional must be filed within the time periods specified in 35 U.S.C. 120, 121, 365(c), or 386(c).
  3. Patent Term: The 20-year patent term is calculated from the earliest non-provisional U.S. filing date. Filing a divisional later in the parent’s pendency will result in a shorter potential patent term for the divisional.

The MPEP 201.06 states:

A divisional application is often filed as a result of a restriction requirement made by the examiner.

While this doesn’t specify a time limit, it implies that divisional applications are typically filed in response to office actions during the examination process. Applicants should consider filing a divisional promptly after receiving a restriction requirement to ensure they don’t miss any deadlines and to maximize the potential patent term.

For more information on Divisional application, visit: Divisional application.

For more information on filing deadline, visit: filing deadline.

What is the significance of the term ‘carved out’ in relation to divisional applications?

The term ‘carved out’ is significant in understanding the nature of divisional applications. According to MPEP 201.06: ‘A later application for an independent or distinct invention, carved out of a pending application and disclosing and claiming only subject matter disclosed in the earlier or parent application, is known as a divisional application or ‘division.”

The significance of ‘carved out’ includes:

  • It implies that the divisional application is derived from a portion of the parent application.
  • The divisional application focuses on a specific invention or set of claims that are distinct from those in the parent application.
  • The subject matter in the divisional is entirely based on the disclosure in the parent application.
  • No new matter can be added to the divisional application beyond what was disclosed in the parent.

This term emphasizes that while the divisional application is separate, it is intrinsically linked to the parent application’s original disclosure, ensuring that the applicant doesn’t gain an unfair advantage by introducing new subject matter.

For more information on Divisional application, visit: Divisional application.

For more information on invention disclosure, visit: invention disclosure.

For more information on patent filing, visit: patent filing.

What is the significance of the filing date in a divisional application?

The filing date of a divisional application is crucial for several reasons:

  • Benefit of earlier filing date: A divisional application is entitled to the benefit of the filing date of the prior-filed application, as stated in MPEP 201.06(c): ‘A divisional application is entitled to the benefit of the filing date of the prior-filed application if filed before the patenting or abandonment of or termination of proceedings on the prior-filed application.’
  • Prior art considerations: The earlier filing date can be used to overcome prior art references that might otherwise be applicable.
  • Patent term calculations: The filing date may affect the calculation of the patent term.
  • Priority claims: It establishes the basis for any priority claims to foreign applications.

It’s important to note that the divisional application must be filed while the parent application is still pending to claim the benefit of the earlier filing date.

For more information on Divisional application, visit: Divisional application.

For more information on patent term, visit: patent term.

For more information on prior art, visit: prior art.

What is the significance of the ‘original disclosure’ in a divisional application?

The ‘original disclosure’ is crucial in a divisional application because it defines the scope of what can be included in the divisional. According to MPEP 201.06:

‘The disclosure of a divisional application must be the same as the disclosure of the prior-filed application, i.e., the original disclosure of the parent or original nonprovisional application or provisional application.’

This means that no new matter can be added to a divisional application. The divisional must rely entirely on the content disclosed in the original parent application. This ensures that the divisional application doesn’t extend beyond the scope of what was initially invented and disclosed.

For more information on Divisional application, visit: Divisional application.

For more information on new matter, visit: new matter.

Claiming benefit to a prior application is crucial for divisional applications as it allows the divisional to retain the earlier filing date of the parent application. This can be significant for:

  • Establishing priority over potential prior art
  • Extending the potential patent term
  • Maintaining continuity in the patent family

The MPEP ยถ 2.01 states:

“Should applicant desire to claim the benefit of the filing date of the prior application, attention is directed to 35 U.S.C. 120, 37 CFR 1.78, and MPEP ยง 211 et seq.”

To properly claim this benefit, applicants must comply with the requirements set forth in 35 U.S.C. 120, 37 CFR 1.78, and MPEP ยง 211, which include making a proper benefit claim and ensuring continuity of disclosure.

For more information on Divisional application, visit: Divisional application.

For more information on patent law, visit: patent law.

What is the required oath or declaration for a divisional application?

For a divisional application, a new oath or declaration is generally required. However, there are exceptions:

  • If the application is filed on or after September 16, 2012, a copy of the oath or declaration from the prior nonprovisional application may be used, provided that it complies with 37 CFR 1.63, 1.64, or 1.67 for the divisional application.
  • If the divisional application was filed before September 16, 2012, and the prior application was filed on or after September 16, 2012, a copy of the oath or declaration from the prior application can be used in the divisional application.

As stated in MPEP 201.06: “A new oath or declaration is required in a divisional application filed under 37 CFR 1.53(b), regardless of whether a copy of the oath or declaration from the prior nonprovisional application is used in the divisional application, unless the divisional application is filed on or after September 16, 2012.”

To learn more:

What is the relationship between a divisional application and its parent application?

A divisional application is closely related to its parent application, as it stems from the same invention disclosure. According to MPEP 201.06, “A divisional application is a later application for an independent or distinct invention, carved out of a pending application and disclosing and claiming only subject matter disclosed in the earlier or parent application.” This means that:

  • The divisional application contains a subset of the subject matter from the parent application.
  • It claims an independent or distinct invention from what is claimed in the parent application.
  • It cannot introduce new matter beyond what was disclosed in the parent application.
  • It can claim the benefit of the parent application’s filing date.

The relationship allows for separate prosecution of different aspects of an invention while maintaining the priority date of the original disclosure.

To learn more:

What is the relationship between a divisional application and a restriction requirement?

A divisional application is often filed in response to a restriction requirement issued by the USPTO. The MPEP 201.06 explains:

“A later application for an independent or distinct invention, carved out of a pending application and disclosing and claiming only subject matter disclosed in the earlier or parent application, is known as a divisional application or ‘division.'”

The relationship between a divisional application and a restriction requirement is as follows:

  • When an examiner determines that a single application contains two or more independent or distinct inventions, they may issue a restriction requirement.
  • The applicant must then elect one invention to pursue in the original application.
  • The non-elected invention(s) can be pursued in one or more divisional applications.
  • Divisional applications allow applicants to protect multiple inventions disclosed in a single parent application without violating the principle of double patenting.

It’s important to note that while divisional applications are often filed in response to restriction requirements, they can also be filed voluntarily if the applicant recognizes distinct inventions in their application before receiving a restriction requirement.

To learn more:

What is the relationship between a divisional application and a continuation application?

Divisional applications and continuation applications are both types of continuing applications, but they serve different purposes. The MPEP 201.06 provides insight into the nature of divisional applications:

“A later application for an independent or distinct invention, carved out of a pending application and disclosing and claiming only subject matter disclosed in the earlier or parent application, is known as a divisional application or ‘division.'”

In contrast, a continuation application typically contains the same disclosure as the parent application but with different claims. The key differences are:

  • Divisional Application: Filed to pursue distinct inventions that were subject to a restriction requirement in the parent application.
  • Continuation Application: Filed to pursue additional aspects of the same invention disclosed in the parent application.

Both types of applications claim the benefit of the parent application’s filing date, but they differ in their purpose and the scope of claims they can contain. It’s important to choose the appropriate type of continuing application based on the specific circumstances and inventive concepts you wish to protect.

For more information on continuation application, visit: continuation application.

For more information on Divisional application, visit: Divisional application.

What is the purpose of a divisional application under 37 CFR 1.53(b)?

A divisional application under 37 CFR 1.53(b) serves to pursue claims to a distinct invention that was disclosed but not claimed in a parent application. According to the MPEP:

“A divisional application is often filed as a result of a restriction requirement made by the examiner.”

This allows inventors to protect multiple inventions that may have been included in a single original application but were deemed to be separate and distinct by the USPTO. Divisional applications maintain the benefit of the filing date of the parent application for the common subject matter.

For more information on Divisional application, visit: Divisional application.

For more information on patent filing, visit: patent filing.

What is the purpose of a divisional application in patent law?

A divisional application is a type of patent application that allows an inventor to pursue protection for different aspects of an invention that were disclosed but not claimed in a parent application. According to MPEP 201.06:

A later application for an independent or distinct invention, carved out of a pending application and disclosing and claiming only subject matter disclosed in the earlier or parent application, is known as a divisional application or ‘division.’

The main purposes of a divisional application are:

  • To protect multiple inventions disclosed in a single parent application
  • To respond to a restriction requirement issued by the USPTO
  • To pursue broader or different claim scope than what was allowed in the parent application

Divisional applications are filed under 37 CFR 1.53(b) and can benefit from the filing date of the parent application for the common subject matter.

For more information on Divisional application, visit: Divisional application.

For more information on patent law, visit: patent law.

What is the procedure for filing a continuation or divisional application without a new oath or declaration?

When filing a continuation or divisional application, a new oath or declaration is not always required. According to MPEP 602.05, the following procedure can be followed:

  1. File a copy of the oath or declaration from the prior application.
  2. Include a statement requesting the filing of the continuation or divisional application without a newly executed oath or declaration.
  3. Pay the filing fee for the continuation or divisional application.

The MPEP states: ‘The applicant may file a continuation or divisional application by filing a copy of the oath or declaration from the prior application, accompanied by a statement requesting the filing of the continuation or divisional application without a newly executed oath or declaration and payment of the filing fee.’

This procedure simplifies the filing process for certain types of continuing applications, reducing paperwork and potentially saving time for applicants.

To learn more:

What is the filing date requirement for a divisional application?

A divisional application must be filed before the patenting or abandonment of or termination of proceedings on the original application or on an earlier-filed divisional application directed to the same elected invention. The MPEP states:

A divisional application is often filed as a result of a restriction requirement made by the examiner. The divisional application may be filed under 37 CFR 1.53(b) (or 1.53(d) if the application is a design application) before the patenting or abandonment of or termination of proceedings on the original application or on an earlier-filed divisional application directed to the same elected invention.”

This means that the divisional application must be filed while the parent application is still pending, ensuring continuity in the examination process.

For more information on Divisional application, visit: Divisional application.

For more information on patent procedure, visit: patent procedure.

What is the filing date of a divisional application?

A divisional application is entitled to the filing date of the parent application. According to MPEP 201.06, “A divisional application is entitled to the benefit of the filing date of the prior-filed application if filed as a nonprovisional application that discloses and claims only subject matter disclosed in the prior-filed (parent) application.” This means that the divisional application inherits the filing date of its parent application, provided it meets the necessary requirements.

To learn more:

According to MPEP 201.06(c), the filing date for a divisional or continuation application filed under 37 CFR 1.53(b) is determined as follows:

‘The filing date of an application filed under 37 CFR 1.53(b) is the date on which a specification, with or without claims, is received in the Office.’

This means that the filing date is established when the USPTO receives the specification, regardless of whether claims are included. It’s important to note that other application components, such as drawings or fees, are not required to secure the filing date.

To learn more:

To learn more:

Incorporating by reference a prior application in a continuation or divisional application can provide important benefits:

  1. It allows the applicant to amend the continuing application to include subject matter from the prior application without the need for a petition, provided the continuing application is entitled to a filing date.
  2. For applications filed on or after September 21, 2004, it can help recover inadvertently omitted material.

The MPEP states: An applicant may incorporate by reference the prior application by including, in the continuing application-as-filed, an explicit statement that such specifically enumerated prior application or applications are “hereby incorporated by reference.” The statement must appear in the specification.

It’s important to note that an incorporation by reference statement added after an application’s filing date is not effective because no new matter can be added to an application after its filing date (see 35 U.S.C. 132(a)).

For more information on 37 cfr 1.57, visit: 37 cfr 1.57.

For more information on continuation application, visit: continuation application.

For more information on Divisional application, visit: Divisional application.

For more information on incorporation by reference, visit: incorporation by reference.

A preliminary amendment filed with a continuation or divisional application can have significant effects:

  1. If present on the filing date, it is considered part of the original disclosure.
  2. It must comply with the requirements of 37 CFR 1.121 for amendments.
  3. The Office may require a substitute specification for extensive preliminary amendments.
  4. The application will be classified and assigned based on the claims that will be before the examiner after entry of the preliminary amendment.

The MPEP states: Any preliminary amendment that is present on the filing date of an application filed under 37 CFR 1.53(b) is part of the original disclosure.

However, applicants should be cautious about introducing new matter through a preliminary amendment. The MPEP advises: Where a copy of the oath or declaration from a prior application was filed in a continuation or divisional application, if the examiner determines that new matter is present relative to the prior application, the examiner should so notify the applicant in the next Office action (preferably the first Office action). The examiner should require applicant to delete the benefit claim or redesignate the application as a continuation-in-part.

For more information on continuation application, visit: continuation application.

For more information on Divisional application, visit: Divisional application.

For more information on new matter, visit: new matter.

What is the difference between a divisional and a continuation application?

While both divisional and continuation applications are types of continuing applications, they serve different purposes:

  • Divisional Application: Filed to pursue claims to an invention that was not elected in a previous application due to a restriction requirement.
  • Continuation Application: Filed to pursue additional claims to the same invention disclosed in the parent application.

The MPEP 201.06(c) states: “A divisional application is often filed as a result of a restriction requirement made by the examiner.” In contrast, a continuation typically doesn’t arise from a restriction requirement but rather from the applicant’s desire to pursue additional aspects of the same invention.

Both types can be filed under 37 CFR 1.53(b) and may claim the benefit of the parent application’s filing date.

For more information on continuation application, visit: continuation application.

For more information on Divisional application, visit: Divisional application.

A continuation application and a divisional application are both types of continuing applications, but they serve different purposes:

  • Continuation application: Used to pursue additional claims to an invention disclosed in a prior application. It contains the same disclosure as the parent application.
  • Divisional application: Used to pursue claims to an invention that was disclosed but not claimed in a prior application. It typically results from a restriction requirement in the parent application.

As stated in MPEP 201.07: “A continuation application is an application for the invention(s) disclosed in a prior-filed copending nonprovisional application, international application designating the United States, or international design application designating the United States.” In contrast, a divisional application focuses on a distinct invention that was not claimed in the parent application.

To learn more:

A divisional application is defined in MPEP 201.06 as follows:

‘A later application for a distinct or independent invention, carved out of a pending application and disclosing and claiming only subject matter disclosed in the earlier or parent application, is known as a divisional application or ‘division.’

This definition highlights that a divisional application:

  • Is filed after an initial application
  • Focuses on a distinct invention
  • Only includes subject matter from the original application
  • Is ‘carved out’ of the pending parent application

For more information on Divisional application, visit: Divisional application.

For more information on patent application types, visit: patent application types.

Filing a divisional application allows an applicant to pursue protection for distinct inventions disclosed in a parent application. Key benefits include:

  • Ability to claim different aspects or embodiments of the invention
  • Potential for broader patent coverage
  • Preservation of priority date for the disclosed subject matter
  • Opportunity to pursue claims that may have been restricted in the parent application

The MPEP indicates: A divisional application is often filed as a result of a restriction requirement made by the examiner. This suggests that divisional applications provide a way to pursue protection for inventions that were not elected in the parent application due to a restriction requirement.

For more information on Divisional application, visit: Divisional application.

For more information on patent strategy, visit: patent strategy.

What is the benefit of filing a divisional application under 37 CFR 1.53(b)?

Filing a divisional application under 37 CFR 1.53(b) offers several benefits:

  • It allows you to pursue claims to an invention that was not elected in a previous application due to a restriction requirement.
  • You can file the divisional application without a new oath or declaration if you meet certain conditions.
  • The divisional application can claim the benefit of the filing date of the prior application.

As stated in the MPEP: “A divisional application is often filed as a result of a restriction requirement made by the examiner.” This means you can protect additional aspects of your invention that weren’t covered in the original application.

For more information on Divisional application, visit: Divisional application.

For more information on patent filing, visit: patent filing.

A divisional application is a later application for an independent or distinct invention, carved out of a nonprovisional application. It discloses and claims only subject matter disclosed in the earlier or parent application. As stated in MPEP 201.06:

A later application for an independent or distinct invention, carved out of a nonprovisional application (including a nonprovisional application resulting from an international application or international design application), an international application designating the United States, or an international design application designating the United States and disclosing and claiming only subject matter disclosed in the earlier or parent application, is known as a divisional application.

To learn more:

A divisional application under 37 CFR 1.53(b) is a type of continuation application that is filed to pursue a distinct invention that was disclosed but not claimed in a prior nonprovisional application. According to the MPEP:

‘A divisional application is often filed as a result of a restriction requirement made by the examiner.’

Divisional applications allow inventors to protect multiple inventions disclosed in a single parent application by filing separate applications for each invention.

For more information on continuation application, visit: continuation application.

For more information on Divisional application, visit: Divisional application.

A divisional application is a type of patent application that claims subject matter that is independent and distinct from that claimed in a prior application. It typically arises when an original application contains multiple inventions, and the Patent Office requires the applicant to restrict the claims to a single invention.

According to MPEP ยถ 2.01, a divisional application:

“appears to claim only subject matter directed to an invention that is independent and distinct from that claimed in the prior application, and names the inventor or at least one joint inventor named in the prior application.”

To claim the benefit of the filing date of the prior application, applicants must comply with 35 U.S.C. 120, 37 CFR 1.78, and MPEP ยง 211 et seq.

For more information on Divisional application, visit: Divisional application.

For more information on patent law, visit: patent law.

For more information on USPTO, visit: USPTO.

A continuation or divisional application is a type of continuing application filed under 37 CFR 1.53(b). According to the MPEP, An application filed under 37 CFR 1.53(b) may be an original or a reissue, a continuation, a divisional, a continuation-in-part, or a substitute. These applications are filed under 35 U.S.C. 111(a) and can be for utility, design, plant, or reissue patents.

To learn more:

To learn more:

What happens to the priority date in a divisional application?

The priority date of a divisional application remains the same as its parent application, provided that the proper procedures are followed. According to MPEP 201.06:

‘A later-filed application which is filed before the patenting or abandonment of or termination of proceedings on an earlier-filed application and which is an application for an invention which is disclosed and claimed in the earlier-filed application is an application for a ‘divisional’ of the earlier-filed application.’

This means that:

  • The divisional application retains the priority date of the parent application for the subject matter disclosed in the parent.
  • This is crucial for establishing the effective filing date for prior art purposes under 35 U.S.C. 100(i).
  • The applicant must ensure that a proper reference to the parent application is made in the divisional to maintain this priority.

Maintaining the original priority date is one of the key advantages of filing a divisional application, as it allows the applicant to protect different aspects of their invention without losing the benefit of the earlier filing date.

For more information on Divisional application, visit: Divisional application.

For more information on effective filing date, visit: effective filing date.

What happens to the parent application when a divisional application is filed?

When a divisional application is filed, the parent application continues to be prosecuted independently. The filing of a divisional application does not automatically affect the status or prosecution of the parent application.

As stated in MPEP 201.06: “The divisional application may be filed either while the original application is still pending or after it has been patented.” This means that:

  • If the parent application is pending, it remains pending and continues through the examination process.
  • If the parent application has already been granted as a patent, it remains a valid patent.

It’s important to note that while the divisional application is independent, it must contain claims to a different invention than the parent application. The examiner will ensure that there is no double patenting between the parent and divisional applications.

Additionally, if the divisional application is filed in response to a restriction requirement in the parent application, the applicant should consider canceling the claims in the parent application that are now being pursued in the divisional to avoid potential issues with duplicate claims.

For more information on Divisional application, visit: Divisional application.

For more information on parent application, visit: parent application.

For more information on patent prosecution, visit: patent prosecution.

Terminal disclaimers filed in a parent application do not automatically carry over to a divisional application. The MPEP 201.06 states:

‘A terminal disclaimer filed to obviate a nonstatutory double patenting rejection in a parent application does not carry over to a divisional application. The applicant filing the divisional application must determine whether a new terminal disclaimer needs to be filed in the divisional application.’

This means:

  • Terminal disclaimers from the parent application are not automatically applied to the divisional
  • The applicant must assess whether a new terminal disclaimer is necessary for the divisional application
  • If required, a new terminal disclaimer must be filed specifically for the divisional application

It’s important for applicants to review potential double patenting issues in the divisional application and file new terminal disclaimers if needed to overcome any rejections.

To learn more:

What happens to continuation or divisional applications filed after a CPA in a design application?

Continuation or divisional applications filed after a Continued Prosecution Application (CPA) in a design application are treated differently than those filed after a CPA in a utility or plant application. According to MPEP 201.06(d):

Any continuation or divisional application filed under 37 CFR 1.53(b) after the filing of a CPA of a design application will be assigned a new application number and will reflect the filing date of the 37 CFR 1.53(b) application.

This means that unlike utility or plant applications, where a continuation or divisional application would get the benefit of the CPA’s filing date, in design applications, these subsequent applications are treated as new applications with their own filing dates.

For more information on continuation application, visit: continuation application.

For more information on CPA, visit: CPA.

For more information on Divisional application, visit: Divisional application.

Affidavits or declarations from a prior application do not automatically become part of a continuation or divisional application filed under 37 CFR 1.53(b). If an applicant wants to rely on these documents, they must take specific actions:

  1. Make remarks of record in the new application referring to the affidavit or declaration.
  2. Include a copy of the original affidavit or declaration filed in the prior application.

The MPEP states: Affidavits or declarations, such as those submitted under 37 CFR 1.130, 1.131 and 1.132 filed during the prosecution of the prior nonprovisional application do not automatically become a part of a continuation or divisional application filed under 37 CFR 1.53(b). Where it is desired to rely on an earlier filed affidavit or declaration, the applicant should make such remarks of record in the 37 CFR 1.53(b) application and include a copy of the original affidavit or declaration filed in the prior nonprovisional application.

It’s important to note that this rule doesn’t apply to continued prosecution applications (CPAs) filed under 37 CFR 1.53(d), where affidavits and declarations do automatically become part of the new application.

For more information on continuation application, visit: continuation application.

For more information on Divisional application, visit: Divisional application.

Introducing new matter in a divisional application is not allowed and can have serious consequences. According to MPEP 201.06:

“A divisional application is a later application for an independent or distinct invention, carved out of a pending application and disclosing and claiming only subject matter disclosed in the earlier or parent application.”

This means that a divisional application must not contain any new matter beyond what was disclosed in the parent application. If new matter is introduced:

  • The new matter will not be entitled to the benefit of the parent application’s filing date.
  • Claims relying on the new matter may be rejected under 35 U.S.C. 112(a) for lack of written description or enablement.
  • The application may lose its status as a proper divisional, potentially affecting its ability to overcome certain types of double patenting rejections.

To avoid these issues:

  1. Carefully review the divisional application before filing to ensure it only contains subject matter from the parent application.
  2. If new subject matter is necessary, consider filing it as a continuation-in-part (CIP) application instead of a divisional.
  3. Consult with a patent attorney or agent to ensure compliance with MPEP guidelines.

For more information on Divisional application, visit: Divisional application.

For more information on new matter, visit: new matter.

For more information on patent filing, visit: patent filing.

If you submit an amendment to refer to a Continued Prosecution Application (CPA) as a continuation or divisional application, it will be denied entry by the patent examiner. The MPEP ยถ 2.34 states:

“Thus, there is no need to amend the first sentence(s) of the specification to refer back to the prior application and any such amendment shall be denied entry.”

This means that the patent examiner will not accept or incorporate such amendments into the application. The CPA itself serves as the necessary reference to the prior application, making additional amendments unnecessary and inappropriate.

For more information on continuation application, visit: continuation application.

For more information on CPA, visit: CPA.

For more information on Divisional application, visit: Divisional application.

For more information on patent examination, visit: patent examination.

What happens if a divisional application is filed after the parent application is patented or abandoned?

If a divisional application is filed after the parent application has been patented or abandoned, it will not be entitled to the benefit of the parent application’s filing date. The MPEP states:

A divisional application is often filed as a result of a restriction requirement made by the examiner. The divisional application may be filed under 37 CFR 1.53(b) (or 1.53(d) if the application is a design application) before the patenting or abandonment of or termination of proceedings on the original application or on an earlier-filed divisional application directed to the same elected invention.

Filing a divisional application after the parent application has been patented or abandoned means:

  • The application will not be considered a proper divisional application
  • It will not receive the benefit of the parent application’s filing date
  • It may be treated as a new, independent application
  • Prior art that was not applicable to the parent application may now be applicable
  • The application may face potential statutory bars under 35 U.S.C. 102

To avoid these issues, it’s crucial to file divisional applications before the patenting, abandonment, or termination of proceedings on the parent application.

For more information on Divisional application, visit: Divisional application.

For more information on late filing, visit: late filing.

For more information on patent procedure, visit: patent procedure.

If a continuation or divisional application contains new matter not disclosed in the prior-filed application, the applicant will be required to either delete the benefit claim or change the relationship to a continuation-in-part application. This is because continuation and divisional applications cannot include new matter.

As stated in MPEP ยถ 2.10.01:

“Applicant is required to delete the benefit claim or change the relationship (continuation or divisional application) to continuation-in-part because this application contains the following matter not disclosed in the prior-filed application:”

This form paragraph is used by examiners when they identify new matter in an application claiming benefit under 35 U.S.C. 120, 121, 365(c), or 386(c).

For more information on continuation application, visit: continuation application.

For more information on Divisional application, visit: Divisional application.

For more information on new matter, visit: new matter.

For more information on patent examination, visit: patent examination.

For continuation, divisional, or continuation-in-part applications filed under 37 CFR 1.53(b), the requirements for submitting an Information Disclosure Statement (IDS) depend on whether the information was previously considered in the parent application. The MPEP provides the following guidance:

  • If the information was considered in the parent application, it need not be resubmitted unless the applicant wants it printed on the patent.
  • If the information was not considered in the parent application, it must be resubmitted in compliance with 37 CFR 1.97 and 37 CFR 1.98.

The MPEP further states:

“Pursuant to 37 CFR 1.98(d), if the IDS submitted in the parent application complies with 37 CFR 1.98(a) to (c), copies of the patents, publications, pending U.S. applications, or other information submitted in the parent application need not be resubmitted in the continuing application.”

When resubmitting information, applicants should use a new listing that complies with the format requirements in 37 CFR 1.98(a)(1) and avoid submitting copies of PTO/SB/08 or PTO-892 forms from other applications to prevent confusion in the record.

To learn more:

The MPEP 201.06(c) outlines the requirements for filing a divisional or continuation application without an executed oath or declaration:

’37 CFR 1.63(d) provides that a newly executed oath or declaration is not required in a continuation or divisional application filed by all or by fewer than all of the inventors named in a prior nonprovisional application containing an oath or declaration as required by 37 CFR 1.63, provided that a copy of the executed oath or declaration filed in the prior application is submitted for the continuation or divisional application and the specification and drawings filed in the continuation or divisional application contain no matter that would have been new matter in the prior application.’

To file without a new oath or declaration:

  • Submit a copy of the executed oath or declaration from the prior application
  • Ensure the new application contains no new matter
  • File a copy of the specification and drawings from the prior application

This procedure allows for expedited filing while maintaining the integrity of the oath or declaration requirements.

To learn more:

To learn more:

To file a Continued Prosecution Application (CPA) in a design application, the following requirements must be met:

  • The application must be a design application.
  • The prior application must be a nonprovisional application.
  • The application must be filed before the earliest of:
    • Payment of the issue fee on the prior application, unless a petition under ยง 1.313(c) is granted in the prior application;
    • Abandonment of the prior application; or
    • Termination of proceedings on the prior application.

As stated in the MPEP 201.06(d): “A continuation or divisional application that is a nonprovisional application may be filed as a CPA under 37 CFR 1.53(d) if the prior nonprovisional application is a design application that is complete as defined by 37 CFR 1.51(b).”

For more information on continuation application, visit: continuation application.

For more information on CPA, visit: CPA.

For more information on Divisional application, visit: Divisional application.

For more information on filing requirements, visit: filing requirements.

For continuation or divisional applications filed before September 16, 2012 under 37 CFR 1.53(b) (excluding continuation-in-part applications), applicants could file using a copy of the oath or declaration from the prior nonprovisional application. This is supported by the MPEP, which states:

“A continuation or divisional application filed before September 16, 2012 under 37 CFR 1.53(b) (other than a continuation-in-part (CIP)) may be filed with a copy of the oath or declaration from the prior nonprovisional application.”

This provision simplified the filing process for certain types of continuing applications by allowing the use of previously submitted documents.

To learn more:

For an application to be considered a divisional, it must meet several requirements as outlined in MPEP ยถ 2.01:

  1. It must disclose and claim only subject matter disclosed in the prior application.
  2. It must claim subject matter that is independent and distinct from that claimed in the prior application.
  3. It must name the inventor or at least one joint inventor named in the prior application.

The MPEP states:

This application, which discloses and claims only subject matter disclosed in prior Application No.[1], filed [2], appears to claim only subject matter directed to an invention that is independent and distinct from that claimed in the prior application, and names the inventor or at least one joint inventor named in the prior application.

Meeting these requirements helps ensure that the divisional application is properly derived from the prior application while claiming a distinct invention.

For more information on Divisional application, visit: Divisional application.

For more information on USPTO, visit: USPTO.

A divisional application has several key characteristics as outlined in MPEP ยถ 2.01:

  1. Independent and Distinct Subject Matter: It ‘appears to claim only subject matter directed to an invention that is independent and distinct from that claimed in the prior application.’
  2. Disclosure in Prior Application: It ‘discloses and claims only subject matter disclosed in prior Application No.[1],filed[2].’
  3. Inventor Continuity: It ‘names the inventor or at least one joint inventor named in the prior application.’
  4. Potential Benefit Claim: It may claim the benefit of the filing date of the prior application, subject to meeting certain requirements.

These characteristics ensure that divisional applications properly separate distinct inventions while maintaining a connection to the original disclosure and inventorship.

For more information on Divisional application, visit: Divisional application.

The filing requirements for a divisional application under 37 CFR 1.53(b) are similar to those for a new nonprovisional application. According to the MPEP:

‘A continuation or divisional application may be filed under 35 U.S.C. 111(a) using the procedures set forth in 37 CFR 1.53(b), by providing: (1) a specification complying with 35 U.S.C. 112, including a claim or claims; (2) drawings, where necessary; and (3) the inventorship named on filing.’

Additionally, you must include:

  • The appropriate filing fees
  • An Application Data Sheet (ADS) specifying that the application is a divisional
  • A proper benefit claim to the parent application

It’s important to note that while a new oath or declaration may not be required under 37 CFR 1.63(d), you must still comply with all other filing requirements.

For more information on Divisional application, visit: Divisional application.

For more information on drawings, visit: drawings.

For more information on filing requirements, visit: filing requirements.

For more information on inventorship, visit: inventorship.

To file a CPA for a design patent application, the following requirements must be met:

  • The prior application must be a design application that is complete under 37 CFR 1.51(b)
  • The CPA must be filed before the earliest of: payment of the issue fee, abandonment, or termination of proceedings in the prior application
  • The CPA must be filed with a request on a separate paper
  • The proper filing fee, search fee, and examination fee must be paid
  • The CPA must only disclose and claim subject matter disclosed in the prior application

As stated in the MPEP: “The filing date of a CPA is the date on which a request on a separate paper for an application under this paragraph is filed.”

For more information on continuation application, visit: continuation application.

For more information on design patents, visit: design patents.

For more information on Divisional application, visit: Divisional application.

Filing a CPA for a design patent application offers several benefits:

  • Minimal filing requirements compared to a regular continuation application
  • No new filing receipt is normally issued
  • Faster processing time, as it uses the same application number and file wrapper
  • For examination priority, CPAs are treated as “amended” applications rather than “new” applications, potentially resulting in faster examination
  • The prior application is automatically abandoned, simplifying the process

As stated in the MPEP: “A CPA has a number of advantages compared to a continuation or divisional application filed under 37 CFR 1.53(b). For example, the papers required to be filed in the U.S. Patent and Trademark Office in order to secure a filing date under 37 CFR 1.53(d) are minimal compared to 37 CFR 1.53(b).”

For more information on continuation application, visit: continuation application.

For more information on design patents, visit: design patents.

For more information on Divisional application, visit: Divisional application.

When a petition under 37 CFR 1.48 to add an inventor was filed in a prior application, the continuation or divisional application could be handled in one of two ways:

  1. Filed with a copy of the executed declaration naming the correct inventive entity from the prior application.
  2. Filed with a newly executed declaration naming the correct inventive entity.

The MPEP states:

“A continuation or divisional application filed under 37 CFR 1.53(b) of a prior application in which a petition (or request) under 37 CFR 1.48 to add an inventor was filed should be filed with a copy of the executed declaration naming the correct inventive entity from the prior application or a newly executed declaration naming the correct inventive entity.”

Importantly, the MPEP also notes that “A copy of any decision under 37 CFR 1.48 from the prior application is not required to be filed in the continuation or divisional application.” This simplifies the filing process by not requiring additional documentation related to the inventorship change in the prior application.

To learn more:

To learn more:

Changes in inventorship for continuation or divisional applications filed under 37 CFR 1.53(b) could be handled in two ways:

  1. Filing with a copy of an oath or declaration from a prior application and a statement requesting the deletion of non-inventors.
  2. Filing with a newly executed oath or declaration naming the correct inventive entity.

The MPEP states:

“If an inventor named in a prior application is not an inventor in a continuation or divisional application filed under 37 CFR 1.53(b), the continuation or divisional application may either be filed (A) with a copy of an oath or declaration from a prior application and a statement requesting the deletion of the name or names of the person or persons who are not inventors of the invention being claimed in the continuation or divisional application (see pre-AIA 37 CFR 1.63(d)), or (B) with a newly executed oath or declaration naming the correct inventive entity.”

This provision allowed applicants to address changes in inventorship efficiently when filing continuation or divisional applications.

To learn more:

Applications with pre-AIA 37 CFR 1.47 status were given special consideration in continuation or divisional filings. The MPEP states:

“A continuation or divisional application of a prior application accorded status under pre-AIA 37 CFR 1.47 will be accorded status under pre-AIA 37 CFR 1.47 if a copy of the decision according pre-AIA 37 CFR 1.47 status in the prior application is filed in the continuation or divisional application, unless an oath or declaration signed by all of the inventors is included upon filing the continuation or divisional application.”

This means that the special status could be carried over to the new application by submitting a copy of the previous decision, unless a complete oath or declaration was provided.

To learn more:

To learn more:

The method for making a specific reference to a prior-filed application depends on when the application was filed:

  • For applications filed before September 16, 2012: The specific reference must be included in the first sentence(s) of the specification following the title or in an application data sheet (ADS) in compliance with pre-AIA 37 CFR 1.76.
  • For applications filed on or after September 16, 2012: The specific reference must be included in an ADS in compliance with 37 CFR 1.76.

Additionally, for benefit claims under 35 U.S.C. 120, 121, 365(c), or 386(c), the reference must include the relationship (i.e., continuation, divisional, or continuation-in-part) of the applications.

It’s important to note that for continued prosecution applications (CPAs) filed under 37 CFR 1.53(d) (design applications under 35 U.S.C. chapter 16 only), a specific reference in the specification or ADS is not required and may not be made. For these applications, The specific reference requirement of 35 U.S.C. 120 is met by the transmittal request for the CPA which is considered to be part of the CPA.

For more information on application data sheet, visit: application data sheet.

For more information on continuation application, visit: continuation application.

For more information on Divisional application, visit: Divisional application.

For more information on specific reference, visit: specific reference.

Special attention is required when dealing with the correspondence address in continuation or divisional applications. According to MPEP 601.03(b):

“Special care should be taken in continuation or divisional applications to ensure that any change of correspondence address in a prior application is reflected in the continuation or divisional application.”

The MPEP further explains that when a copy of the oath or declaration from the prior application is submitted for a continuation or divisional application filed under pre-AIA 37 CFR 1.53(b), and this copy designates an old correspondence address, the USPTO may not recognize the change of correspondence address made during the prosecution of the prior application.

To address this, the MPEP advises: “Applicant is required to identify the change of correspondence address in the continuation or divisional application to ensure that communications from the Office are mailed to the current correspondence address.”

To learn more:

To learn more:

How does the USPTO assign application numbers to continuation or divisional applications?

The USPTO assigns application numbers to continuation or divisional applications in the same manner as new applications. According to MPEP 503:

A continuation or divisional application (including a continued prosecution application) filed under 37 CFR 1.53(b) is assigned a new application number.

This means:

  • Each continuation or divisional application receives a unique number
  • The number is assigned in the same sequence as other new applications
  • The application number does not indicate its relationship to the parent application

It’s important to reference the parent application in the appropriate sections of the new application to establish the continuity.

For more information on continuation application, visit: continuation application.

For more information on Divisional application, visit: Divisional application.

For more information on USPTO procedures, visit: USPTO procedures.

The presence of new matter in a continuation or divisional application can significantly affect its filing date. If new matter is found, the application may lose the benefit of the earlier filing date for the new subject matter.

According to MPEP ยถ 2.10.01:

“Applicant is required to delete the benefit claim or change the relationship (continuation or divisional application) to continuation-in-part because this application contains the following matter not disclosed in the prior-filed application:”

If the application is changed to a continuation-in-part, the new matter will have the filing date of the current application, while the previously disclosed matter retains the earlier filing date. This can have important implications for determining prior art and patent term.

For more information on continuation application, visit: continuation application.

For more information on continuation-in-part application, visit: continuation-in-part application.

For more information on Divisional application, visit: Divisional application.

For more information on new matter, visit: new matter.

How does the oath or declaration work in divisional applications?

In divisional applications, the oath or declaration process is simplified to reduce the burden on applicants. According to MPEP 201.06(c):

37 CFR 1.63(d) provides that a newly executed oath or declaration is not required in a divisional application filed under 37 CFR 1.53(b) that contains the subject matter described in the prior nonprovisional application, provided a copy of the executed oath or declaration filed in the prior nonprovisional application is submitted.

Key points about the oath or declaration in divisional applications:

  • A new oath or declaration is not required if the divisional application contains the same subject matter as the parent application.
  • A copy of the executed oath or declaration from the parent application can be submitted.
  • The copy must show the USPTO seal or stamp indicating it was received.
  • If the inventorship changes, a new oath or declaration may be required.

This provision under 37 CFR 1.63(d) streamlines the filing process for divisional applications while ensuring proper inventorship documentation.

For more information on declaration, visit: declaration.

For more information on Divisional application, visit: Divisional application.

For more information on oath, visit: oath.

How does the oath or declaration requirement differ for divisional applications?

For divisional applications, the oath or declaration requirement can be simplified under certain conditions. According to MPEP 201.06(c):

“The divisional application may be filed under 37 CFR 1.53(b) using all or part of a copy of the oath or declaration filed in the prior nonprovisional application under 37 CFR 1.63(d), as long as the oath or declaration is still applicable to the subject matter of the divisional application.”

This means that if the original oath or declaration from the parent application still covers the subject matter in the divisional application, it can be reused. However, if new matter is introduced or the inventors change, a new oath or declaration may be required.

For more information on Divisional application, visit: Divisional application.

For more information on oath or declaration, visit: oath or declaration.

For more information on patent filing, visit: patent filing.

Micro entity status does not automatically carry over to continuation or divisional applications. A new certification is required for each new application:

  1. A new assertion of entitlement to micro entity status is required for any continuation, divisional, or continuation-in-part application.
  2. This applies even if micro entity status was properly claimed in the parent application.
  3. The new certification must be filed in the continuing application.

The MPEP states: The refiling of an application under 37 CFR 1.53 as a continuation, divisional, or continuation-in-part application (including a continued prosecution application under 37 CFR 1.53(d) (design applications only)), requires a new certification of entitlement to micro entity status in the continuing application.

It’s important to note that this requirement for a new certification applies specifically to micro entity status. For small entity status, a new assertion is also required, but the rules are slightly different.

For more information on continuation application, visit: continuation application.

For more information on continuation-in-part application, visit: continuation-in-part application.

For more information on Divisional application, visit: Divisional application.

The inventorship of a divisional application must include at least one inventor named in the parent application, but it doesn’t have to be identical. The MPEP states:

The inventorship in the divisional application must include at least one inventor named in the prior-filed application, and the divisional application must claim the benefit of the prior-filed application under 35 U.S.C. 120, 121, 365(c), or 386(c).

If the divisional application is filed with fewer inventors than the parent application, a statement may be filed requesting deletion of the names of the additional inventors. For applications filed on or after September 16, 2012, the statement must be filed with the divisional application and signed by a party set forth in 37 CFR 1.33(b).

To learn more:

The filing date of a divisional application can affect the patent term in several ways:

  1. 20-year term: The 20-year patent term is calculated from the earliest U.S. filing date to which the divisional application claims priority. This means that the term of a patent issued on a divisional application will generally be shorter than that of its parent application.
  2. Patent Term Adjustment (PTA): The divisional application may be eligible for PTA based on delays in its own prosecution, but not for delays in the prosecution of its parent application.
  3. Terminal Disclaimer: If there’s a risk of obviousness-type double patenting, a terminal disclaimer may be required, potentially limiting the term of the divisional patent to that of the parent patent.

The MPEP 201.06(c) does not directly address patent term, but it’s important to consider these factors when filing a divisional application. For more details on patent term, refer to MPEP 2701.

For more information on Divisional application, visit: Divisional application.

For more information on patent term, visit: patent term.

For more information on patent term adjustment, visit: patent term adjustment.

The disclosure requirements for claiming the benefit of provisional and nonprovisional applications differ in some aspects:

Provisional Applications (35 U.S.C. 119(e)):

For a nonprovisional application to claim the benefit of a provisional application:

  • The written description and drawings (if any) of the provisional application must adequately support and enable the subject matter claimed in the nonprovisional application.
  • A claim is not required in the provisional application.

Nonprovisional Applications (35 U.S.C. 120, 121, 365(c), or 386(c)):

For continuation and divisional applications:

  • The disclosure must be the same as the prior-filed application.
  • No new matter can be added.

For continuation-in-part applications:

  • New matter may be included.
  • Only claims supported by the prior-filed application are entitled to the earlier filing date.

As stated in MPEP 211.05:

“Under 35 U.S.C. 120, a claim in a U.S. application is entitled to the benefit of the filing date of an earlier filed U.S. application if the subject matter of the claim is disclosed in the manner provided by 35 U.S.C. 112(a) except for the best mode requirement, in the earlier filed application.”

This requirement applies to both provisional and nonprovisional applications, ensuring that the earlier application provides proper support for the claimed invention.

To learn more:

Restriction requirements can affect joint inventorship in patent applications. The MPEP explains:

If an application by joint inventors includes more than one independent and distinct invention, restriction may be required with the possible result of a necessity to change the inventorship named in the application if the elected invention was not the invention of all the originally named inventors.

In such cases:

  • A ‘divisional’ application complying with 35 U.S.C. 120 would be entitled to the benefit of the earlier filing date of the original application.
  • Examiners should remind applicants to correct inventorship under 37 CFR 1.48 if an invention is elected and claims to the invention of one or more inventors are canceled.

This process ensures that the correct inventors are named for each distinct invention in the case of a restriction requirement.

To learn more:

To learn more:

The treatment of new matter varies depending on the type of continuing application:

  • Continuation and Divisional Applications: Must not contain any new matter. The disclosure must be the same as the prior application.
  • Continuation-in-Part (CIP) Applications: Can include new matter not disclosed in the parent application.
  • Continued Prosecution Applications (CPAs): Cannot contain new matter.

For continuation and divisional applications, the MPEP states: “The disclosure presented in the continuation must not include any subject matter which would constitute new matter if submitted as an amendment to the parent application.”

For CIPs, the MPEP notes: “A continuation-in-part is an application… repeating some substantial portion or all of the prior-filed application and adding matter not disclosed in the prior-filed application.”

If new matter is found in a continuation or divisional application, the examiner will require the applicant to delete the benefit claim or change the application to a continuation-in-part. For CPAs, any new matter introduced will be required to be canceled.

It’s important to note that claims in a CIP application that rely on new matter will only be entitled to the filing date of the CIP, not the parent application’s filing date.

For more information on continuation application, visit: continuation application.

For more information on continuation-in-part application, visit: continuation-in-part application.

For more information on CPA, visit: CPA.

For more information on Divisional application, visit: Divisional application.

For more information on new matter, visit: new matter.

Filing a continuation or divisional application under 37 CFR 1.53(b) does not automatically abandon the prior application. The status of the prior application depends on the actions taken by the applicant:

  1. The prior application remains pending unless expressly abandoned.
  2. To expressly abandon the prior application, a separate paper must be filed and signed in accordance with 37 CFR 1.138.
  3. If a notice of allowance has been issued in the prior application, it can become abandoned by nonpayment of the issue fee.

The MPEP states: Under 37 CFR 1.53(b) practice, the prior nonprovisional application is not automatically abandoned upon filing of the continuing application. If the prior nonprovisional application is to be expressly abandoned, such a paper must be signed in accordance with 37 CFR 1.138.

It’s important to note that if the issue fee has been paid in the prior application, a petition to withdraw the application from issue must be filed before it can be abandoned, even if a continuing application has been filed.

For more information on continuation application, visit: continuation application.

For more information on Divisional application, visit: Divisional application.

Patent examiners carefully compare the content of the continuation or divisional application with the prior-filed application to identify any new matter. If they find information not disclosed in the prior application, they will issue a notice to the applicant.

The MPEP ยถ 2.10.01 instructs examiners:

“In bracket 1, provide an example of the matter not disclosed in the prior-filed application.”

This means the examiner must specifically point out the new matter found in the application, allowing the applicant to understand and address the issue.

For more information on continuation application, visit: continuation application.

For more information on Divisional application, visit: Divisional application.

For more information on new matter, visit: new matter.

For more information on patent examination, visit: patent examination.

Patent examiners use specific criteria to determine if an application may qualify as a divisional. According to MPEP ยถ 2.01, an examiner looks for the following indicators:

  1. The application claims only subject matter disclosed in a prior application.
  2. The claimed subject matter appears to be independent and distinct from that claimed in the prior application.
  3. At least one inventor from the prior application is named in the current application.

The MPEP provides guidance to examiners:

“This form paragraph should only be used if it appears that the application may be a divisional, but a benefit claim has not been properly established.”

If these conditions are met, but a proper benefit claim hasn’t been made, the examiner may use this paragraph to notify the applicant of the possible divisional status and the need to establish a benefit claim if desired.

For more information on Divisional application, visit: Divisional application.

For more information on patent examination, visit: patent examination.

For more information on USPTO, visit: USPTO.

To claim the benefit of a prior application’s filing date for a divisional application, applicants must follow specific procedures outlined in patent law and regulations. The MPEP ยถ 2.01 states:

Should applicant desire to claim the benefit of the filing date of the prior application, attention is directed to 35 U.S.C. 120, 37 CFR 1.78, and MPEP ยง 211 et seq.

This means applicants should:

  • Ensure compliance with 35 U.S.C. 120 requirements for continuity and disclosure
  • Follow the procedures outlined in 37 CFR 1.78 for claiming benefit
  • Consult MPEP ยง 211 for detailed guidance on benefit claims

It’s crucial to properly establish the benefit claim to secure the earlier filing date for the divisional application.

For more information on Divisional application, visit: Divisional application.

A terminal disclaimer filed in a parent application can affect a divisional application in specific ways. According to MPEP 201.06:

‘The divisional application must be directed to subject matter described and claimed in the prior nonprovisional application.’

This means that if a terminal disclaimer was filed in the parent application, it may impact the divisional application as follows:

  • Inheritance of Terminal Disclaimer: Generally, a terminal disclaimer filed in the parent application does not automatically apply to the divisional application.
  • Potential Double Patenting: If the claims in the divisional application are not patentably distinct from those in the parent application, a new terminal disclaimer may be required in the divisional to overcome a double patenting rejection.
  • Examination Considerations: Examiners will consider the relationship between the parent and divisional applications when determining if a terminal disclaimer is necessary.

It’s important to consult with a patent attorney or agent to understand the specific implications of terminal disclaimers on your divisional application, as each case can have unique considerations.

To learn more:

How does a restriction requirement lead to a divisional application?

A restriction requirement often leads to the filing of a divisional application. This process occurs as follows:

  1. An examiner issues a restriction requirement when they believe the original application contains claims to two or more independent or distinct inventions.
  2. The applicant must then elect one invention to pursue in the original application.
  3. The non-elected inventions can be pursued in one or more divisional applications.

As stated in MPEP 201.06:

“The inventions embodied in the claims of a divisional application are generally further characterized as (1) independent inventions or (2) related inventions which are distinct.”

This means that the divisional application allows the applicant to protect inventions that were disclosed but not pursued in the original application due to the restriction requirement.

For more information on Divisional application, visit: Divisional application.

For more information on patent examination, visit: patent examination.

How does a divisional application differ from other continuing applications?

A divisional application is a unique type of continuing application that focuses on a distinct invention separated from a parent application. According to the MPEP:

A divisional application is one filed for an invention which has been disclosed and claimed, but not allowed, in a prior nonprovisional application or international application designating the United States. It is a later application for an independent and distinct invention carved out of a pending application. (MPEP 201.02)

Key differences of a divisional application include:

  • It is filed in response to a restriction requirement in the parent application
  • It claims only subject matter that was disclosed but not claimed in the parent application
  • It does not add new matter
  • It allows the applicant to pursue protection for inventions that were deemed distinct in the original application

Unlike continuations or CIPs, divisional applications are specifically for pursuing separate inventions identified during the examination of the parent application.

To learn more:

Divisional applications and provisional applications are distinct types of patent applications with different purposes and characteristics:

  • Divisional Application: Claims subject matter from a prior non-provisional application that is independent and distinct from the original claims.
  • Provisional Application: A temporary application that establishes a priority date but does not mature into an issued patent.

The MPEP explicitly states in MPEP ยถ 2.01:

“An application claiming the benefit of a provisional application under 35 U.S.C. 119(e) should not be called a ‘divisional’ of the prior application.”

This distinction is important because divisional applications claim the benefit under 35 U.S.C. 120, while provisional applications are claimed under 35 U.S.C. 119(e).

For more information on Divisional application, visit: Divisional application.

For more information on patent law, visit: patent law.

For more information on provisional application, visit: provisional application.

For more information on USPTO, visit: USPTO.

A divisional application discloses and claims only subject matter disclosed in the parent application, while a continuation-in-part (CIP) application may include new matter. The MPEP states:

A continuation-in-part application should not be designated as a divisional application. The Court of Appeals for the Federal Circuit has concluded that the protection of the third sentence of 35 U.S.C. 121 does not extend to continuation-in-part applications, stating that ‘the protection afforded by section 121 to applications (or patents issued therefrom) filed as a result of a restriction requirement is limited to divisional applications.’

This means that divisional applications have certain protections against double patenting rejections that CIPs do not have.

To learn more:

A divisional application does not extend the patent term beyond that of the parent application. The term of a patent issuing from a divisional application is calculated from the filing date of the earliest application for which a benefit is claimed under 35 U.S.C. 120, 121, 365(c), or 386(c), excluding any benefit claims to provisional applications.

This means that while you can pursue additional patent protection through a divisional application, you don’t get extra time beyond the original 20-year term from the earliest non-provisional filing date.

It’s important to note that patent term adjustments may apply independently to divisional applications based on delays in their own prosecution.

To learn more:

A divisional application’s patent term adjustment (PTA) is calculated independently from its parent application. The MPEP 201.06 provides guidance on this matter:

‘A divisional application will receive its own patent term adjustment under 35 U.S.C. 154(b), which will be calculated based on the prosecution of the divisional application itself.’

Key points to understand:

  • The divisional application’s PTA is based solely on its own prosecution history
  • Any delays or adjustments from the parent application do not affect the divisional’s PTA
  • The PTA calculation starts from the filing date of the divisional application
  • Applicants should monitor the prosecution of the divisional application to ensure accurate PTA calculation

For more information on patent term adjustment, refer to MPEP 2730.

To learn more:

To learn more:

37 CFR 1.63(d) provides a simplified process for submitting an oath or declaration in a divisional application. The MPEP states:

’37 CFR 1.63(d) provides that a newly executed oath or declaration is not required in a continuation or divisional application filed by all or by fewer than all of the inventors named in a prior nonprovisional application containing a signed oath or declaration.’

This means that if the divisional application is filed by the same inventor(s) as the parent application, and there’s no new matter added, the oath or declaration from the parent application can be used for the divisional application. This simplifies the filing process and reduces paperwork.

For more information on continuation application, visit: continuation application.

For more information on Divisional application, visit: Divisional application.

For more information on oath or declaration, visit: oath or declaration.

Continuation, divisional, and continuation-in-part (CIP) applications are all types of continuing applications, but they have key differences:

  • Continuation application: Discloses and claims only subject matter disclosed in the prior application. No new matter is added.
  • Divisional application: Results from a restriction requirement in the parent application. Claims an independent and distinct invention carved out of the parent application. No new matter is added.
  • Continuation-in-part (CIP) application: Repeats a substantial portion of the prior application and adds matter not disclosed in the prior application (i.e., new matter is added).

As stated in the MPEP regarding continuation-in-part applications:

A continuation-in-part is an application filed during the lifetime of a prior-filed nonprovisional application, international application designating the United States, or international design application designating the United States repeating some substantial portion or all of the prior-filed application and adding matter not disclosed in the prior-filed application.

All three types of applications must claim the benefit of the prior application under 35 U.S.C. 120, 121, 365(c), or 386(c) and comply with other requirements set forth in 37 CFR 1.78.

For more information on continuation application, visit: continuation application.

For more information on continuation-in-part application, visit: continuation-in-part application.

For more information on Divisional application, visit: Divisional application.

For more information on new matter, visit: new matter.

For more information on patent application types, visit: patent application types.

Applicants can avoid the need for preliminary amendments in patent applications by following the guidance provided in MPEP 608.04(b):

Applicants can avoid the need to file a preliminary amendment by incorporating any desired amendments into the text of the specification, even where the application is a continuation or divisional application of a prior-filed application.

This approach involves directly including any necessary changes or additions in the original specification text when preparing the application. By doing so, applicants can ensure that all desired content is present from the initial filing, eliminating the need for separate preliminary amendments. This method is recommended for all types of applications, including continuations and divisionals, to streamline the application process and reduce potential processing delays.

To learn more:

For applications filed on or after September 16, 2012, an applicant can file a continuation or divisional application using a copy of an oath or declaration from a prior application if:

  • The oath or declaration complies with 35 U.S.C. 115 as revised effective September 16, 2012
  • It was executed by or with respect to the inventor in the earlier-filed application
  • A copy of the oath, declaration, or substitute statement showing the signature or an indication it was executed is submitted in the continuing application

As stated in the MPEP: For applications filed on or after September 16, 2012, the filing of a continuing application by all or by fewer than all of the inventors named in a prior application without a newly executed oath or declaration is permitted provided that an oath or declaration in compliance with 37 CFR 1.63, or a substitute statement under 37 CFR 1.64, was executed by or with respect to such inventor and was filed in the earlier-filed application, and a copy of such oath, declaration, or substitute statement showing the signature or an indication thereon that it was executed, is submitted in the continuing application.

It’s important to note that the oath or declaration must meet the current requirements, even if the prior application was filed before September 16, 2012.

For more information on continuation application, visit: continuation application.

For more information on declaration, visit: declaration.

For more information on Divisional application, visit: Divisional application.

For more information on oath, visit: oath.

For continuation or divisional applications filed under 37 CFR 1.53(b) of an application previously accorded status under pre-AIA 37 CFR 1.47, the process is simplified. The MPEP states: In a continuation or divisional application filed under 37 CFR 1.53(b) of an application accorded status under pre-AIA 37 CFR 1.47, if a copy of a declaration from a prior application and a copy of a decision according status under pre-AIA 37 CFR 1.47 are filed as permitted by pre-AIA 37 CFR 1.63(d)(3)(i), the notice will not be repeated.

This means that if the proper documentation from the original application is provided, the USPTO won’t repeat the notification process for nonsigning inventors. This streamlines the application process for related applications.

To learn more:

To learn more:

The treatment of continuation or divisional applications filed under former 37 CFR 1.62 depends on the filing date of the original application:

  • For applications filed on or after June 8, 1995: Requests filed on or after December 1, 1997, are treated as requests for continued examination (RCE) under 37 CFR 1.114.
  • For applications filed before June 8, 1995: Requests filed on or after December 1, 1997, are treated as improper applications.

As stated in the MPEP: A request for a continuation or divisional application filed under former 37 CFR 1.62 on or after December 1, 1997, in an application that was filed on or after June 8, 1995, will be treated as a request for continued examination (RCE) under 37 CFR 1.114, see MPEP 706.07(h), paragraph IV.

To learn more:

The treatment of continuation or divisional applications filed under former 37 CFR 1.62 depends on the filing date of the original application:

  • For applications filed on or after June 8, 1995: “A request for a continuation or divisional application filed under former 37 CFR 1.62 on or after December 1, 1997, in an application that was filed on or after June 8, 1995, will be treated as a request for continued examination (RCE) under 37 CFR 1.114.” This means these applications are now processed under the RCE procedure.
  • For applications filed before June 8, 1995: Requests for continuation or divisional applications filed on or after December 1, 1997, will be treated as improper applications.

For more details on RCE procedures, refer to MPEP 706.07(h), paragraph IV.

For more information on continuation application, visit: continuation application.

For more information on Divisional application, visit: Divisional application.

For more information on RCE, visit: RCE.

For applications filed on or after June 8, 1995, continuation or divisional applications filed under former 37 CFR 1.62 on or after December 1, 1997, are treated differently. The MPEP states: “A request for a continuation or divisional application filed under former 37 CFR 1.62 on or after December 1, 1997, in an application that was filed on or after June 8, 1995, will be treated as a request for continued examination (RCE) under 37 CFR 1.114.” This means such applications are now processed under the RCE procedure rather than the former File Wrapper Continuing Procedure.

For more information on continuation application, visit: continuation application.

For more information on Divisional application, visit: Divisional application.

For more information on RCE, visit: RCE.

According to the MPEP, A continuation or divisional application filed under 37 CFR 1.60 on or after December 1, 1997, will automatically be treated as an application filed under 37 CFR 1.53(b). This means that any continuation or divisional application filed after this date is processed under the new regulations, specifically 37 CFR 1.53(b), regardless of whether the applicant intended to file under the old 37 CFR 1.60 procedure.

To learn more:

To learn more:

Can the specification be omitted when filing a divisional or continuation application?

No, the specification cannot be omitted when filing a divisional or continuation application. However, the process is simplified. According to MPEP 201.06(c):

“A new specification (including the claims) may be submitted in a divisional application filed under 37 CFR 1.53(b). In such a case, the applicant must file a copy of the prior application as filed or a new specification.”

This means that while you must include a specification, you have two options:

  • Submit a copy of the prior application’s specification
  • Submit a new specification tailored to the divisional or continuation application

If you choose to submit a copy of the prior application, it’s important to ensure that it properly supports the claims in the new application. Any new matter introduced in a continuation application may result in a rejection under 35 U.S.C. 112(a).

Remember, the claims are considered part of the specification, so you must include claims with your application, whether you’re using the prior specification or submitting a new one.

For more information on continuation application, visit: continuation application.

For more information on Divisional application, visit: Divisional application.

For more information on patent application requirements, visit: patent application requirements.

For more information on USPTO, visit: USPTO.

No, new matter cannot be added to a divisional application. The divisional application must only contain subject matter disclosed in the parent application. The MPEP clearly states:

Thus the disclosure presented in a divisional application must not include any subject matter which would constitute new matter if submitted as an amendment to the parent application.

Any attempt to add new matter to a divisional application will be rejected under 35 U.S.C. 112(a) as lacking written description support in the parent application.

To learn more:

Can I file a priority claim in a continuing application?

Yes, you can file a priority claim in a continuing application, but there are specific requirements and deadlines to consider. According to MPEP 214.01, “In the case of a continuation or divisional application filed under 35 U.S.C. 111(a), the claim for priority must be made during the pendency of the application and within the later of four months from the actual filing date of the application or sixteen months from the filing date of the prior foreign application.” This means that even in a continuing application, you must adhere to the time limits for filing the priority claim. It’s important to note that the priority claim must be made in each continuing application; a priority claim in a parent application does not automatically carry over to child applications.

Yes, you can file a priority claim in a continuation or divisional application, but there are specific rules you must follow. According to MPEP 214.01:

‘In the case of a continuation or divisional application filed under 35 U.S.C. 111(a), the claim for priority must be made during the pendency of the application and within the four-month period set forth in 37 CFR 1.55(d)(1).’

This means:

  • The priority claim must be made while the continuation or divisional application is still pending (not abandoned or issued).
  • You have a four-month window from the actual filing date of the continuation or divisional application to make the priority claim.
  • The priority claim should refer back to the foreign application through the parent application(s).

It’s important to note that the continuation or divisional application must have copendency with the parent application to maintain the priority chain. Always consult the latest version of the MPEP and consider seeking advice from a patent attorney to ensure compliance with current USPTO rules.

To learn more:

Can I file a divisional application without a new oath or declaration?

Yes, you can file a divisional application without a new oath or declaration under certain conditions. According to MPEP 201.06(c):

“Under 37 CFR 1.63(d), a newly executed oath or declaration is not required in a divisional application filed under 37 CFR 1.53(b) if: (1) the prior nonprovisional application contained an oath or declaration as prescribed by 37 CFR 1.63 that was executed by or with respect to the inventor and was signed in compliance with 37 CFR 1.64; and (2) the divisional application was filed by all or by fewer than all of the inventors named in the prior application.”

This provision simplifies the filing process for divisional applications, allowing inventors to use the oath or declaration from the original application, provided the specified conditions are met.

For more information on declaration, visit: declaration.

For more information on Divisional application, visit: Divisional application.

For more information on oath, visit: oath.

No, you cannot amend the specification of a Continued Prosecution Application (CPA) to refer to it as a continuation or divisional application. According to MPEP ยง 201.06(d):

“The amendment filed [date] requesting that the specification be amended to refer to the present Continued Prosecution Application (CPA) as a [continuation/divisional] application of Application No. [X] has not been entered. As set forth in 37 CFR 1.53(d)(7), a request for a CPA is the specific reference required by 35 U.S.C. 120 to every application assigned the application number identified in such request. Thus, there is no need to amend the first sentence(s) of the specification to refer back to the prior application and any such amendment shall be denied entry.”

This means that the CPA request itself serves as the required reference to the prior application, and no additional amendment to the specification is necessary or allowed.

For more information on continuation application, visit: continuation application.

For more information on CPA, visit: CPA.

For more information on Divisional application, visit: Divisional application.

Can an inventor’s oath or declaration from a parent application be used in a divisional application?

Yes, an inventor’s oath or declaration from a parent application can often be used in a divisional application. According to MPEP 201.06(c):

‘In accordance with 37 CFR 1.63(d), a newly executed oath or declaration is not required in a divisional application filed under 37 CFR 1.53(b) if: (1) the divisional application claims are directed to subject matter disclosed in the prior-filed application; (2) a copy of the inventor’s oath or declaration from the prior-filed application is provided for the divisional application; and (3) the prior-filed application contains a statement by the applicant that any required oath or declaration, including any substitute statement under 37 CFR 1.64 or assignment-statement under pre-AIA 37 CFR 1.63(e), has been filed.’

This provision simplifies the filing process for divisional applications by allowing the reuse of the original oath or declaration, provided certain conditions are met. However, it’s important to ensure that all inventors named in the divisional application are covered by the oath or declaration from the parent application.

For more information on declaration, visit: declaration.

For more information on Divisional application, visit: Divisional application.

For more information on inventor’s oath, visit: inventor’s oath.

For more information on parent application, visit: parent application.

Yes, according to MPEP ยถ 2.05, an application can potentially be classified as both a continuation and a division. The MPEP states:

“Accordingly, this application may constitute a continuation or division.” (MPEP ยถ 2.05)

This dual classification possibility arises because:

  • A continuation application discloses and claims only subject matter from the prior application
  • A divisional application is filed as a result of a restriction requirement in the prior application
  • In some cases, an application may meet both criteria

The final determination of whether the application is a continuation, division, or both will depend on the specific circumstances of the application and its relationship to the prior application. The examiner will make this determination during the examination process.

For more information on continuation application, visit: continuation application.

For more information on Divisional application, visit: Divisional application.

No, a provisional application cannot be called a ‘divisional’ application. The MPEP ยถ 2.01 explicitly states:

An application claiming the benefit of a provisional application under 35 U.S.C. 119(e) should not be called a ‘divisional’ of the prior application.

This means that while an application can claim the benefit of a provisional application’s filing date under 35 U.S.C. 119(e), it is not considered a divisional application. Divisional applications are specifically related to non-provisional applications and involve claiming independent and distinct inventions from a parent application.

For more information on 35 USC 119(e), visit: 35 USC 119(e).

For more information on Divisional application, visit: Divisional application.

For more information on provisional application, visit: provisional application.

Can a priority claim be made in a continuation or divisional application?

Yes, a priority claim can be made in a continuation or divisional application. However, there are specific requirements and time limits to consider. According to MPEP 214.01:

“In the case of a continuation or divisional application, the claim for priority must be made during the pendency of the application.”

This means:

  • The priority claim must be filed within the later of four months from the actual filing date of the continuation or divisional application or sixteen months from the filing date of the prior foreign application.
  • If the claim is filed outside this period, it will be considered untimely and require a petition under 37 CFR 1.55(e).
  • The priority claim should reference the parent application if it properly claimed the benefit of the foreign application.

It’s important to note that the continuation or divisional application must be filed within 12 months of the foreign application’s filing date (or 6 months for design applications) to be eligible for priority.

Yes, a divisional or continuation application can claim benefit from multiple prior applications. The MPEP 201.06(c) states:

‘A continuation or divisional application may be filed under 35 U.S.C. 111(a) using the procedures set forth in 37 CFR 1.53(b), by providing: (A) a copy of the prior application as filed; and (B) a new specification, claim or claims and drawings, where applicable.’

This provision allows for claiming benefit from multiple prior applications by following the correct procedures. It’s crucial to ensure that the chain of priority is properly maintained and that all necessary documents and declarations are filed.

To learn more:

To learn more:

No, a divisional application cannot include new matter that was not disclosed in the parent application. The MPEP 201.06 clearly states:

‘A divisional application is a later application for an independent or distinct invention, carved out of a prior application and disclosing and claiming only subject matter disclosed in the prior application.’

This means:

  • The divisional application must be limited to the subject matter disclosed in the parent application
  • No new matter can be added to the divisional application
  • The claims in the divisional must be supported by the disclosure of the parent application

If an applicant wishes to add new matter, they should consider filing a continuation-in-part (CIP) application instead of a divisional. For more information on new matter, refer to MPEP 2163.

To learn more:

Yes, a divisional application can claim the benefit of the filing date of the prior application. The MPEP ยถ 2.01 states:

Should applicant desire to claim the benefit of the filing date of the prior application, attention is directed to 35 U.S.C. 120, 37 CFR 1.78, and MPEP ยง 211 et seq.

To claim this benefit, the applicant must meet the requirements set forth in 35 U.S.C. 120 and 37 CFR 1.78, which include making a proper benefit claim and ensuring the divisional application is filed before the patenting or abandonment of the prior application.

For more information on Divisional application, visit: Divisional application.

For more information on patent law, visit: patent law.

Yes, a divisional application can claim priority to its parent application. This is one of the key benefits of filing a divisional application. According to MPEP 201.06:

‘A divisional application is entitled to the benefit of the filing date of the prior application.’

This means that the divisional application:

  • Retains the priority date of the parent application for the subject matter disclosed in the parent
  • Can claim the benefit under 35 U.S.C. 120 or 365(c)
  • Must include a specific reference to the parent application in its application data sheet (ADS)

It’s important to note that the divisional application must be filed while the parent application is still pending to claim this benefit.

For more information on 35 U.S.C. 120, visit: 35 U.S.C. 120.

For more information on Divisional application, visit: Divisional application.

For more information on patent law, visit: patent law.

Yes, a divisional application can claim priority to a provisional application under certain conditions. According to MPEP 201.06:

‘A divisional application may claim the benefit of a provisional application under 35 U.S.C. 119(e) if the divisional application is filed within twelve months of the provisional application filing date (or an appropriate later date in accordance with 37 CFR 1.78).’

To claim this benefit, the divisional application must:

  • Be filed within 12 months of the provisional application’s filing date (or later if permitted under 37 CFR 1.78)
  • Include a proper reference to the provisional application
  • Have at least one common inventor with the provisional application

To learn more:

To learn more:

Yes, a divisional application can claim the benefit of its parent application’s filing date. According to MPEP 201.06(c):

“A divisional application is entitled to the benefit of the filing date of the prior-filed application if the prior-filed application discloses the invention claimed in the divisional application in the manner provided by the first paragraph of 35 U.S.C. 112.”

This means that as long as the invention claimed in the divisional application was adequately described in the parent application, it can claim the parent’s filing date. This is crucial for establishing priority and avoiding potential prior art issues.

For more information on Divisional application, visit: Divisional application.

Can a divisional application claim benefit from a provisional application?

Yes, a divisional application can claim benefit from a provisional application through its parent application. The MPEP 201.06 states: “A later-filed application may be filed as a continuation, divisional, or continuation-in-part of a prior nonprovisional application or international application designating the United States. Unless the filing date of the earlier nonprovisional application is to be claimed for prior art purposes, there is no need for the later-filed application to be copending with it.” This means that if the parent application claimed benefit from a provisional application, the divisional application can also claim that benefit, extending its effective filing date back to the provisional application’s filing date.

To learn more:

Can a divisional application be filed without a restriction requirement?

While divisional applications are typically filed in response to a restriction requirement, they can be filed voluntarily without one. The MPEP 201.06 states:

“A divisional application is often filed as a result of a restriction requirement made by the examiner.”

However, the use of “often” implies that this is not always the case. Applicants may choose to file a divisional application voluntarily if they:

  • Recognize distinct inventions in their application
  • Want to pursue different claim scopes separately
  • Need to address potential unity of invention issues proactively

It’s important to note that voluntarily filing a divisional application without a restriction requirement may affect the application of the safe harbor provision under 35 U.S.C. 121, which protects against double patenting rejections in certain cases.

For more information on Divisional application, visit: Divisional application.

Can a divisional application be filed after the parent application is abandoned?

Yes, a divisional application can be filed after the parent application is abandoned, as long as certain conditions are met. The MPEP 201.06 states:

“A divisional application may be filed after abandonment of the parent application, provided the parent application is not relied upon for benefit claims under 35 U.S.C. 120, 121, 365(c), or 386(c) in an application whose effective filing date is on or after March 16, 2013.”

This means that you can file a divisional application even if the parent application has been abandoned. However, it’s crucial to note that if you want to claim the benefit of the parent application’s filing date, and the effective filing date of your divisional application is on or after March 16, 2013, the parent application must not have been abandoned at the time of filing the divisional.

It’s important to consult with a patent attorney to ensure compliance with all requirements when filing a divisional application, especially in cases involving abandoned parent applications.

For more information on Divisional application, visit: Divisional application.

For more information on effective filing date, visit: effective filing date.

No, a copy of the original oath or declaration cannot be used in a continuation or divisional application filed under 37 CFR 1.53(b). A new oath or declaration must be filed.

According to MPEP 602.02: ‘A new oath or declaration in compliance with 37 CFR 1.63 or 1.67 is required in a continuation or divisional application filed under 37 CFR 1.53(b), regardless of whether a copy of the oath or declaration from the prior application is submitted.’

This requirement ensures that the inventors reaffirm their commitment to the claims in the new application, which may differ from those in the parent application.

To learn more:

Yes, certain applications are still processed under former 37 CFR 1.62. Specifically, all continuation, divisional, and continuation-in-part (CIP) applications filed under former 37 CFR 1.62 prior to December 1, 1997, continue to be processed and examined under the procedures set forth in that regulation.

The MPEP clarifies this point: All continuation, divisional and CIP applications filed under former 37 CFR 1.62 prior to December 1, 1997, will continue to be processed and examined under the procedures set forth in former 37 CFR 1.62.

For more detailed information on the practice and procedure under former 37 CFR 1.62, the MPEP directs readers to consult MPEP ยง 201.06(b) in the MPEP 8th Edition, Rev. 1 (February 2003), which is available on the USPTO website.

To learn more:

To learn more:

For continuation or divisional applications filed before September 16, 2012, a new oath or declaration is generally not required if certain conditions are met. According to pre-AIA 37 CFR 1.63(d):

A newly executed oath or declaration is not required under ยง 1.51(b)(2) and ยง 1.53(f) in a continuation or divisional application, provided that:

  • The prior nonprovisional application contained a proper oath or declaration
  • The continuation or divisional application was filed by all or fewer than all inventors named in the prior application
  • The specification and drawings contain no new matter
  • A copy of the executed oath or declaration from the prior application is submitted

However, a new oath or declaration is required if naming an inventor not named in the prior application. The MPEP states: A newly executed oath or declaration must be filed in a continuation or divisional application naming an inventor not named in the prior application.

To learn more: